Jump to content

Wikipedia:Reference desk/Science: Difference between revisions

From Wikipedia, the free encyclopedia
Content deleted Content added
 
Line 1: Line 1:
<!--- Please DO NOT enter your question at the top here. Put it at the bottom of the page. An easy way to do this is by clicking the "new section" tab ---><noinclude>{{Wikipedia:Reference desk/header|WP:RD/S}}
<noinclude>{{pp-move-indef}}</noinclude>
[[Category:Non-talk pages that are automatically signed]]
{{bots|deny=ClueBot NG}}
[[Category:Pages automatically checked for incorrect links]]
[[Category:Non-talk pages that are automatically signed]]<noinclude>{{Wikipedia:Reference desk/header|WP:RD/S}}
[[Category:Wikipedia resources for researchers]]
[[Category:Wikipedia resources for researchers]]
[[Category:Wikipedia help forums]]</noinclude>
[[Category:Wikipedia help forums]]
[[Category:Wikipedia reference desk|Science]]
[[Category:Wikipedia help pages with dated sections]] </noinclude>


= May 29 =
{{Wikipedia:Reference_desk/Archives/Science/2011 January 15}}


== Elderly digestion ==
{{Wikipedia:Reference_desk/Archives/Science/2011 January 16}}


Older people have more distended intestines due to loss of muscle tone. It also means food is pushed along more slowly along the digestive tract. Does this mean that per ounce of ingested food, elderly people will extract more nutrients?
=January 17=


Of course, if this causes them to eat less, it may not mean more calories absorbed. [[User:Imagine Reason|Imagine Reason]] ([[User talk:Imagine Reason|talk]]) 09:24, 29 May 2024 (UTC)
== Suffocation ==
:I don't know if any of the premises of your argument are correct. But consider this; for about half a billion years our ancestors have had a digestive system. Extracting every usable bit of nutrients from our food has been under strong selection the whole time, because starvation kills and food is limited. Why then would a malfunctioning elderly system do better? <span style="font-family: Cambria;"> [[User:Abductive|<span style="color: teal;">'''Abductive'''</span>]] ([[User talk:Abductive|reasoning]])</span> 18:54, 29 May 2024 (UTC)
::There is an evolutionary pressure to extract every usable bit of nutrients from our food. There's also an evolutionary pressure to keep the power-to-weight ratio of the digestive system high. Wasting some nutrients to keep the digestive system light may be benificial. I'm not suggesting I disagree with your conclusion. [[User:PiusImpavidus|PiusImpavidus]] ([[User talk:PiusImpavidus|talk]]) 19:54, 29 May 2024 (UTC)
:::The human digestive system is far from perfect in extracting calories. [[User:Imagine Reason|Imagine Reason]] ([[User talk:Imagine Reason|talk]]) 21:52, 31 May 2024 (UTC)


== Elvish astronomy ==
Why did humans evolve to feel pain from abnormally high CO<sub>2</sub> levels, rather than abnormally low O<sub>2</sub>? Unlike the former, the latter would allow people to avoid going into areas with high N<sub>2</sub> and little or no O<sub>2</sub> ([[nitrogen asphyxiation]]) --[[Special:Contributions/75.15.161.185|75.15.161.185]] ([[User talk:75.15.161.185|talk]]) 02:55, 17 January 2011 (UTC)


Considering [https://physics.stackexchange.com/questions/122785/could-legolas-actually-see-that-far Legolas' canonical feats], what would be his naked-eye [[limiting magnitude]], assuming seeing conditions in which the average human would get 6.0?
How often did early humans found themselves in such conditions throughout their evolution? [[User:Dauto|Dauto]] ([[User talk:Dauto|talk]]) 03:33, 17 January 2011 (UTC)
:You might find the articles on [[Carotid body]] and [[hypoxia]] interesting. I think the answer is that we didn't really evolve a mechanism to detect low Oxygen levels, but CO<sub>2</sub> is actually toxic, so it was much easier to evolve a mechanism to detect high levels of it. Keep in mind that it is incorrect to assume that EVERY single trait or mechanism is evolved for fitness. There are a lot of traits or mechanisms which are evolved sympathetically or inversely due to another trait which is evolved for fitness. For a very basic example, why aren't antelope legs thicker and stronger and less prone to breakage? Because longer and leaner legs are better at running. In the same way, why did we evolve that CO2 will kill us in the first place? Obviously, we didn't, it's just the result of us evolving to breathe oxygen. [[User:Vespine|Vespine]] ([[User talk:Vespine|talk]]) 03:54, 17 January 2011 (UTC)
::And where on earth do you naturally get an extra high nitrogen level? The only likely place with low oxygen is high altitudes, which are usually very cold too, so no extra protective measures are needed. [[User:Graeme 2|Graeme Bartlett]] ([[User talk:Graeme 2|talk]]) 04:03, 17 January 2011 (UTC)
:::In electronics compartments near anything highly flammable. Many people have died when they go into a compartment to repair electronics without first venting the N<sub>2</sub> and adding O<sub>2</sub>. --[[Special:Contributions/75.15.161.185|75.15.161.185]] ([[User talk:75.15.161.185|talk]]) 04:08, 17 January 2011 (UTC)
::::I suspect that Graeme meant naturally occurring places. Sure, modern humans can create unusual atmospheres, but in order for low oxygen levels to have been an important evolutionary pressure for our species, our ancestors would have had to encounter low oxygen levels with some reasonable frequency. However, the presence of low oxygen levels (without also having high CO2 levels) would seem to be a very rare condition in the natural environment, and hence not something that humans would be expected to have evolved a response to. [[User:Dragons flight|Dragons flight]] ([[User talk:Dragons flight|talk]]) 04:13, 17 January 2011 (UTC)
:::::It should be noted that large amounts of carbon dioxide actually form [[carbonic acid]] when dissolved in water; thus the carbon dioxide literally tends to create [[acidosis]] of the blood. This is a very large perturbation of body chemistry, and the body's response to it really is not all that impressive. To consider how much further evolution could go with such things, consider that [[mosquito]]es are able to ''smell'' carbon dioxide and follow it for about 100 feet to their victims - despite, of course, producing it themselves! [[User:Wnt|Wnt]] ([[User talk:Wnt|talk]]) 08:47, 17 January 2011 (UTC)
:according to our articles both [[Carotid bodies]] and [[aortic bodies]] primarily detect O2, not pH or CO2. So the answer is we already do detect it. [[User:Rmhermen|Rmhermen]] ([[User talk:Rmhermen|talk]]) 16:47, 17 January 2011 (UTC)
::See also [[hypoxemia]], [[hypercapnia]] and [[shallow water blackout]]. ~<font color="blue">[[User:AstroHurricane001/A|A]][[User:AstroHurricane001|H]][[User:AstroHurricane001/D|1]]</font><sup>([[User:AstroHurricane001/T|T]][[Special:Contributions/AstroHurricane001|C]][[User:AstroHurricane001/U|U]])</sup> 01:27, 18 January 2011 (UTC)


(The main reason for this question is that in ''Morgoth's Ring'' an Elvish name for Neptune is given. Though I suppose they might've been using palantíri as telescopes.) [[User:Double sharp|Double sharp]] ([[User talk:Double sharp|talk]]) 09:36, 29 May 2024 (UTC)
=== Tropic response to atmospheric nitrogen? ===
As a rule &ndash; or at least as a rule of thumb &ndash; we mammals are utter crap at evolving new signalling and sensing pathways because we just don't run through generations fast enough. We can tinker with existing pathways and mechanisms, but developing 'new' sensors is improbable virtually to the point of impossibility. So is there anything in our biological heritage that includes a [[tropism|tropic]] response to nitrogen gas? Even more broadly, are there any organisms known which respond actively to concentrations of nitrogen? (I don't count dying/not dying a a 'response' here.) Motile microbes which will move up or down a nitrogen concentration gradient, for instance? [[User:TenOfAllTrades|TenOfAllTrades]]([[User_talk:TenOfAllTrades|talk]]) 15:31, 17 January 2011 (UTC)
:It seems unlikely, given that nitrogen makes up the bulk of the atmosphere. It's hard to do a definitive search, though, because sensors for fixed nitrogen are very common, and I can't think of a way to set up a search that filters them out reliably. (The bacterial systems that fix nitrogen could be thought of as nitrogen gas sensors, but I expect that they always operate at saturation under normal conditions.) [[User:Looie496|Looie496]] ([[User talk:Looie496|talk]]) 18:32, 17 January 2011 (UTC)


:Perhaps elves have [[eagle eye]]s; Tolkein is silent on the issue. [[User:Alansplodge|Alansplodge]] ([[User talk:Alansplodge|talk]]) 11:15, 29 May 2024 (UTC)
:Evolution often finds solutions that are merely "good enough". Detecting lack of oxygen rather than excess CO2 would be better, but it would be very rare that it would actually make a difference to reproductive success. Also, you wouldn't want to lose the ability to detect excess CO2 because there are situations where you get a CO2 build up without a significant drop of oxygen levels (too many people in an enclosed place, dissolved CO2 in a lake suddenly coming out of solution (see [[Limnic eruption]]), etc.). That means you would need both abilities, which is a drain on resources. Being able to detect a lack of oxygen may not be worth the cost. --[[User:Tango|Tango]] ([[User talk:Tango|talk]]) 19:26, 17 January 2011 (UTC)
::He does kind of imply at least that the naked-eye limiting magnitude is greater for Elves than for Men. In ''The Nature of Middle-Earth'', "Dark and Light" it is written: {{tq|The Quendian imagination of the shape of Arda and of the visible Heaven (Menel) above it, was due to the acute minds of a people endowed with sight far keener than the human norm.}} Though this is for the Round World version, in which the Sun and Moon already exist from the beginning. As for quantitative figures, I guess we're stuck with the data point of Legolas counting 105 horsemen from a distance of 24 km.
::Actually, it occurs to me that (perhaps more interestingly than limiting magnitudes), Elves really ''ought'' to be able to resolve the Galilean moons of Jupiter. This provides an alternate solution to the longitude problem, at least if you take the Round World versions. :) [[User:Double sharp|Double sharp]] ([[User talk:Double sharp|talk]]) 15:20, 29 May 2024 (UTC)
:::The Galilean moons would be a cinch for an elf. I had a (human) friend who could do this (he was tested on it several times). Not being particularly interested in astronomy, he only found out in his adult years that this was not usual for most people. {The poster formerly known as 87.81.230.195} [[Special:Contributions/94.2.67.173|94.2.67.173]] ([[User talk:94.2.67.173|talk]]) 19:42, 29 May 2024 (UTC)
::::Hmm, probably also the crescent of Venus should be resolvable for them. This perhaps has implications on the shapes of the Silmarils. (Although in the Round World Version, Venus already exists beforehand, and its identification with Eärendil is said to be mythologising. From the same essay I quoted: {{tq|Certain stars (no doubt those we call planets) and among them especially Venus, which they called ''Elmō'' (and later mythologically ''Eärendil''), they early observed were “wayward” and altered their places with regard to the “farstars” (fixed stars). These they called companions of the Sun and thought them quite small heavenly bodies – derived from the Sun.}}) [[User:Double sharp|Double sharp]] ([[User talk:Double sharp|talk]]) 02:31, 30 May 2024 (UTC)
:The limiting magnitude can be increased with a sharper view. The sharper the view, the smaller the area of the detector (retina) on which the light falls. The background light is fixed per unit of surface area of the detector, so with the signal on a smaller area, less background competes in this area, increasing signal-to-noise. A sharper view can also help to take a faint object out of the glare of a nearby bright object; relevant to see the Galilean moons.
:The other way to increase the limiting magnitude is by increasing sensitivity. No matter how sharp your eyes are, you need a couple of photons before you can see anything. The more photons you detect, the lower the relative [[Poisson noise]]. The sensitivity can be increased by (A) a better detector, detecting a larger fraction of the incoming photons; (B) larger aperture, i.e. a bigger pupil; (C) increasing integration time. Many nocturnal animals (and elves may be somewhat nocturnal) have better, more sensitive detectors, although at a price. The [[tapetum lucidum]] found in many animals reduces resolution somewhat; some species sacrificed colour vision for better low-light vision. Bigger eyes help to see better, but although Tolkien often writes that Elven eyes are keen and fair, he never writes (AFAIK) that they're big. Maybe elves can at will increase the integration time of their eyes. For humans it's fixed at several centiseconds, but if elves can boost it to a full second (they would largely loose the ability to detect motion), seeing Neptune shouldn't be too hard. Still takes a lot of dedication and patience to find out which of those tens of thousands of faint stars slowly moves, but patience is something you should have if you live forever. Ents might disagree. [[User:PiusImpavidus|PiusImpavidus]] ([[User talk:PiusImpavidus|talk]]) 20:55, 29 May 2024 (UTC)
::Why would Ents disagree? They have loads of patience (though it is unclear if they are immortal or just extremely long-lived). [[User:Clarityfiend|Clarityfiend]] ([[User talk:Clarityfiend|talk]]) 22:23, 29 May 2024 (UTC)
:::Ents might disagree with the statement that elves have patience. [[User:PiusImpavidus|PiusImpavidus]] ([[User talk:PiusImpavidus|talk]]) 11:07, 30 May 2024 (UTC)
::Thanks for the details! I guess I'm personally more inclined now towards explaining the Neptune thing by an Elvish invention of the telescope.
::Or maybe the Valar told them where to look, noting that Neptune's magnitude is actually brighter than the most extreme reports of naked-eye viewing of stars. It would be a lot easier to find Neptune if you already know where it is, than to find which of those myriad faint stars is slowly moving. In NoME, ''Elvish Reincarnation'' implies that the Eldar were informed about [[isotope]]s by the Valar, so this isn't unreasonable in-universe. (Though finding that passage makes me amused by the thought of seeing Galadriel's NMR spectra.) [[User:Double sharp|Double sharp]] ([[User talk:Double sharp|talk]]) 04:23, 30 May 2024 (UTC)
::In canon Men are sometimes mistaken for Elves, so there must not be any gross difference in eye size. [[User:Tamfang|—Tamfang]] ([[User talk:Tamfang|talk]]) 21:37, 30 May 2024 (UTC)
:::Assuming a comparable physiology, an Elven retina may pack more sensitive photoreceptor cells, while the lens may have better optical qualities. &nbsp;--[[User talk:Lambiam#top|Lambiam]] 06:08, 31 May 2024 (UTC)
I wouldn't take it for granted that the Solar System outside of Middle Earth is the same as the real one. According to ''[[The Silmarillion]]'' Earth (or "Arda") was explicitly created by God and smaller deities with their omnipotent powers, and so were the peoples of that world (Elves, Men, Dwarves, and surely Hobbits and Ents, too). And the part that would drive mad the astronomers reading that book, the Sun and the Moon were also created by those beings... ages ''after'' the creation of Arda, and ''after'' life existed on that world. [[User:Cambalachero|Cambalachero]] ([[User talk:Cambalachero|talk]]) 13:00, 31 May 2024 (UTC)
:Indeed. But Tolkien planned in his later years to make an altered [[Round World Version of Tolkien's legendarium|Round World Version]] in which the Sun and Moon exist from the beginning of Arda. In that case the Two Trees simply preserve their light as it was before they were tainted by Melkor. As I quoted above, these late rewrites imply that the planets in Tolkien's world are the same ones that we have. :) [[User:Double sharp|Double sharp]] ([[User talk:Double sharp|talk]]) 11:05, 1 June 2024 (UTC)
::Hmm, curious where you heard about these supposed intended revisions to create a lore more consistent with actual astral bodies. I'm a little skeptical without seeing a source on this, just because it doesn't seem to add up with other well-established details. Bear in mind that every element of the legendarium that bears light on the creation and cosmology of Eä was published after J.R.R. Tolkien's death. Christopher Tolkien finished ''The Silmarillion'' by trying to faithfully patch together the content (and fill in gaps himself) using an express desire to have the final work reflect what his father, the original author, would have intended. Nobody would have been in a better position than the younger Tolkien (who made the completion of the legendarium a substantial part of his own life's work and had the fullest access to the existing materials) to know what the elder intended in this respect, and he would have had every opportunity to cause the final work to reflect it, as he did with countless other details. {{pb}} Further, thematically it just doesn't seem to fit: the trees are so fundamental to the cosmonogy of the legendarium, as well as its evolution and eschatology. The trees are part of the more gnostically "pure" version of the world after it was sung into existence out of the higher pleroma by the music of Ainur. The fall of the trees, though instigated by the machinations of Melkor and Ungoliant, are thematically (and arguably psuedo-naturally) the result of an inevitable and unarrestable trend of the world trending away from the direct influence of the Valar and towards a world more defined by physicality and all the ills that come with it. {{pb}} All of this being the result of Melkor's discordant notes, which Eru permitted to remain a part of the song of creation. First the Valar leave Middle Earth and retire to Valinor, and as time wears on, begin making less and less in the way of even indirect influence over it. Next, the Trees are destroyed and the First Age begins. Later the world is reshapped such that Valinor is not even entirely on the same physical plane as the rest of Arda, and reachable only via the Straight Path. The elves diminish and go into the West, returning ultimately to Valinor, the magic and grace of their realms failing and converting them into mundane lands. Meanwhile the light of the trees persists in the silmarils and the phial of Galadriel, bitter sweet echoes of a purer but irretrievable age. Magic fades and Illuvatar's second (and less ethereal) children, humans, inherit the world. {{pb}} You see what I mean? The idea of the world's inclination to a state defined increasingly more by a base, more purely physical state and away from the direct influence of the spiritual animus that gave birth to it is baked into the narrative and the lore, from start to finish. And the trees are the ultimate symbol of the starting point (or at least the start after the music was finished and the Ainur descended into Arda). Besides, the sun and the moon predating the round world just doesn't make much sense: a flat Arda wouldn't be able to rotate on is axis. Mind you, not to imply that there's in way to get the legendarium's cosmology to work with actual astrophyics. Which Tolkien very well knew: all indications are that this was a part of the point. But introducing the sun and the moon before the First Age just makes the discontinuities more obvious and intrusive.{{pb}}All that said, very curious to see where this comes from originally. Certainly there is no shortage of matters that Tolkien went back and forth on over his decades of revisions of the relevant works, nor issues where his son had to make best-guess efforts in choosing among the disparate versions of events. But personally, I tend to doubt that Tolkien seriously considered this particular shake-up. Tonally and in terms of continuity, it just doesn't add up. -sighs a sigh of glutted satisfaction, having sucked out all of the fun the subject matter and wrecked it as surely as a giant spider sucking the magic out a world tree and poisoning it.- ''[[User:Snow Rise|<b style="color:#19a0fd;">S</b><b style="color:#66c0fd">n</b><b style="color:#99d5fe;">o</b><b style="color:#b2dffe;">w</b><b style="color:#B27EB2;">Rise</b>]][[User talk:Snow Rise|<sup><b style="color:#d4143a"> let's rap</b></sup>]]'' 02:51, 3 June 2024 (UTC)
:::{{ping|Snow Rise}} There is a good summary [https://realelvish.net/2024/04/14/it-always-had-been-a-vast-globe/ here], though the primary sources (JRRT's texts) are still in ''[[Morgoth's Ring]]'' (ed. Christopher Tolkien) and ''[[The Nature of Middle-earth]]'' (ed. Carl Hostetter). A scholarly paper about it is [https://dc.swosu.edu/mythlore/vol42/iss1/10/ here].
:::A slightly shorter summary: pre-ROTK, JRRT produced a version of the ''Ainulindalë'' with the changes to the cosmology. Melkor seizes part of the Earth to make the Moon for his stronghold, before the Valar cast him out of it and cleanse it. For the time being, it was only an experiment.
:::But in the late 1950s, JRRT came to believe that the making of the Sun and Moon was too "astronomically absurd" to write in an age when most people believe that the Earth is spherical and is more or less like an island in space. So he came up with a new concept: the "Flat World Versions" are traditions that were handed down by the Númenoreans and then in Arnor and Gondor, that are inescapably blended and confused with Mannish myths. The Elves had their own lore from Valinor that was astronomically and geologically in accord with what we know instead.
:::JRRT describes the new cosmology across some essays published in the aforementioned collections and in an interview. The Sun and Moon were created together with the Earth, and originally they had the Primeval Light, and what Melkor did instead was corrupt them. Middle-earth was then twilit, because Morgoth darkened the Earth with clouds, such that the stars and moon were invisible and the Sun was only a dim twilight (something like the real Venus without the greenhouse effect, I might add). The significance of the Two Trees is that only ''there'' was the Primeval Light preserved, and Varda domed Valinor over to keep Morgoth's corruption out and have it only be lit by the stars. The world was also round ''from the beginning'', but you could not circumnavigate it before the drowning of Númenor because Aman would block the way. The Númenorean Catastrophe removes the inhabitants of Aman from the physical world, though the landmass remains and becomes America after significant geographical upheaval. Thus JRRT writes in these notes {{tq|Aman and Eressëa would be the memory of the Valar and Elves of the former land.}}
:::Crucially, in 1966 ''The Hobbit'' was slightly edited in accordance with this new revision: where once read {{tq|In the Wide World the Wood-elves lingered in the twilight before the raising of the Sun and Moon}} (completely correct per the Flat World version), there now read {{tq|In the Wide World the Wood-elves lingered in the twilight of our Sun and Moon}}. JRRT's late texts published and edited by Christopher in ''[[The Peoples of Middle-earth]]'' sometimes also imply this: the new description of Fëanor's burning of the ships specifies that it was done "in the night", and that "In the morning the host was mustered", which makes no sense in the old cosmology because the Sun wouldn't yet have first risen; and Thingol's throne room of Menelrond is supposed to be based on the domes of Varda (which don't exist before the changes). The heraldry JRRT drew in the 1960s for the House of Finwë is a "Winged Sun", which also makes more sense in the new cosmology because Finwë otherwise could not have seen a Sun that only rose after he died. So the evidence is clear that the elder Tolkien really seriously intended this change.
:::As for why it was not adopted by Christopher: Christopher's commentary on these texts indicates that he seems to have thought it a bad idea on the part of his father. I think Christopher made a sensible call, as going on with the intended changes would require much more editorial intervention than leaving things as they were: it does not seem that JRRT ever ''finished'' the planned rewrites. But from my perspective as a fan, the whole idea is fascinating and allows the amusing conceit of trying to figure out what was really going on behind the scenes, which is why I asked the initial question about how astronomically plausible the reworking was in one aspect (the visibility of Neptune). [[User:Double sharp|Double sharp]] ([[User talk:Double sharp|talk]]) 03:45, 3 June 2024 (UTC)
::::Fascinating! Just when I thought the paratextual narrative of this particular piece of literature couldn't get more complicated! Personally I like the ultimate approach adopted (even beyond the concern you rightly point out of it requiring more intervention--I still think I prefer the narrative irrespective of whether you hypothesize a scenario in which J.R.R. could make the adjustments himself). The Age of the Trees being treated as literal (or at least facially literal from the perspective of the Silmarillion/legendarium's narrative; all of the books are presented to varying degrees as possibly unreliable ancient historical text, after all) just feels very at home with both the broader cosmogony as well as the thematics of the overall work. But then, I have revisited those books so many times over the years (at least compared to my once, ''maybe'' twice and done policy for most literature: soooo many things to read!) that I may have some bias! ''[[User:Snow Rise|<b style="color:#19a0fd;">S</b><b style="color:#66c0fd">n</b><b style="color:#99d5fe;">o</b><b style="color:#b2dffe;">w</b><b style="color:#B27EB2;">Rise</b>]][[User talk:Snow Rise|<sup><b style="color:#d4143a"> let's rap</b></sup>]]'' 16:54, 3 June 2024 (UTC)
:::::{{ping|Snow Rise}} I kind of think of JRRT's problem as being that in seeking increased verisimilitude, he'd gone from wanting to write a mythology to wanting to write a science fiction novel. Only, instead of starting a new science fiction novel and leaving the mythology as it was in its almost-complete state, he ended up trying to turn his mythology into a science fiction novel, with predictable results. But I suppose the whole narrative was too dear to his heart for him to come up with another like it, just like the Silmarilli were to Fëanor. Which is understandable: I share your obsession, and how much greater must it have been for the author? :)
:::::With that said: even in this take, the Age of the Trees would still be literal in the sense that there were luminescent trees in Valinor that got killed by a giant spider. They would only not be literal in the sense that the Sun would still be shining (in its form post-tainting by Melkor) over Middle-earth. Only in the northwest (over Beleriand) is there a volcanic winter due to the fumes of Thangorodrim – and from that, we can see where the idea of the Sun first rising later comes from as an in-universe myth. [[User:Double sharp|Double sharp]] ([[User talk:Double sharp|talk]]) 10:42, 4 June 2024 (UTC)


= May 30 =
::I think that bats, bloodhounds, and star-nosed moles would each disagree with you in their own special way. While it seems like some mammals are "terminally differentiated", reproducing too slowly to evolve quickly or even to prevent losses of useful little things like vitamin C from the genome, we yet see that some of the longest-lived mammals somehow evolved rapidly in form and function to become humans, which was one of evolution's more curious accomplishments. And of course there remains a vast pool of various little shrewlike things that outnumber all the rest of us, just waiting to start down some new evolutionary path and claim a new place in the limelight. (Oh, and note the comment about O2 and the [[carotid body]] above) [[User:Wnt|Wnt]] ([[User talk:Wnt|talk]]) 22:03, 17 January 2011 (UTC)


== Volume of honey in a bee nest ==
:::Detecting N<sub>2</sub> directly would be very difficult/impossible because it is [[Chemical polarity|non-polar]] which means it is difficult for any other molecules to grab hold of, after all, this is why there is so much of it in the atmosphere, because it doesn't react with anything. For species that fix nitrogen using [[nitrogenase]], there is no need to have a tropism to it because N<sub>2</sub> is everywhere. [[User:Smartse|SmartSE]] ([[User talk:Smartse|talk]]) 00:49, 18 January 2011 (UTC)


What is the average volume of honey in a bee nest in the wild? I was able to find information on the average volume of a bee nest, but I know that not all of that volume is honey, of course. Thank you! [[User:HeyArtemis|HeyArtemis]] ([[User talk:HeyArtemis|talk]]) 07:49, 30 May 2024 (UTC)
::::Well, as I wrote above, your own answer explains how it could be done: to build a biological N<sub>2</sub> detector, all you would have to do is couple nitrogenase to a detector for fixed nitrogen, of which there are many examples. But as you also say, such a thing would be useless, because N<sub>2</sub> is everywhere. [[User:Looie496|Looie496]] ([[User talk:Looie496|talk]]) 22:07, 18 January 2011 (UTC)
:[[Apis mellifera|It]] [[Apis dorsata|varies]] [[Apis cerana|alot]]. [[Honey#Production]] has a number for ''Apis mellifera''. [[User:Sean.hoyland|Sean.hoyland]] ([[User talk:Sean.hoyland|talk]]) 08:39, 30 May 2024 (UTC)


== Does light decay? ==
==Future space journeys==
in future the man will find better ways for solve weightloss problem in space jurnays . i forcast that this mystry is hidden in comets
and sattlite and space craft groups will have diffrent missions . we will find severel mystrys of planets and moons with particle sending method that i want to publish . i have good ideas for this. a. mohammadzade <small><span class="autosigned">— Preceding [[Wikipedia:Signatures|unsigned]] comment added by [[User:A.mohammadzade|A.mohammadzade]] ([[User talk:A.mohammadzade|talk]] • [[Special:Contributions/A.mohammadzade|contribs]]) 05:02, 17 January 2011 (UTC)</span></small><!-- Template:Unsigned --> <!--Autosigned by SineBot-->
:Hello, I'm guessing English isn't your first language so apologies if there is a question there which I'm not seeing. The reference desk is for people who seek referenced answers to specific questions relating to science. Do you have a question? [[User:Vespine|Vespine]] ([[User talk:Vespine|talk]]) 05:43, 17 January 2011 (UTC)
:The [[artificial gravity]] article may be of interest to you. [[Special:Contributions/71.198.176.22|71.198.176.22]] ([[User talk:71.198.176.22|talk]]) 06:32, 18 January 2011 (UTC)
that is so ,my mother toungh is turkish ,so i have some difficultys in english . I have write my special questions about solar system
last jan 11 under the subject "any one with new ideas for creation theories of solar system "wich shows my new theory upgrading from new explanation of objects and planets belts and astroids in solar system .
I wrote them not to find replays but for showing nessesity for development and changing in thus theories A.mohammadzade jan 18 iran<span style="font-size: smaller;" class="autosigned">—Preceding [[Wikipedia:Signatures|unsigned]] comment added by [[Special:Contributions/ 78.38.28.3| 78.38.28.3]] ([[User talk: 78.38.28.3|talk]]) 04:03, 18 January 2011</span><!-- Template:UnsignedIP2 -->


Let's say that an object, such as a star, emits a beam of light and it moves across the space. It goes at the speed of light and, unless it reaches an opaque object, it would keep going... for how much time? Forever? Or is there a point when light would simply dimish and disappear? [[User:Cambalachero|Cambalachero]] ([[User talk:Cambalachero|talk]]) 19:53, 30 May 2024 (UTC)
== Electromagnet. ==
: No.
: It may react with matter, if it encounters any. A flux of many photons will spread out to a larger volume and so the intensity (number of photons passing through an area) will diminish in accord with the [[inverse square law]]. But light passing through a vacuum does not 'decay' or have a limit on its range.
: BTW, this theory that light can only travel a few thousand miles before 'running out' is part of flat earther canon for some models, as an explanation of how nighttime happens. But then they're flat earthers. [[User:Andy Dingley|Andy Dingley]] ([[User talk:Andy Dingley|talk]]) 20:11, 30 May 2024 (UTC)
::The idea of [[tired light]] used to be somewhat popular but has been entirely discarded by now. --[[User:Wrongfilter|Wrongfilter]] ([[User talk:Wrongfilter|talk]]) 20:22, 30 May 2024 (UTC)
:::I'm afraid it is a zombie idea.<sup>[https://www.uottawa.ca/about-us/media/news/new-research-suggests-our-universe-has-no-dark-matter]</sup> &nbsp;--[[User talk:Lambiam#top|Lambiam]] 05:58, 31 May 2024 (UTC)
:I guess the apparent reality that things in motion continue in motion forever by default, if unperturbed, seems a bit unnatural when you live in a macroscopic world. [[User:Sean.hoyland|Sean.hoyland]] ([[User talk:Sean.hoyland|talk]]) 07:32, 31 May 2024 (UTC)
::Correct. If that wasn't the case, we wouldn't be able to see stars that are too far away. We are currently seeing stars so far away that they aren't just stars. They are galaxy or similar objects so far away that they show up as one little blob of light. The limit is not how far light can travel before pooping out. It is how old the stars are. For example, a 100 year old star that is 90 light years away would not be visible becuase the original light hasn't reached us yet (ignoring the complication of direction of relative movement). So, the argument is that the night sky should be flooded with light from all directions. And, it is. It only looks black in a relative sense. [[Special:Contributions/12.116.29.106|12.116.29.106]] ([[User talk:12.116.29.106|talk]]) 14:51, 4 June 2024 (UTC)
:::I imagine it would be much worse than not being able to see stars. If excitations of quantum fields got tired we would be in big trouble. [[User:Sean.hoyland|Sean.hoyland]] ([[User talk:Sean.hoyland|talk]]) 15:05, 4 June 2024 (UTC)


= May 31 =
Define '''Electromagnet'''.[[User:Chetan the Champ|Chetan the Champ]] ([[User talk:Chetan the Champ|talk]]) 10:29, 17 January 2011 (UTC)
==Is redshift calculated differently for different spectra?==
:Do you need a dictionary? Here: [[wikt:electromagnet|Electromagnet]]. Or is this a homework question? [[User:Ariel.|Ariel.]] ([[User talk:Ariel.|talk]]) 11:03, 17 January 2011 (UTC)
I ask because I came across an article, [[TXS 1545-234]], in the course of regular gnoming. The article claims this radio galaxy to be one of the most distant known objects, but its redshift is only around {{math|z}} = 2.754 (which I take to be measured from radio emissions). It does appear in [https://ned.ipac.caltech.edu/level5/Sept08/Miley/Miley9.html this source] {{nowrap|({{para|date}}}} at least 2006), and I'm wondering whence the claim of such great distance paired with such pedestrian redshift. I'm not able to understand our article [[Redshift]]. Also, if anyone has any ideas about how to de-orphan the article linked, please do have at. [[User:Folly Mox|Folly Mox]] ([[User talk:Folly Mox|talk]]) 11:31, 31 May 2024 (UTC) {{small|{{ins|edited 11:52, 31 May 2024 (UTC)}}}}
:It is certainly not one of the most distant galaxies known. The author of the article, {{ping|Galaxybeing}}, should explain why they think it is. --[[User:Wrongfilter|Wrongfilter]] ([[User talk:Wrongfilter|talk]]) 12:12, 31 May 2024 (UTC)
::They appear to have made a similar claim at [[Special:Permalink/1226518102|MRC 0406-244]] ({{math|z}} = 2.44) although thankfully not at another recent creation, [[Special:Permalink/1226388188|QSO J0100-2708]] ({{math|z}} = 3.52). [[User:Folly Mox|Folly Mox]] ([[User talk:Folly Mox|talk]]) 12:57, 31 May 2024 (UTC)
:The redshift, a [[dimensionless quantity]], is the same for the whole spectrum emitted by an object. Compared to TXS 1545-234, [[JADES-GS-z13-0]] is thought to be more than three times as far away from us. &nbsp;--[[User talk:Lambiam#top|Lambiam]] 12:37, 31 May 2024 (UTC)
::Thanks both for your answers, and confirming my suspicion that the claim was merely incorrect. [[User:Folly Mox|Folly Mox]] ([[User talk:Folly Mox|talk]]) 13:00, 31 May 2024 (UTC)
:A redshift of 2.754 is pretty far away, but stating it's one of the most distant known objects is overstating it a bit. Distances to far-away galaxies are normally expressed in redshift, as redshift is directly observable, in contrast to distance, which depends on a model of the expansion of the universe. But when using redshift as a distance measure, one has to keep in mind that it's highly non-linear. Also, distance is a bit of a strange concept when dealing with these cosmologically distant objects. Are we talking about the distance today, or at the time the light was emitted, or the distance travelled by the light? That last at least has some relevance as it translates to the time that the light has travelled and therefore when it was emitted. JADES-GS-z13-0 may be several times farther away today than TXS 1545-234, but most of that is thanks to the expansion of the universe after the light was emitted. When the universe was young and small (although still infinite), it expanded fast in absolute numbers (percentage per year). In light travel distance, the difference isn't so much.
:These objects can be studied to learn more about the early universe. For that, knowing the distance to us is not so important; we want to know about the distance (or time) to the Big Bang. At some point, distances (times) to the Big Bang are known more accurately than distances (times) to us. In any case, the redshift tells us immediately that the universe expanded by a factor of five between the times when the light of JADES-GS-z13-0 was emitted and when the light of TXS 1545-234 was emitted. That puts JADES-GS-z13-0 a lot closer to the Big Bang, although only a small fraction farther from us. [[User:PiusImpavidus|PiusImpavidus]] ([[User talk:PiusImpavidus|talk]]) 11:05, 1 June 2024 (UTC)
::On a tangent but an amusing use (abuse?) of "red shift": chemists use red shift generically to mean "moves to lower energy". Even in the IR part of the spectrum, the term red shift would be used to describe the shift of a band to lower E, say 2000 to 1950 cm-1. This language is of course strange because, formally speaking, a shift toward red for an IR band would mean a shift to higher E. Just sayin'.--[[User:Smokefoot|Smokefoot]] ([[User talk:Smokefoot|talk]]) 17:11, 1 June 2024 (UTC)
:::Radioastronomers apply the same "abusive" terminology. We see this linguistic phenomenon also in uses of the verb "to dial", as in the advise to "<u>dial</u> 9-1-1 for any emergency" given to users of smartphones with touch screens. (Using the original rotary sense of the verb in connection with the casual parlance of [https://www.nbcnews.com/politics/politics-news/rudy-giuliani-butt-dials-nbc-reporter-heard-discussing-need-cash-n1071901 "butt dial"] results in the unfortunate mental image of Giuliani twerking.) Other examples are referring to cotton bed sheets as "linens", or (in the US) to stainless-steel knives, forks and spoons as "silverware", and the computer-graphics terminology calling a screen region a "canvas". I think there is a learned term for this phenomenon if the sense of a word getting abstracted from the physical embodiment after which it was originally named. &nbsp;--[[User talk:Lambiam#top|Lambiam]] 05:29, 2 June 2024 (UTC)
:::[[File:JADES-GS-z13-0.png|thumb|JADES-GS-z13-0]]
:::Astronomers talk about redshift if it goes to longer wavelengths, in radio, IR, UV etc., and about blueshift if it goes to shorter wavelengths, in radio, IR, UV etc. Things can also redshift past red. See how red that distant object in this picture is? It's a feature coming from UV, shifted to IR. The [[Lyman-alpha]] absorption line is at 121.6&nbsp;nm, here broadened into a [[Gunn–Peterson trough]], redshifted to 1.6&nbsp;μm, between the F150W and F200W filters of the camera in JWST. It's how they make a first estimate of the redshift, based on broadband images. A precise number follows later from spectroscopy, but takes far more observing time, so this is only done for the most promising targets. [[User:PiusImpavidus|PiusImpavidus]] ([[User talk:PiusImpavidus|talk]]) 10:18, 2 June 2024 (UTC)


== Bread ==
== Physics problem ==


What are the health problems from eating stale whole wheat or seed bread? If there is no mold on it and it was in the fridge? --[[Special:Contributions/41.213.125.249|41.213.125.249]] ([[User talk:41.213.125.249|talk]]) 10:40, 17 January 2011 (UTC)
A shop sign is made of a panel that protrudes slightly from the wall on which it is hung, forming an angle of 5° with it. It is 0.74 m tall and has a mass of 8.9 kg. The upper side of the panel is attached to the wall by two cables, one from the right side and one from the left side. Find the tension of the 2 cables [[Special:Contributions/78.211.54.11|78.211.54.11]] ([[User talk:78.211.54.11|talk]]) 19:40, 31 May 2024 (UTC)
:Why should there be health problems? Even if there was mold on it it would probably be harmless (most, but not all, molds are harmless when eaten (as opposed to inhaled)). What do you mean by "seed bread" - all bread is made from seeds. [[User:Ariel.|Ariel.]] ([[User talk:Ariel.|talk]]) 11:00, 17 January 2011 (UTC)
::The term probably refers to [[Whole grain]] bread. It may also refer to [[Seed cake]]. Not sure which the OP means. Regarding health effects, presuming that there isn't any alergic concerns (i.e. you wouldn't have negative health effects from eating it fresh) and it isn't rotten or spoiled (as the OP says, no mold) then stale bread is just "dried out", i.e. slightly less water than fresh bread. I would say that such bread is perfectly safe, again with the caveats that there is no spoilage and there is no allergy concerns. --[[User:Jayron32|<font style="color:#000099">Jayron</font>]]'''''[[User talk:Jayron32|<font style="color:#009900">32</font>]]''''' 13:59, 17 January 2011 (UTC)
::: I regularly eat soda-bread which contains sunflower seeds, linseeds and sesame seeds. Maybe something like this is what the OP is referring to. I agree that there are few health risks in eating stale bread, with or without seeds. Our stale bread (not the soda bread) is made into breadcrumbs for use in other dishes. I'm still here! Oops, sorry that's [[WP:OR]]. [[User:Richard Avery|Richard Avery]] ([[User talk:Richard Avery|talk]]) 14:16, 17 January 2011 (UTC)


:<small>Original in Italian, now edited by IP to English.</small>Translates as: "A shop sign consists of a panel that protrudes slightly from the wall on which it hangs, forming an angle of 5° with it. It is 0.74 m high and has a mass of 8.9 kg. The upper end is attached to the wall by two cables, one on the right side and one on the left side. Find the tension in the 2 cables."
:Stale white bread should be made into [[bread and butter pudding]]. [[User:DuncanHill|DuncanHill]] ([[User talk:DuncanHill|talk]]) 16:16, 17 January 2011 (UTC)
:IP editor: as it says at the top of this page, we don't answer homework questions and what we do answer should preferably be asked in English. [[User:Michael D. Turnbull|Mike Turnbull]] ([[User talk:Michael D. Turnbull|talk]]) 19:46, 31 May 2024 (UTC)
::{{small|Asking homework questions also causes tension. ←[[User:Baseball Bugs|Baseball Bugs]] <sup>''[[User talk:Baseball Bugs|What's up, Doc?]]''</sup> [[Special:Contributions/Baseball_Bugs|carrots]]→ 01:15, 1 June 2024 (UTC)}}
::Doesn't the tension depend on the angle the cables (idealized as straight line segments) make with the panel? &nbsp;--[[User talk:Lambiam#top|Lambiam]] 02:53, 1 June 2024 (UTC)
:::Correct. The answer to the question as asked is a curve of tension vs cable length or vertical location. [[User:Greglocock|Greglocock]] ([[User talk:Greglocock|talk]]) 22:56, 1 June 2024 (UTC)
[[File:The hanging sign for 'Fieldys' public house - geograph.org.uk - 5284520.jpg|thumb|left|Is it like this? [[User:Graeme Bartlett|Graeme Bartlett]] ([[User talk:Graeme Bartlett|talk]]) 22:42, 1 June 2024 (UTC)]][[File:Tension of cable in hanging sign.svg|thumb|or is it like this? [[User:Graeme Bartlett|Graeme Bartlett]] ([[User talk:Graeme Bartlett|talk]]) 22:22, 1 June 2024 (UTC)]]
{{clear}}
[[File:Statica-balk1.png|thumb|100px|like this but not equilateral]]
In my interpretation of the problem – which is not necessarily the intended one – the lower edge of the panel is attached to the wall and the panel can rotate along that edge, like in the "or is it like this?" diagram, which has an exaggerated thickness for the panel. Unlike that diagram, the panel does not stick out at a 90° angle but is standing almost upright. Also, the cables are attached to the upper edge. So it is more like the situation here to the right, but instead of a bottom 60° angle we have a 5° angle. Not enough info has been given to determine the other angles. &nbsp;--[[User talk:Lambiam#top|Lambiam]] 04:45, 2 June 2024 (UTC)<br>P.S. Another way to frame the problem could have been in terms of a [[:File:Drawbridge.gif|drawbridge]]. &nbsp;--[[User talk:Lambiam#top|Lambiam]] 04:53, 2 June 2024 (UTC)
:<small> The answer is 42 (glorps). I'll leave it to you to figure out the conversion factor to mks. [[User:Clarityfiend|Clarityfiend]] ([[User talk:Clarityfiend|talk]]) 12:06, 3 June 2024 (UTC) </small>
<small><PRE> |
|'.
| '.
| '.
| '.
| '.
| ::
| ::
|< 5deg >::
| ::
| ::
| ::.
| :: .
| :: .
| :: .
| :: .
|:: V
|* 8.9kg


Equate turning moments at *
::[[Rye bread]] and other breads infected by ''fungus'' could historically result in [[Ergotism]]. ~<font color="blue">[[User:AstroHurricane001/A|A]][[User:AstroHurricane001|H]][[User:AstroHurricane001/D|1]]</font><sup>([[User:AstroHurricane001/T|T]][[Special:Contributions/AstroHurricane001|C]][[User:AstroHurricane001/U|U]])</sup> 01:23, 18 January 2011 (UTC)
clockwise = anticlockwise
8.9 kg x sin(5 deg) x 0.74m / 2 = t/2 x 0.74m
where t = tension in each of 2 cables
= 8.9 sin(5 deg)
= 0.775686... kg</PRE> [[User:Philvoids|Philvoids]] ([[User talk:Philvoids|talk]]) 13:16, 3 June 2024 (UTC)</small>


:Shouldn't there be a sine or cosine factor in the anticlockwise term? Imagine the cables being attached to the wall very close to the <tt>*</tt>. Consider dU/dL, where U is potential energy of the panel and L is the length of the cables. &nbsp;--[[User talk:Lambiam#top|Lambiam]] 15:27, 3 June 2024 (UTC)
== gastric acid ==
::Although not accurately shown in the ASCII sketch my calculation presumes that the two cables are at right angles to the sign, are parallel and are fastened to the wall separately at two points. This minimises the cable lengths and tensions. [[User:Philvoids|Philvoids]] ([[User talk:Philvoids|talk]]) 10:28, 4 June 2024 (UTC)
:::The homework problem did not specify where the cables are attached or that the tension should be minimized, making it unsolvable for the average student. For an old-fashioned drawbridge over a castle moat, as in the animation, a better choice is to place the attachment point of the cables at a distance from the hinge equal to the length of the bridge, making the triangle isosceles. Otherwise, the tension grows without bounds as the bridge near a vertical position. &nbsp;--[[User talk:Lambiam#top|Lambiam]] 15:37, 4 June 2024 (UTC)
::::I could not find an ASCII symbol for a hinge but installing one at the base of the sign is a well thought mechanical improvement that can stop the thing flapping about in a wind. This reference desk cannot grant permission to construct a drawbridge with walkway at this location and the shop owner of the wall would likely protest at your plan to cut a hole for passage. Unless he is a herder of sheep or other small animals and himself less than 74cm tall and sees some advantage. The OP doesn't ask for the cable tension when the sign/bridge is vertical but we can say there are two cases: ''Case #1:'' The hinge is broken or absent. t = 8.9/2 = 4.45 kg plus distributed weight of the cable itself; ''Case #2:'' The hinge takes the weight, t = 8.9 sin (0) so both cable lengths and tensions are zero. In that case it would be simpler just to nail the sign to the wall. Or put out a call to [[Banksy]] who can save us the cost of the sign, usually upset someone and greatly increase the value of the wall. [[User:Philvoids|Philvoids]] ([[User talk:Philvoids|talk]]) 18:39, 4 June 2024 (UTC)
:::::For all we know, there is already an opening behind the sign for letting the proprietor's homing pigeons in. If they are equipped with an RFID chip, the sign can be let down automatically on their arrival. &nbsp;--[[User talk:Lambiam#top|Lambiam]] 05:39, 5 June 2024 (UTC)
::::::<small>Pigeon RFIDs can be compromised by malicious [[Yinpterochiroptera]] and [[Yangochiroptera]]. Members of this notorious "YinYangBat Gang" equipped with ultrasound-to-UHF converters can mount [[Spoofing attack]]s on the shared ID frequency. An ad hoc [[Aposematism|aposematic]] remedy such as a wall painting of a cat with the sign text "I EAT BATS" does not alone give security. To this end all pigeons must be urged to register a pass-coo that is less obvious than the too common "Coo Coo". [[User:Philvoids|Philvoids]] ([[User talk:Philvoids|talk]]) 10:28, 5 June 2024 (UTC)</small>


= June 2 =
do all other animals use HCl in their stomach as we do? <small><span class="autosigned">— Preceding [[Wikipedia:Signatures|unsigned]] comment added by [[User:Sina-chemo|Sina-chemo]] ([[User talk:Sina-chemo|talk]] • [[Special:Contributions/Sina-chemo|contribs]]) 11:09, 17 January 2011 (UTC)</span></small><!-- Template:Unsigned --> <!--Autosigned by SineBot-->
:Yes, it does seem so (although I can't say with 100% certainty). Chlorine is very common in the environment - from ordinary salt, so it gets used as the acid plus when it's time to neutralize the acid it makes ordinary salt, and salt is used for other things as well (sodium is the [[Action potential|main driver in nerves]]). However not all animals have stomach acid, acid breakdown is not the only form of digestion. [[User:Ariel.|Ariel.]] ([[User talk:Ariel.|talk]]) 11:52, 17 January 2011 (UTC)


== Why females produce androgens ==
:<s>[http://insectspedia.blogspot.com/2010/10/digestion.html No]: ''In most insects, midgut pH is either mildly acidic or neutral. Lepidopteran and trichopteran larvae, scarabaeid beetles, and nematoceran flies have alkaline midguts, ...''. --[[User:TotoBaggins|Sean]] 15:27, 17 January 2011 (UTC)</s>
::Your reference to the midgut confuses the issue -- humans also have basic midguts. HCl is secreted in the stomach, but bicarbonate is secreted from the pancreas, and a look at our [[midgut]] and [[foregut]] articles seems to establish the change of ''foregut'' to ''midgut'' at the point of fusion with the [[bile duct]] -- looking at the bile duct article reveals that this duct meets with the pancreatic duct and enters the [[duodenum]] at the [[ampulla of Vater]]. Thus, insects are hardly unique in having basic midguts -- I'd say humans and most mammals do to. '''[[User:DRosenbach|<span style="color:#006400">DRosenbach</span>]]''' <sup>([[User_talk:DRosenbach|<span style="color:#006400">Talk</span>]] | [[Special:Contributions/DRosenbach|<span style="color:#006400">Contribs</span>]])</sup> 19:17, 17 January 2011 (UTC)


If human embryons of both sexes start off from a female blueprint and given that females lack the male Y chromosome, how it came that women also produce [[androgens]] (even if in small quantity), with related limb and facial hair? [[Special:Contributions/212.180.235.46|212.180.235.46]] ([[User talk:212.180.235.46|talk]]) 19:59, 2 June 2024 (UTC)
== Vikrell wha ==
:The article you linked says that the ovaries also produce androgens. ←[[User:Baseball Bugs|Baseball Bugs]] <sup>''[[User talk:Baseball Bugs|What's up, Doc?]]''</sup> [[Special:Contributions/Baseball_Bugs|carrots]]→ 22:33, 2 June 2024 (UTC)
:Biological systems weren't built by any sort of logical designer. They in no way resemble a computer program, a computer, or, for that matter, anything else in the universe. In the case of androgens, the article mentions that androgens are the precursors to estrogens. Males need estrogens too, btw. All these are [[steroids]], which are fundamental to life and are derived from [[cholesterol]]. <span style="font-family: Cambria;"> [[User:Abductive|<span style="color: teal;">'''Abductive'''</span>]] ([[User talk:Abductive|reasoning]])</span> 23:18, 2 June 2024 (UTC)
::"Biological systems weren't built by any sort of logical designer." Which is why so-called "[[intelligent design]]" is just nonsense. The "design" is emphatically not "intelligent". --[[User:Khajidha]] ([[User talk:Khajidha|talk]]) ([[Special:Contributions/Khajidha|contributions]]) 12:07, 3 June 2024 (UTC)
:::I'm not sure biology and computing are as far apart as they used to be now that systems essentially create the gigantic opaque function that transforms input to output themselves in response to their environment/what they have seen and the objectives. Generative adversarial networks for example seem a bit closer to biology than systems used to be. [[User:Sean.hoyland|Sean.hoyland]] ([[User talk:Sean.hoyland|talk]]) 13:06, 3 June 2024 (UTC)
::::Not in a billion years. <span style="font-family: Cambria;"> [[User:Abductive|<span style="color: teal;">'''Abductive'''</span>]] ([[User talk:Abductive|reasoning]])</span> 17:51, 3 June 2024 (UTC)
:::::The technology for [[synthetic biology]], still in its infancy, is advancing with large strides. Whether you like it or not, sooner rather than later it will become possible to design and create complete viable and functioning biological organisms. &nbsp;--[[User talk:Lambiam#top|Lambiam]] 06:40, 4 June 2024 (UTC)
::::::Further, the idea that it is so complicated that nobody understands what it does because it thinks for itself is a farce. No matter what is being used for the computing hardware, be it electronic or biological, the mechanism of operation is very well understood by the engineers who developed it. It just sounds cool to say that it is beyond comprehension. It doesn't sound cool to say that the engineers understand it very well and could trace input through to the output if they wanted to, but simply don't care to do because they have other things to work on. [[Special:Contributions/12.116.29.106|12.116.29.106]] ([[User talk:12.116.29.106|talk]]) 14:42, 4 June 2024 (UTC)
:::::::Tracing the input through to the output is IMO not a helpful concept. Not only do we not know why the most advanced chess or go playing programs make certain surprising moves, but it is not even clear what it means to "understand" why they did this. The computing platform performs a calculation with a certain outcome. The engineers can perform the same calculation by hand, or using abaci, and if they make no mistake they may arrive at the same result in a few billion years: 42. But can they say more than that the answer is 42 because this is the consequence of the rules applied to the input? They knew that already. If someone wants to know ''why'' it is the consequence, they can tell them to repeat the calculation. &nbsp;--[[User talk:Lambiam#top|Lambiam]] 15:51, 4 June 2024 (UTC)
:::::::IP, LLMs for example involve activations in very high dimensional spaces. Trying to map those activation patterns to things we can understand, like concepts etc., is the whole field of interpretability, and it is in its infancy (and safety critical). Engineers are still far from understanding why input A to a model is transformed into output B. If this is something that interests you, have a look at the work being done in Anthropic's lab. [[User:Sean.hoyland|Sean.hoyland]] ([[User talk:Sean.hoyland|talk]]) 17:16, 4 June 2024 (UTC)
::::::::Overgrown Markov Chain models are pretty much useless in any technical field. BTDT [[User:Greglocock|Greglocock]] ([[User talk:Greglocock|talk]]) 04:48, 5 June 2024 (UTC)
:::::::::I guess that's one of the reasons why "Attention Is All You Need" turned out to be such a great title for a paper. [[User:Sean.hoyland|Sean.hoyland]] ([[User talk:Sean.hoyland|talk]]) 08:39, 5 June 2024 (UTC)


= June 5 =
whats Vikrell <small><span class="autosigned">— Preceding [[Wikipedia:Signatures|unsigned]] comment added by [[User:Tommy35750|Tommy35750]] ([[User talk:Tommy35750|talk]] • [[Special:Contributions/Tommy35750|contribs]]) 17:19, 17 January 2011 (UTC)</span></small><!-- Template:Unsigned --> <!--Autosigned by SineBot-->
:[http://www.kohler.com/video/get.do?top=1&mid=0&vidid=85733311001 Resin & filler]. --[[User:TotoBaggins|Sean]] 17:47, 17 January 2011 (UTC)
:e/c Any context? Can't find anything about it on Wikipedia, but it appears to be a product name by the Sterling (Kohler) plumbing company for a range of bathroom and kitchen products: see [http://www.google.com.au/search?hl=en&safe=off&client=firefox-a&hs=kaE&rls=org.mozilla%3Aen-GB%3Aofficial&q=sterling+kohler+vikrell&aq=f&aqi=g1g-v1&aql=&oq=]. --[[User:Jjron|jjron]] ([[User talk:Jjron|talk]]) 17:48, 17 January 2011 (UTC)
: " Vikrell is a solid composite material made of resins, fiberglass and filler that is exclusive to Kohler Company." [http://www.sterlingplumbing.com/baths-and-showers/shower-stalls/] (See "shower stalls FAQs" box at bottom right of page). It apparently has integral pigment rather than a paint-like top layer, hence chips don't show. [[Special:Contributions/81.131.68.227|81.131.68.227]] ([[User talk:81.131.68.227|talk]]) 17:52, 17 January 2011 (UTC)


==Prostheses==
:It's a brand name for a [[Solid surface]] countertop material. [[User:Ariel.|Ariel.]] ([[User talk:Ariel.|talk]]) 20:32, 17 January 2011 (UTC)


what are the matrials uesed in implantable artificial prostheses, such as artificial hearts and small-diameter blood vessels; in the engineering of living tissues ? -- 196.153.184.240
== Gaseous compounds without gaseous elements ==


Are there any chemical compounds which are gaseous at STP, despite containing no chemical element which is gaseous at STP? --[[Special:Contributions/84.61.155.161|84.61.155.161]] ([[User talk:84.61.155.161|talk]]) 17:37, 17 January 2011 (UTC)
: Heart valves usually made from [[pyrolytic carbon]]. https://www.ncbi.nlm.nih.gov/pmc/articles/PMC10034107/#:~:text=Mechanical%20valves%2C%20usually%20made%20from,stroke%20(3%2C%204). [[Special:Contributions/41.23.55.195|41.23.55.195]] ([[User talk:41.23.55.195|talk]]) 06:13, 5 June 2024 (UTC)
:Not that I can think of, but [[carbon disulfide]] (b.p. 46.3&nbsp;°C, 115&nbsp;°F) comes pretty close. [[User:Physchim62|Physchim62]] [[User talk:Physchim62|(talk)]] 19:07, 17 January 2011 (UTC)
::[[Methyl bromide]] only works when you consider [[methanol]] an "element" (which it isn't), or [[phosphorus tribromide]] which only produces gaseous [[fume]]s during [[hydrolysis]]. Possibly [[:wikt:sulfur dibromide|sulfur dibromide]]? ~<font color="blue">[[User:AstroHurricane001/A|A]][[User:AstroHurricane001|H]][[User:AstroHurricane001/D|1]]</font><sup>([[User:AstroHurricane001/T|T]][[Special:Contributions/AstroHurricane001|C]][[User:AstroHurricane001/U|U]])</sup> 01:20, 18 January 2011 (UTC)
:::Sulfur dibromide doesn't exist at STP. The next nearest I can find is [[boron tribromide]] (b.p. 91.3&nbsp;°C, 196&nbsp;°F). [[User:Physchim62|Physchim62]] [[User talk:Physchim62|(talk)]] 12:19, 18 January 2011 (UTC)


:Many different materials are used depending on requirements. For details see [[prosthesis]] and the many links therefrom.[[User:Shantavira|Shantavira]]|[[User talk:Shantavira|<sup>feed me</sup>]] 15:30, 5 June 2024 (UTC)
== Citric acid in fruit ==


= June 6 =
My family has been kicking around a recipe for corn cob jelly, and our most recent attempt at making it was somewhat of a flop. According to the package, pectin requires acid to work, and the typical way of doing this is to add lemon juice. The problem is that corn cobs don't make for a very strong flavor and the end product was rather more lemon-tasting than I'd like. Other than laboratory supply of citric acid (or hydrochloric for that matter!) is there a source of food-grade acid that isn't quite as strong a flavor as lemon juice? I was thinking vinegar, but that's unlikely to work well with a sweet product like a jelly. [[User:SDY|SDY]] ([[User talk:SDY|talk]]) 20:16, 17 January 2011 (UTC)
:I don't know where you live but in NZ you can buy [[citric acid]] and [[tartaric acid]] in most supermarkets. However from my experience in making [[paneer]], citric acid still imparts a fairly strong taste. Haven't though of trying tartaric acid for some reason. You could try various acidic fruits I guess. [[Pineapple]] or green [[kiwifruit]] perhaps? Perhaps even [[lime]] will be a taste you find okay. While looking in to tartaric acid for making paneer I found this [http://sphinxsai.com/july-sept_2010_vol2.3/pharmtech/pharmtechvol2.3july-sept210/PT=41%20_1916-1923_.pdf] where they also tried [[lactic acid]] and [[ascorbic acid]] (you'll want to be careful you don't overdose on [[vitamin C]] I guess). Finally you may want to do a few tests, perhaps you're using way more lemon juice then you need? [[User:Nil Einne|Nil Einne]] ([[User talk:Nil Einne|talk]])
::C is water soluble and it's very hard to get an overdose. A and D and the other fat-solubles are the problems for overdose. I'll have to look at a different grocery store. Safeway (US, Washington state) doesn't carry citric and tartaric from what I've seen, though I guess I've never asked. [[User:SDY|SDY]] ([[User talk:SDY|talk]]) 20:46, 17 January 2011 (UTC)
::: The vitamin C article says it exhibits remarkably low toxicity, the LD50 for an average person would be over 800 grams, and "The mechanism of death ... may be more mechanical than chemical". :) [[Special:Contributions/81.131.16.71|81.131.16.71]] ([[User talk:81.131.16.71|talk]]) 20:58, 17 January 2011 (UTC)
::::Okay no concerns for ascorbic acid then so that's one possibility. I should clarify the citric acid and tartaric acid I'm thinking of is normally available in crystalline form, see [http://www.foodtown.co.nz/Shop/SearchProducts?search=acid].
::::This result from a search for 'where to buy citric acid' may be helpful [http://forums.moneysavingexpert.com/showthread.php?t=237548] (the purpose there may not be food related but most of the suggestions sound like they will be fine). In particular a brewing shop may be something to look in to. I know someone who makes their own energy gels and citric acid is used, so perhaps a health food or sports/exercise supplement shop will also have something (although they may just want to sell you premade stuff). [http://www.jibbering.com/sports/gels.html] (UK apparently) suggests a home brewing shop as well hence my recommendation.
::::P.S. Reading the forum suggestions a little more closely, it also sounds like you may not want to (try to) purchase too large quantities...
::::[[User:Nil Einne|Nil Einne]] ([[User talk:Nil Einne|talk]]) 21:50, 17 January 2011 (UTC)
:Cheese-making supply shops sell citric acid. --Sean <span style="font-size: smaller;" class="autosigned">—Preceding [[Wikipedia:Signatures|unsigned]] comment added by [[Special:Contributions/208.54.44.51|208.54.44.51]] ([[User talk:208.54.44.51|talk]]) 00:25, 18 January 2011 (UTC)</span><!-- Template:UnsignedIP --> <!--Autosigned by SineBot-->


== Health risk of [[taurine]] consumption ==
== Flash flood tanks ==


Realistically, how worried should people be about the health risks of consuming taurine as a supplement? I only ask because half the literature says they find it helpful and beneficial at some unknown dosage, while the other half says it is potentially carcinogenic and could contribute to colon cancer. As a layperson, I find this very confusing. Some of the literature says it could be simply a matter of dosage, but nobody seems to know what the safe or harmful limits are. Can anyone offer some risk analysis devoid of emotion? Should we avoid anything with taurine in it, or not worry at all about it? [[User:Viriditas|Viriditas]] ([[User talk:Viriditas|talk]]) 01:27, 6 June 2024 (UTC)
Can a flash flood carry and destroy an army tank? --[[Special:Contributions/109.78.87.34|109.78.87.34]] ([[User talk:109.78.87.34|talk]]) 21:26, 17 January 2011 (UTC)
:I don't see why it couldn't carry one away. Flash floods can remove houses from their foundations, and in terms of absolute weight, a house is heavier than a tank. "Destroy" may be a harsh word, but I could see a tank being lifted off its treads and moved by such a flood. --[[User:Jayron32|<font style="color:#000099">Jayron</font>]]'''''[[User talk:Jayron32|<font style="color:#009900">32</font>]]''''' 21:34, 17 January 2011 (UTC)
::From [[M4 Sherman]]: 66,800 pounds / (5.84 m * 2.62 m * 2.74 m) = .723 g/cm<sup>3</sup> leading to the surprising result that tanks float. Of course that assumes that it's a cube, which it isn't. It's probably denser than that, but it's still reasonably close to the density of water, so water will have no trouble carrying it. Houses float too BTW, that's the main reason flash floods can move them. [[User:Ariel.|Ariel.]] ([[User talk:Ariel.|talk]]) 21:57, 17 January 2011 (UTC)


:As far as Wikipedia is concerned, [[WP:MEDRS]] applies. That means that the minimum quality level for [[WP:RS]] making medical claims are systematic reviews indexed for MEDLINE (there are some exceptions from this indexation, but generally speaking MEDLINE is the gold standard). [[User:tgeorgescu|tgeorgescu]] ([[User talk:tgeorgescu|talk]]) 03:29, 6 June 2024 (UTC)
: I don't believe so, or at least not to a catastrophic degree. Many modern main battle tanks are capable of [[Amphibious_vehicle#Deep_fording|deep wading]], which means they can drive through water that's as deep as they are tall (with the help of a snorkel) - see [[:File:Zwei_Leopard_2A5_beim_durchqueren_eines_Gewässer.ogg]] where two modern [[Leopard 2]] MBTs ford a river, with only their snorkels and the ends of their gun barrels above water. They can only achieve this if they're heavier than the water they displace (that is, that they don't float). It seems other MBTs, including the T90 and the Abrams M1 can deep wade too. So that demonstrates that, fully immersed in water, they have enough net downforce to allow the treads to engage enough to drive the vehicle through the river at some speed (which shows they're not just on the ground, they're still quite firmly on the ground). There have been fully amphibious ("swimming", which means floating) tanks, like WW2's Swimming Sherman, but those either needed an inflatable float (see [[DD tank#Sherman DD]]) or to be specifically designed to float (like the lighter [[Expeditionary Fighting Vehicle]]). Now whether the driver of a wading tank fancies the idea of driving it around in what has become essentially a murky, obstacle-filled, fast-flowing river is another matter. The wading capability is designed either for hot landings on beaches from landing craft, or for fording rivers, so they're not designed to be terribly effective at doing anything but going in a straight line in water, and not for very long. -- [[User:Finlay McWalter|Finlay McWalter]] ☻ [[User talk:Finlay McWalter|Talk]] 22:23, 17 January 2011 (UTC)
:Our article states that there is no good clinical evidence that taurine supplements provide any benefit to human health, Why pay for useless supplements? The human body naturally produces <u>a large amount</u> of taurine, far more than one can reasonably take in as a supplement. There is increasing evidence that taurine actually plays a role in preventing cancer.<sup>[https://link.springer.com/chapter/10.1007/978-3-030-93337-1_11]</sup> Any carcinogenicity of supplements can only be due to their being fake, or a lack of quality control in their production. &nbsp;--[[User talk:Lambiam#top|Lambiam]] 07:23, 6 June 2024 (UTC)
::Maybe it's the difference between loaded weight and unloaded. Probably the number given in the article is for unloaded. Also a tank is not a cube, so the simplistic volume calculation is certainly too large. I was just going for an order of magnitude calculation. [[User:Ariel.|Ariel.]] ([[User talk:Ariel.|talk]]) 22:54, 17 January 2011 (UTC)
::The underlying issue is that large doses of taurine are added to energy drinks. Nobody seems to know why. A current study is looking at an association between energy drink consumption and the rise in colon cancer in young adults.[https://www.cancer.gov/research/participate/clinical-trials-search/v?id=NCI-2024-02154&r=1] [[User:Viriditas|Viriditas]] ([[User talk:Viriditas|talk]]) 08:23, 6 June 2024 (UTC)
:::There's not much difference, that I can figure, between the loaded and unloaded weight. Looking at the Leopard 2, I reckon its ammo (43 rounds at [http://www.fas.org/man/dod-101/sys/land/m830a1.htm 25 kg/round]) weighs 1075kg, the fuel (1160 litres of diesel at around 0.84 kg/litre) weighs 975kg, and if three of the four crew swim for it (say 3 x 100kg) that's 2.3 tonnes. That sounds like a lot (it's more than two cars) but when you consider the full Leopard 2 weighs in at a jaw-dropping 62 tonnes, the laden/unladen differential is only 4% or so. -- [[User:Finlay McWalter|Finlay McWalter]] ☻ [[User talk:Finlay McWalter|Talk]] 23:17, 17 January 2011 (UTC)
:::Even disregarding any potential carcinogenic risks, there are enough studies that show damaging health effects of high consumption levels of energy drinks.<sup>[https://www.ncbi.nlm.nih.gov/pmc/articles/PMC4682602/][https://www.kxan.com/news/energy-drinks-show-damaging-effects-on-young-adults-multiple-studies-show/]</sup> &nbsp;--[[User talk:Lambiam#top|Lambiam]] 18:28, 6 June 2024 (UTC)
: It seems that a "flash flood" can mean anything from an accumulation of rainwater at low points of streets where the water is essentially standing still, to a vertical wall of water roaring through a canyon downstream of a failed dam. A tank should have decent chances of operating in the former, but has essentially no possibility of surviving the latter.
: As soon as the flooding water is ''flowing'', lateral forces on an obstacle become at least as important as byoyancy. [[User:Henning Makholm|Henning Makholm]] ([[User talk:Henning Makholm|talk]]) 22:56, 17 January 2011 (UTC)
::::¼ liter of my favorite energy drink has 80 mg caffeine, while adults usually consume up to 400 mg caffeine per day. And I use the energy drink totally without sugar. [[User:tgeorgescu|tgeorgescu]] ([[User talk:tgeorgescu|talk]]) 18:59, 6 June 2024 (UTC)
:::::The energy drink thing may be related; most energy drinks contain [[vitamin B12]]. There is a concern that (mega)dosing B12 in excess of daily requirements carries with it a slight increase in risk of cancer due to B12 containing cobalt, which is both a heavy metal and has a trace of radioactive [[cobalt-60]]. <span style="font-family: Cambria;"> [[User:Abductive|<span style="color: teal;">'''Abductive'''</span>]] ([[User talk:Abductive|reasoning]])</span> 20:25, 6 June 2024 (UTC)
::For an extreme example, the [[Johnstown Flood]] picked up and moved entire locomotives; I am pretty sure such locomotives compare favorably in terms of density and weight to a tank. --[[User:Jayron32|<font style="color:#000099">Jayron</font>]]'''''[[User talk:Jayron32|<font style="color:#009900">32</font>]]''''' 00:11, 18 January 2011 (UTC)
::::::Thank you, that's helpful. I just read the comments by the epidemiologist on that topic, and while my reading might be flawed, they seemed to indicate that dosage and tobacco smoking played a significant role in the risk. [[User:Viriditas|Viriditas]] ([[User talk:Viriditas|talk]]) 22:02, 6 June 2024 (UTC)
:::And floods aren't just water moving, either. There's stuff being carried in that water. Boulders. --[[User:Jpgordon|jpgordon]]<sup><small>[[User talk:Jpgordon|::==( o )]]</small></sup> 07:21, 20 January 2011 (UTC)
:::::::Real skepticism cuts both ways. Taurine is in human breast milk, evidence of benefit of at least one energy drink exceeding risk. :-) But our article has imho excessive, even [[Georg Kreisel|dubious doubt]] in that section on its conditional essentiality or benefit for infants, and thus its common use in [[Infant formula|another energy drink]]. Doubt that appears to stem from OR or opinion rather than the source, which says e.g. "Thus the new data provide further support for the view that taurine is a conditionally essential nutrient for the preterm infant" & that ethical considerations seem to prevent further research.[[User:John Z|John Z]] ([[User talk:John Z|talk]]) 03:57, 7 June 2024 (UTC)


=January 18=


== Rocking Cooking Pot ==


What causes a (metal) cooking pot (no handle) to rock from side-to-side when on a stove burner (electric in my case) just as the water is about to boil? This motion can be halted temporarily by grasping the pot briefly. When released, the rocking resumes.--[[User:Koosharem|Koosharem]] ([[User talk:Koosharem|talk]]) 01:20, 18 January 2011 (UTC)
:Even before proper boiling, bubbles form in the water. These bubbles can exert forces on the walls of the pot. It could also be from [[Cavitation]], which is the net effect of lots of tiny shock waves created by tiny bubbles collapsing. --[[User:Jayron32|<font style="color:#000099">Jayron</font>]]'''''[[User talk:Jayron32|<font style="color:#009900">32</font>]]''''' 03:03, 18 January 2011 (UTC)
::Short, uncomplicated answer - The water is moving. [[User:Dodger67|Roger]] ([[User talk:Dodger67|talk]]) 07:43, 18 January 2011 (UTC)


== undetectable viral load risk ==


= June 9 =
how risky is it for those that have unprotected sex with someone that is hiv+ but has an undetectable viral load? I know that most people get HIV from other people that have just gotten it because those people have the highest amounts of replicas of the virus in their bloodstream. I know it is less likely that you would get hiv from someone on antiretrovirals or that is undetectable but is it nearly safe sex or not? Are there any statistics on this? Where would I find the answer to this question?[[User:Thisbites|Thisbites]] ([[User talk:Thisbites|talk]]) 01:29, 18 January 2011 (UTC)
:See the second to last question [http://www.avert.org/hiv-aids-transmission.htm here] and this [http://www.avert.org/media/pdfs/HIV-transmission-and-antiretroviral-therapy-Briefing-sheet.pdf briefing sheet]. [[User:Ariel.|Ariel.]] ([[User talk:Ariel.|talk]]) 01:45, 18 January 2011 (UTC)


== Accessing Previous Questions and Dates asked. ==


How do I access a previously asked question.eg Jan 8 2011? <span style="font-size: smaller;" class="autosigned">—Preceding [[Wikipedia:Signatures|unsigned]] comment added by [[Special:Contributions/118.208.117.68|118.208.117.68]] ([[User talk:118.208.117.68|talk]]) 02:01, 18 January 2011 (UTC)</span><!-- Template:UnsignedIP --> <!--Autosigned by SineBot-->
:You can search the reference desk archives with the "search" button at the top of this page; or you can browse them at [[Wikipedia:Reference desk/Archives]]. Here is [[Wikipedia:Reference_desk/Archives/Science/2011 January 8]]. [[User:Nimur|Nimur]] ([[User talk:Nimur|talk]]) 02:10, 18 January 2011 (UTC)


== Weigeltisaurus species clarification ==
== Light Sails. ==
Is this [[Weigeltisaurus]] reptile in this family [[Rhynchocephalia]]. Would that statement be true to say. Its for this article [[Johannes Weigelt]]. There is source that states it but I don't know how accurate it is. '''<span style="text-shadow:7px 7px 8px black; font-family:Papyrus">[[User:scope_creep|<span style="color:#3399ff">scope_creep</span>]]<sup>[[User talk:scope_creep#top|Talk]]</sup></span>''' 21:23, 9 June 2024 (UTC)


:While Weigeltisaurus flourished in the [[Permian|Late Permian]], our article on the order Rhynchocephalia states that the oldest record of the group is dated to the [[Middle Triassic]]. According to this chronology, a gap of several million years separates them. &nbsp;--[[User talk:Lambiam#top|Lambiam]] 07:21, 10 June 2024 (UTC)
Arthur C Clarke and others have postulated "Gossamer thin sails" for interstellar travel using light presure.Since light by definition is massless how can it exert pressure to 'fill' the sails?
::Right. I'll leave the Rhynchocephalia bit out and only mention the Weigeltisaurus bit, since I don't understand it. Thanks {{ping|Lambiam}} '''<span style="text-shadow:7px 7px 8px black; font-family:Papyrus">[[User:scope_creep|<span style="color:#3399ff">scope_creep</span>]]<sup>[[User talk:scope_creep#top|Talk]]</sup></span>''' 08:52, 10 June 2024 (UTC)
John Cowell[[Special:Contributions/118.208.117.68|118.208.117.68]] ([[User talk:118.208.117.68|talk]]) 02:15, 18 January 2011 (UTC)


= June 11 =
: See [[solar sail]], which operate by [[radiation pressure]]. -- [[User:Finlay McWalter|Finlay McWalter]] ☻ [[User talk:Finlay McWalter|Talk]] 02:21, 18 January 2011 (UTC)


== Can we agree on what the rule of the Council of Nicaea actually was? ==
::Photons (light) have [[momentum]] even though they have no mass. In modern physics, [[Momentum#Momentum_in_relativistic_mechanics|relativistic momentum]] is more complicated than just the simple "mass × velocity" relationship; in fact, momentum is related to the mass, velocity, and energy of the particle, according to a [[Lorentz transform]]. (So, even zero-mass photons can have non-zero momentum). This was formalized by [[Einstein]] as part of the theory of [[Special Relativity]]. [[User:Nimur|Nimur]] ([[User talk:Nimur|talk]]) 02:25, 18 January 2011 (UTC)
:::Photons have no [[rest mass]]. However, they clearly have [[energy]] and [[Mass–energy equivalence|energy is mass]], so anything which has energy can also exert a force. The [[photoelectric effect]], not-so-coincidentally also described by Einstein at the same time he explained [[special relativity]] is an atomic-scale example of a massless photon moving a massed particle (the electron). It works just as well on solar sails. --[[User:Jayron32|<font style="color:#000099">Jayron</font>]]'''''[[User talk:Jayron32|<font style="color:#009900">32</font>]]''''' 03:00, 18 January 2011 (UTC)


* ''Transferred from Humanities desk''
:Photons are not massless. They have no rest mass (or to put it another way, at rest they don't exist), but while in transit they do have mass. [[User:Ariel.|Ariel.]] ([[User talk:Ariel.|talk]]) 03:01, 18 January 2011 (UTC)
::There's no disagreement about the physics here - this is strictly a matter of semantics and terminology. I use the term "mass" to specifically refer to rest mass - as do many physicists. An equally-valid but different definition of the unqualified term "mass" refers to ''the sum'' of the rest mass plus the normalized kinetic energy, per mass-energy equivalence. Photons have an exactly-zero ''rest mass''. Many physicists (myself included) thus say "it has no mass." Many other physicists dislike this terminology, and always use "mass" to refer to relativistic mass. I don't think we need to devolve in to word-mungling pedantry over this, because we're all in agreement about the actual ''phenomena''. See [[Mass#Mass_and_energy_in_special_relativity|the ''mass'' article]], [[Mass_in_special_relativity#Terminology|the ''terminology'' section]], and [[Mass_in_special_relativity#References|the list of reference texts and papers]] for a bunch of different opinions about which definition is "better." [[User:Nimur|Nimur]] ([[User talk:Nimur|talk]]) 04:08, 18 January 2011 (UTC)
:::I wasn't arguing with you (not sure if you thought I was), it was a reply to the OP. I prefer to assign the word mass to relativistic mass because that's the one you need to use for calculations - momentum, gravity, inertia, etc. The trouble with doing so is that relativistic mass is not a constant (because it depends on who you are comparing it with), so I understand the arguments for assigning it to rest mass. [[User:Ariel.|Ariel.]] ([[User talk:Ariel.|talk]]) 04:22, 18 January 2011 (UTC)


Let me highlight Jack's comment above:
== Energy in a magnet. ==


{{xt|...What is or is not legal is a matter for courts or legislatures to determine. Not the media, not individual partisan politicians, not the man in the street, not the reasonable man, and not random commentators (on Wikipedia or anywhere else). -- 22:23, 6 June 2024}}
For more than 20 years I used a 3 inch magnet on a daily basis.It was so strong it would fly out of my hand if I got too close steel and the only way to release it from the deck of my truck was to slide it to the edge and carefuly remove.As a tool of trade for a scrap metal buyer it even exerted a 'pull' on 304 and 316 stainless steel.It was used for holding steel for welding,holding spring doors open etc.It would have lifted the equivalent of hundreds of tons over the years without diminishing in strength.
How can so much potential energy be incorporated in something so small,how is it stored and calculated,and surely there must be a multitude more practical uses for such a mini powerpack other than computers and maybe speakers?
John Cowell[[Special:Contributions/118.208.117.68|118.208.117.68]] ([[User talk:118.208.117.68|talk]]) 03:07, 18 January 2011 (UTC)
:The nutshell answer is that a [[magnet]] exerts [[force]], which is neither [[energy]] nor [[work]]. Work, rather, is force acting over a distance. When the magnet leaps from your hand to the steel, it's done some work, because it's moved. However, it's then stuck to the steel and does no further work. For it to move again, you've got to do work of your own (counteracting the work the magnet did) to get it back away from the steel. As for other practical uses (I'll leave aside whether they're ''more'' practical, as computers are way up there), magnets are essential to electric motors, generators, transformers, and medical devices. &mdash; [[User talk:Lomn|Lomn]] 05:14, 18 January 2011 (UTC)
::I was thinking about this the other day again because magnet questions are quite common on the ref desk. Maybe someone has used this analogy before but I thought a spring analogy might be good at illustrating the difference between force and energy. A loaded spring exerts a strong force, try holding a strong spring closed just with your arms for a long time! It can take a lot of energy on YOUR part. But a spring can only do an amount of work relative to its length and force it was loaded with to begin with. Once a spring is "unloaded" that's it, no more force. Same as a magnet. [[User:Vespine|Vespine]] ([[User talk:Vespine|talk]]) 05:48, 18 January 2011 (UTC)


So why is Wikipedia still saying [[Wedding of Prince Harry and Meghan Markle#Preceding private ceremony]] "The Church of England sources commented that this was not a [[Marriage in England and Wales|legally recognised marriage]] ceremony, which requires two witnesses", citing journalist and radio commentator Camilla Tominey, who screamed down the telephone '''I WILL NEVER WRITE THE STORY''' that Camilla and Charles' wedding ceremony was a "non-qualifying ceremony" (i.e. void), following up with a stream of invective which only ended when she was cut off? And as the words "Registrar General" are ''eiusdum generis'' with the list of occupations provided by Jack, why is the linked article still saying "the Registrar General, Len Cook, determined that a civil marriage would in fact be valid"? The articles are so full of editorialising that it is best not to read them and go to [http://www.dailymail.co.uk/news/article-338481/Charles-wedding-illegal-warn-experts.html] and [http://www.spectator.co.uk/article/fact-check-did-harry-and-meghan-have-a-secret-wedding] instead. Needless to say the "impediments to marriage" listed in the licence do not apply to the marriages of non-royals, because they were abolished by the Marriage Act which specifically states that its provisions have no applicability whatsoever to royal marriages. [[Special:Contributions/92.25.129.245|92.25.129.245]] ([[User talk:92.25.129.245|talk]]) 16:45, 8 June 2024 (UTC)
:: OP, This is by no means the first time a human has had this thought! :) Magnets are quite baffling until you understand how they work. It's the mystery of magnets that has led men on life-long fruitless and often fraudulent attempts at gaining "free" energy/money (Steorn, for example). Until a material is discovered that can block magnetic fields, no "work" can be done by them, just simple attraction. They are still fun to play with though[[User:Zzubnik|Zzubnik]] ([[User talk:Zzubnik|talk]]) <span style="font-size: smaller;" class="autosigned">—Preceding [[Wikipedia:Signatures|undated]] comment added 12:39, 18 January 2011 (UTC).</span><!--Template:Undated--> <!--Autosigned by SineBot-->
:::<small>[[Miracles (Insane Clown Posse song)|This work]] has some interesting things to say regarding the science behind magnets. --[[User:Jayron32|<font style="color:#000099">Jayron</font>]]'''''[[User talk:Jayron32|<font style="color:#009900">32</font>]]''''' 14:43, 18 January 2011 (UTC)</small>
::::<small>For those of you wondering, the song in the link Jayron gave is the source of the "Fucking magnets, how do they work?" Internet meme (which may not ''really'' be a meme, the talk page argues...). Also, I never realized people got hung up about the use of the word "miracles" in that song :rolleyes: [[User:TomorrowTime|TomorrowTime]] ([[User talk:TomorrowTime|talk]]) 15:16, 18 January 2011 (UTC)</small>
:::::Ha ha ha, that is silly. This is how they work [http://www.youtube.com/watch?v=wMFPe-DwULM YouTube link to discussion of how magnets work.].[[User:Zzubnik|Zzubnik]] ([[User talk:Zzubnik|talk]]) 17:01, 18 January 2011 (UTC)
:I'd like to point out the enormous work I do every day by exerting a force of mumble Newton with by behind to the seat of my chair - not to mention the nightly work quota I exert on my bed. ;-) --[[User:Stephan Schulz|Stephan Schulz]] ([[User talk:Stephan Schulz|talk]]) 17:32, 18 January 2011 (UTC)
::The [[Tesla]] is not actually a unit of energy, so you can't say how much energy it takes to make a magnet of a certain strength. And yet, it ''would'' be interesting to know how much actual energy it takes to turn a non-magnetic chunk of metal or rare earth into a powerful magnet like the one described; and conversely whether there is any theoretical way by which the entropy of the magnet as it degrades from perfect order could be used to drive a thermodynamic process that produces work. Hmmm - is this the same as a measurement of any excess thermal energy released if the magnet is pulverized under a powerful ram? I would think that the amount of energy needed to create two magnets of opposite polarity ''must'' exceed the amount of energy they release by coming forcefully together - I suspect that this is by some large factor - I wonder what physical factor describes this necessary limitation on the ease with which permanent magnetism can be induced in a material. [[User:Wnt|Wnt]] ([[User talk:Wnt|talk]]) 06:10, 19 January 2011 (UTC)
:::I suspect that the 'crushing' process would actually be endothermic, not exothermic. Ordered, aligned spins in a ferromagnetic material have a lower energy than randomly oriented spins; you have to put in energy to break the alignment. At the [[Curie temperature]], a phase transition occurs when you move from aligned to random spins; this shows up as a measurable endothermic bump by [[differential thermal analysis]]. Similarly, if you just pull aligned-spin particles or domains apart without jumbling their orientations, that costs you energy as well &mdash; the energy of aligned spins is lower than the energy of the free or separated particles. [[User:TenOfAllTrades|TenOfAllTrades]]([[User_talk:TenOfAllTrades|talk]]) 14:30, 19 January 2011 (UTC)
::::Well, I have to admit, you're right about that. My intuitive feeling was that a pile of powdered magnet shouldn't spontaneously assemble into a powerful magnet ball - I wonder if that was also false. [[User:Wnt|Wnt]] ([[User talk:Wnt|talk]]) 18:07, 19 January 2011 (UTC)


:Readers may be interested in a dream I just had. Following the inception of the discussion ([[Wikipedia:Reference desk/Humanities#June 7]]) about al-Biruni and the [[Special:Permalink/1188536894#The Reichenau Primer (opposite Pangur Bán)|Byzantine calendar]], before I went to bed last night (30 ''James'', the day before the date marked with this year's "golden number" (11), 1 ''Eloise''), I looked for the new moon and saw a very slender crescent low in the north-west. In the dream, I was at work and was told that Camilla Tominey and another presenter had arrived to interview me. The male presenter remained outside and Camilla came in. She was aggressive and incoherent, and after a while an aide entered and bundled her out. I heard him say as he took her away "You're not on the programme any more. We're changing the presenter." Then the other presenter came in and said "You're going to be on the programme - ATV." As we left, he commented "You're casually dressed." I said "Shall I go home and change into something more formal?" and he said "We'll dress you at the studio." Then he asked an assistant "She's carrying a handbag, take it from her." I thought "This is for the general election." Outside there were a number of vehicles. Beside one of them a woman staff member was restraining a short man with his hands behind his back. I woke up at this point and went downstairs. It was 1:30 AM. I switched on the radio and this song began playing: [http://www.youtube.com/watch?v=BqGTb4ZFAS8]. [[Special:Contributions/92.12.79.187|92.12.79.187]] ([[User talk:92.12.79.187|talk]]) 15:42, 9 June 2024 (UTC)
== Will an object past the event horizon of a black hole inevitably reach the singularity? ==
::As a matter of historical interest, one grouse the Muslims had about the imposition by the Caliphs of the new, unintercalated Islamic year for administrative purposes to replace the 365-day Zoroastrian year previously used in Persia was that it was shorter, which meant that the land tax would come round before the harvest had been collected! The Caliphs relented and reinstated the ''araji'' (land-tax) year for fiscal purposes.
::But then another problem presented itself. The Muslim era began in AD 622 with the ''Hegira'' (flight) - the Persians later fixed the beginning of this era on Friday, 19 March. The Zoroastrian year needed periodic readjustment because it was itself on average six hours shorter than the Julian. Al-Biruni notes that there was a double adjustment of the start of the ''araji'' year during the reign of Yazdegerd I (AD 399-420). An ''araji'' era was introduced dating from AD 621. In a further recalibration, the Yazdegerdi era, still used by the Zoroastrians (Parsees), dates from the accession of Yazdegerd III on 16 June AD 632, so the Yazdegerdi era is eleven years behind the ''araji''. In AD 895 there was another double readjustment of the start of the ''araji'' year. It moved from 1 ''Frawardin'' (12 April) to 1 ''Khordad'' (11 June), referred to as 11 ''Haziran'' (I think the Turks call June by this name to this day).
::Today's Byzantine date (Monday, 2 ''Eloise''), marks the imminent arrival of the Jewish Feast of Weeks (Pentecost), observed on the 6th of the corresponding Jewish month. It is barred from falling on Tuesday, Thursday or Saturday, and slots in this year this coming Wednesday (4 ''Eloise''). Orthodox Whitsun is not till Sunday, 23 June (15 ''Eloise''), because of the rule that Orthodox Easter falls on the Sunday after the Wednesday after the date of the Paschal Full Moon (14 ''Miri'') between 1800 and 2099 inclusive, a sharp contrast with the Roman Catholic Church (but not it's Byzantine arm), which likes to time its Easter festivities to fall in the week after the "''[[Purim]]'' Full Moon". The Orthodox consider the Catholics to be in error and they are right - the Council's direction was that the Easter full moon must not precede the equinox, but in 1582 Gregory XIII directed that it must be the first after the equinox. This led to his calendar being banned in Orthodoxy. [[Special:Contributions/2.30.124.132|2.30.124.132]] ([[User talk:2.30.124.132|talk]]) 15:43, 10 June 2024 (UTC)
:::Some common Serbian names feature in the story. This article <ref>{{cite web|url=http://www.religionunplugged.com/news/2024/5/3/easter-calendar-question-continues-to-split-the-orthodox-church|last=Trpkovic|first=Jovan|title=Orthodox Easter: calendar question continues to split the Church|date=3 May 2024|accessdate=11 June 2024}}</ref> includes the following points:


* In addition to the issue of calendar reform, the [[Council of Constantinople (1923)]] also discussed possible union with the Anglican church and second marriage for priests
An object around a gravitational body like the Earth can orbit indefinitely if its velocity is sufficiently high, but is it possible for an object to remain stationary within a black hole? Likewise, can an observer past the event horizon perceive other objects? By definition, light can not escape the horizon, so would it be impossible for photons to reach the observer from further within the black hole? <span style="font-size: smaller;" class="autosigned">—Preceding [[Wikipedia:Signatures|unsigned]] comment added by [[Special:Contributions/68.40.57.1|68.40.57.1]] ([[User talk:68.40.57.1|talk]]) 03:27, 18 January 2011 (UTC)</span><!-- Template:UnsignedIP --> <!--Autosigned by SineBot-->


* Vladimir Dimitrijevic, a conservative author, said the Council of Constantinople was among the greatest mistakes of the Orthodox Church in the 20th century
:Yes, in case of Schwarzschild black hole, object that crosses event horizon reaches the singularity sooner or later. All the paths inside of event horizon lead to singularity. So harder it tries to avoid, sooner it reaches singularity. For other types of black holes (charged or rotating black holes), there is possibility of avoiding singularity. I am doubtful whether an object can remain stationary within a black hole. About your last question, are you asking whether photons from within black hole can reach observer who is outside event horizon? No, photons cannot escape from black hole. Or, are you asking whether observer within the black hole can observe photons? Yes, observer falling inside black hole continues to observer things according to his clock, he cannot determine whether he has reached event horizon or has crossed it. - [[User:Ranemanoj|manya]] ([[User talk:Ranemanoj|talk]]) 04:21, 18 January 2011 (UTC)


The author is Jovan Trpkovic. An article by M S Dimitrijevic and others<ref>{{cite journal|url=http://www.adsabs.harvard.edu/abs/2008JAHH...11...50D|last1=Dimitrijevic|first1=M S|last2=Theodossiou|first2=E Th|last3=Mantarakis|first3=P Z|title=Milutin Milankovic and the reform of the Julian calendar in 1923|journal=Journal of Astronomical History and Heritage|volume=11|issue=1|year=2008|pages=50-54}}</ref> includes these points:
::To clarify, I was asking whether the observer ''within'' the event horizon can detect photons from further inside the black hole. Also, your statement regarding the observer not being able to determine whether he has reached the [[event horizon]] is in contradiction of the article on it. --[[Special:Contributions/68.40.57.1|68.40.57.1]] ([[User talk:68.40.57.1|talk]]) 04:34, 18 January 2011 (UTC)


* The Serbian delegation came to the Congress with a proposition for calendar reform authored by Maksim Trpovic. He proposed the intercalation rule that the secular years in centuries which when divided by 9 have remainders of 0 or 4 will be leap years
:::No, it is exactlyt confirmed by the [[event horizon]] article. Let me quote it for you. "An observer crossing a black hole event horizon can calculate the moment they've crossed it, but will not actually see or feel anything special happen at that moment." --[[User:Jayron32|<font style="color:#000099">Jayron</font>]]'''''[[User talk:Jayron32|<font style="color:#009900">32</font>]]''''' 04:40, 18 January 2011 (UTC)


* The general opinion of the participants was that the better solution was to retain the Julian calendar and only delete thirteen days...
:::I presume you meant to say "within the event horizon", not "within the singularity". Yes, for a while an observer inside the event horizon will be able to see an object that's further inside the black hole, that fell in shortly before the observer. However, the object will rapidly move further away from the observer, due to the extreme [[tidal force]]s, and the object's image will become [[red shift]]ed, to the point where the observer soon won't be able to see it at all. In other words, the appearance of the object will qualitatively appear very similar to the appearance of an infalling object as seen from an observer that's outside the event horizon. [[User:Red Act|Red Act]] ([[User talk:Red Act|talk]]) 06:16, 18 January 2011 (UTC)


The information was duly sourced and added to [[Julian calendar]]. This led to one editor demanding that the contributor be banned from Wikipedia for "inserting false information into articles" because "if thirteen days are excised from the Julian calendar it is no longer Julian." He further demanded that articles be pre-emptively semi-protected to prevent editors removing any unsourced falsehoods added by others. His campaign was successful and false information, for example that the Greek government introduced the Gregorian calendar in 1923, is embedded in articles all over Wikipedia. The original Serbian proposal was in fact a proposal that the calendar already legislated by the Greek government should now be adopted by the Orthodox Church. As to where the Greek government got ''its'' calendar from, it's described here<ref>{{cite book|url=http://books.google.co.uk/books?id=bPcNAAAAQAAJ|last=Oriani|first=Barnaba|title= De usu fractionum continuarum ad inveniendos ciclos calendarii novi et veteris. In: Appendix ad ephemerides anni 1786|date=1785|location=Milan|pages=132-154}}</ref> although Trpkovic (who proposed it in 1900) denied all knowledge of Barnaba Oriani's calendar. Another example is the claim that "The reform of the calendar was authorised by a canon of the Council of Trent in 15.." (the last two digits of the date of the alleged canon are something of a movable feast, since they change periodically). [[Special:Contributions/92.19.71.221|92.19.71.221]] ([[User talk:92.19.71.221|talk]]) 15:14, 11 June 2024 (UTC)
:::[[File:KruskalKoords.gif|right|180px]]You have to be specific about what you mean by "further inside", because black hole spacetimes are nothing like the ordinary flat space where concepts like "inside" are normally defined and make sense. The most obvious thing you might mean by "further inside" is that the Schwarzschild r coordinate is smaller. Outside the event horizon, that coordinate measures distance from the center of the hole, more or less. Inside the event horizon, though, it measures time. The singularity is in the future, not in a particular place, and "further inside", in this sense, would really be "later in time". You can't see the future, even inside a black hole. You can, however, see objects that crossed the event horizon before you did. The easiest way to understand how this works is to look at a diagram like the one to the right. Ignore everything but the upper right quadrant. The straight diagonal line is the event horizon; below and to the right of that is the outside world; above and to the left is the black hole interior. The singularity is the boundary of the grey region. The grey region itself is nothing (it's not part of the solution). The past is down, the future is up. Light travels along 45° diagonal lines. You can only cross the event horizon from right to left, since you're limited by the speed of light. If you draw two sub-light worldlines crossing the horizon, it's easy to see that light from the first one will reach the second, so you can see someone who crossed the horizon before you did. You will hit the singularity before you see them hit the singularity, though. The animated blue lines are lines of constant Schwarzschild r. -- [[User:BenRG|BenRG]] ([[User talk:BenRG|talk]]) 07:25, 18 January 2011 (UTC)


{{reflist-talk}}
== Ticklish sensation near forehead ==

If an object is brought close to my forehead, but does not touch it, my forehead becomes sensitive and starts tingling and becoming very "ticklish", for lack of a better term. For several years, I've asked certain people if they've had the same sensation, and I've only met one girl who did. I have unsuccessfully tried testing the sensation with my eyes closed. As the object (any object) is brought closer to the forehead, the sensation deepens. It used to be really bad when I was a child, but now as an adult, I hardly even think about it anymore. It just popped into my head a minute ago and I decided to ask about it :)[[User:Reflectionsinglass|Reflectionsinglass]] ([[User talk:Reflectionsinglass|talk]]) 03:56, 18 January 2011 (UTC)

:You are not alone.[http://boards.straightdope.com/sdmb/archive/index.php/t-370228.html] -- I'm particularly interested in this comment from DocCathode:
:"Nothing supernatural about it. It's a mixture of a sense of danger/discomfort, the urge to move away, and the repression of that urge.
:"To prove this, you need a friend who trusts you and follows instructions. Tell them to stay still, and to open and close their eyes when you tell them. Begin approaching with finger about two feet away. Tell them to close their eyes. Move the finger foot closer and tell them to open their eyes. Tell them to close their eyes. Move the finger to six inches away. Tell them to open their eyes. Tell them to close their eyes. Move the finger to three inches. Tell them to open their eyes. Tell them to close their eyes. Move the finger to two inches. Tell them to open their eyes. Tell them to close their eyes. Move the finger to an inch. Tell them to open their eyes. Tell them to close their eyes. Move the the finger to half an inch from their forhead. Tell them to open their eyes. Tell them to close their eyes. Slowly and quietly move your arm away from them. Tell them to open their eyes.
:"The tingling sensation is most intense the last time, despite the fact the no object is near their forehead."
:Please try that and let us know what happens. [[Special:Contributions/71.198.176.22|71.198.176.22]] ([[User talk:71.198.176.22|talk]]) 05:57, 18 January 2011 (UTC)

== what killed Dinosaurs==
I read an article that had say something about the death of mamots in 35000years ago that may be general for last happening for dinosoros 0 the radiation of any supper nova can kill dinosoros
in some million years ago. A.mohammadzade jan 18 iran <span style="font-size: smaller;" class="autosigned">—Preceding [[Wikipedia:Signatures|unsigned]] comment added by [[Special:Contributions/78.38.28.3|78.38.28.3]] ([[User talk:78.38.28.3|talk]]) 04:23, 18 January 2011 (UTC)</span><!-- Template:UnsignedIP --> <!--Autosigned by SineBot-->
:I'm not sure I understand your question properly. Dinosaurs became extinct during the [[Cretaceous–Tertiary extinction event]]. There are various theories about what caused this to happen. The most popular theories have to do with an asteroid (or asteroids) hitting the earth. You may also find the [[:simple:K/T extinction event|simple wikipedia article]] or the [[:fa:انقراض دوران سوم کرتاسه|farsi wikipedia article]] on this topic to be helpful. [[User:Calliopejen1|Calliopejen1]] ([[User talk:Calliopejen1|talk]]) 04:55, 18 January 2011 (UTC)
::3500 years ago? Are you thinking about [[Noah's_Ark|Noah's flood]] (Nuh in Islam)? According to the (Christian) [[Ussher chronology]] this occurred 2348 [[before common era|BCE]]. Modern science places the dinosaurs extinction to the Cretaceous–Tertiary event 65.5 million years ago <small>(as Calliopejen1 noted)</small>, and [[woolly mammoths]] to about 10 thousand years ago. BTW, several web-browsers have spell-checkers for their text-entry boxes; you should consider installing one for all the languages you speak. [[User:Csmiller|CS Miller]] ([[User talk:Csmiller|talk]]) 05:36, 18 January 2011 (UTC)
::We have an article on [[near-Earth supernova]]e, which discusses their effects. A supernova is believed to have caused the [[Ordovician-Silurian extinction events]], which was one of the first major extinctions, around 450 million years ago. [[User:Csmiller|CS Miller]] ([[User talk:Csmiller|talk]]) 05:45, 18 January 2011 (UTC)
:::Actually, the OP said 35,000 years ago, not 3,500 - no need to drag young earth creationism into this. As you note, mammoths were still alive and well at that point, so it's a curious date to bring up. Our article on [[35,000 BC]] <small>No, we don't really have an article on that date, but it is a nice redirect... </small> doesn't mention anything particularly germane. 15:08, 18 January 2011 (UTC)
::::Ooops. My apologies. I can only think the lack of punctuation made me not see one of the zeros, and conflate the OP's timescale with that of [[young earth creationism|YECs]]. [[User:Csmiller|CS Miller]] ([[User talk:Csmiller|talk]]) 17:05, 18 January 2011 (UTC)
::There is really no question that the asteroid killed the dinos. Headline writers and popular science in general will never tire of stirring up controversies, but a panel of 41 international experts have met and agreed that there is consensus on the cause. This is mentioned in the articles. [[User:Imagine Reason|Imagine Reason]] ([[User talk:Imagine Reason|talk]]) 00:00, 19 January 2011 (UTC)
:::<small>Actually, I killed the dinosaurs. It was a time travel mistake, sorry, but when you gotta sneeze, you gotta sneeze. I take no responsibility for the mammoths, though. I think Baseball Bugs did them in. --[[User_talk:Ludwigs2|<span style="color:darkblue;font-weight:bold">Ludwigs</span><span style="color:green;font-weight:bold">2</span>]] 00:04, 19 January 2011 (UTC)</small>
:::<small>I thought [http://www.youtube.com/watch?v=Aj-nfYMcJpk Tezuka] killed the dinosaurs. — [[User:DanielLC|DanielLC]] 06:02, 19 January 2011 (UTC)</small>
:::<small>Really? Could've sworn it was [[Adric]]. [[User:Crimsonraptor|Crimsonraptor]] &#124; [[User talk:Crimsonraptor|(Contact me)]] <small>[[Special:Contributions/Crimsonraptor|Dumpster dive if you must]]</small> 14:23, 19 January 2011 (UTC)</small>

== Sprouts with quickly growing roots ==

What is the fastest growing, soil erosion preventing, preferably edible plant suitable for San Francisco's climate? [[Special:Contributions/71.198.176.22|71.198.176.22]] ([[User talk:71.198.176.22|talk]]) 05:47, 18 January 2011 (UTC)

:[[Lespedeza]] are commonly used in the Southeastern U.S. and meet all of your requirements except edibility. Not sure if it will grow in the Bay area, however. --[[User:Jayron32|<font style="color:#000099">Jayron</font>]]'''''[[User talk:Jayron32|<font style="color:#009900">32</font>]]''''' 05:53, 18 January 2011 (UTC)

==never say death star for pulsars==
the crab nebula is sending several rays and is alive there in space . the heart of the star is sending palses so we ought never say the super nova has died . a. mohammad zade iran --[[Special:Contributions/78.38.28.3|78.38.28.3]] ([[User talk:78.38.28.3|talk]]) 06:06, 18 January 2011 (UTC)
:There is no question that the [[crab nebula]] article uses the term "dead" only when citing [http://chandra.harvard.edu/photo/2006/crab/]. [[Special:Contributions/71.198.176.22|71.198.176.22]] ([[User talk:71.198.176.22|talk]]) 06:03, 18 January 2011 (UTC)

== when the earth was shining ==
i think that the earth was shining for some million years . if you want to know why then that will explain with my new theory a. mohammad zade
:Why don't you ask whether your theory is consistent with the evidence so we can critique it for you? To do so, you would have to explain it. It is true that the elements heavier than iron came from supernovae in the Earth's distant past. [[Special:Contributions/71.198.176.22|71.198.176.22]] ([[User talk:71.198.176.22|talk]]) 06:06, 18 January 2011 (UTC)
::thanks for giving commnets .i will soon write my theory hear i have some problems in safe evaluation andpublishing . so i save it such as patent . it will wrote here as soon as i published .--[[Special:Contributions/78.38.28.3|78.38.28.3]] ([[User talk:78.38.28.3|talk]]) 06:13, 18 January 2011 (UTC)
:::The Earth is still shining, see [[earthshine]]. [[User:Pfly|Pfly]] ([[User talk:Pfly|talk]]) 10:54, 18 January 2011 (UTC)

:Wikipedia is not the right place for people to develop their new theories. See our [[WP:Original Research]] policy. [[User:Comet Tuttle|Comet Tuttle]] ([[User talk:Comet Tuttle|talk]]) 18:38, 18 January 2011 (UTC)

== Pre-existing conditions and Medical Insurance ==

I really don't understand why people who have a pre-existing conditions are making SUCH a big deal out of having to pay more for insurance. I understand they get mad if they are denied, but they should have to pay more. It's the same way I have to pay more for auto insurance because I am younger and have had several speeding tickets in the past few years, I am a greater risk. Many (not all, but many) CHOOSE to have these pre-existing conditions. Many people who are obese, choose to have several other related problems, many people who high cholesterol choose to have it, same with so many other conditions. Is there a flaw in my thinking? I'm already paying over $200 a month for insurance when I don't ever use it because of all these abusers, when I am perfectly healthy, take care of myself, follow a healthy diet, and go to the gym 5 days a week. If everyone was covered, it would just make the problem worse and more expensive for everyone. <span style="font-size: smaller;" class="autosigned">—Preceding [[Wikipedia:Signatures|unsigned]] comment added by [[Special:Contributions/76.169.33.234|76.169.33.234]] ([[User talk:76.169.33.234|talk]]) 07:44, 18 January 2011 (UTC)</span><!-- Template:UnsignedIP --> <!--Autosigned by SineBot-->

: What country are you from? People there might be grateful they don't live here in Australia. Here, if you have a pre-existing condition, you can be denied ''any'' benefits for treatment of that condition for 12 months after taking out the insurance; but after that, you're fully covered like everybody else. There's no alteration to premiums, though; that's strictly outlawed. -- [[User:JackofOz|<font face="Papyrus">Jack of Oz</font>]] [[User talk:JackofOz#top|<font face="Papyrus">[your turn]</font>]] 08:00, 18 January 2011 (UTC)

::USA. I don't know how people think in Australia or in other countries, but I work in a pharmacy and my customers think just because they are taking a cholesterol medication, they can eat whatever they want. My customer was joking about it the other day. <span style="font-size: smaller;" class="autosigned">—Preceding [[Wikipedia:Signatures|unsigned]] comment added by [[Special:Contributions/76.169.33.234|76.169.33.234]] ([[User talk:76.169.33.234|talk]]) 08:04, 18 January 2011 (UTC)</span><!-- Template:UnsignedIP --> <!--Autosigned by SineBot-->

:::Yes, insurance companies have to discriminate, but it's sometimes hard to be tough on those with self-inflicted conditions while being fair to those whose ailments are innocently gained. [[User:HiLo48|HiLo48]] ([[User talk:HiLo48|talk]]) 08:09, 18 January 2011 (UTC)
::::No, insurance companies '''''don't''''' have to discriminate, but only if you run a system of compulsary insurance. This is what happens, in one form or another, across most of the developed world: compulsary insurance with a ban on discrimination. [[User:Physchim62|Physchim62]] [[User talk:Physchim62|(talk)]] 12:39, 18 January 2011 (UTC)
:I'm actually kind of struck by your first sentence. My fiancee, in trying to get private coverage, has been rejected by ''every insurer'' in California due to a pre-existing condition, which we're told will continue to be the case for another couple years. This is not a matter of paying more - she's not even allowed in the existing high-risk pools. I hate to think where she'd be if neither she nor I had a job. HIPAA and COBRA (expensive) do ignore pre-existing conditions, but they are not available to everyone. As for whether people choose to have a pre-existing condition, I have no comment. [[User:Someguy1221|Someguy1221]] ([[User talk:Someguy1221|talk]]) 08:15, 18 January 2011 (UTC)

::[[HIPAA]] is a health information privacy act, not an insurance company. [[Consolidated Omnibus Budget Reconciliation Act of 1985|COBRA]] is Federally mandated insurance coverage for those who recently left employment. I do not see how those apply in the context you are using. -- [[User:Kainaw|<font color='#ff0000'>k</font><font color='#cc0033'>a</font><font color='#990066'>i</font><font color='#660099'>n</font><font color='#3300cc'>a</font><font color='#0000ff'>w</font>]][[User talk:Kainaw|&trade;]] 17:28, 18 January 2011 (UTC)

:::Further, California has a state managed program specifically for high-risk patients who have been denied insurance. It is explained [http://www.mrmib.ca.gov/ here]. -- [[User:Kainaw|<font color='#ff0000'>k</font><font color='#cc0033'>a</font><font color='#990066'>i</font><font color='#660099'>n</font><font color='#3300cc'>a</font><font color='#0000ff'>w</font>]][[User talk:Kainaw|&trade;]] 17:31, 18 January 2011 (UTC)

::::I wasn't even aware of that plan, thanks (but thankfully she has a job, and I don't even want to know how much that costs). I was referring to [http://www.hmohelp.ca.gov/dmhc_consumer/hp/hp_hipaacp.aspx HIPAA health plans], by the way. I was using both that and COBRA as examples of coverage potentially available to individuals with pre-existing conditions who do not get such coverage through their current employer, or who are unemployed. [[User:Someguy1221|Someguy1221]] ([[User talk:Someguy1221|talk]]) 00:04, 19 January 2011 (UTC)

Really it is a question of what costs we think are fair to spread across society, and which we don't. If you happen to be develop a terribly expensive health condition, through no fault of your own, should society bear this cost, or should you? Thinking from a position behind the [[veil of ignorance]], one might prefer to spread such costs across society. Also, you note that some health conditions are in part caused by deliberate choices. This may be true, but most health conditions are not, and what sort of surveillance regime would be necessary to monitor which people this applies to? Such surveillance would likely be prohibitively expensive, as well as intrusive on privacy. Thinking forward into the future, as genetic health data becomes more readily available, it is plausible that insurance companies could very easily calculate your health risks to a much greater precision. (Assuming this were legal.) Is this something you would want? It is essentially just an extension of your principle of paying a premium that reflects your health risks. It would assign costs much more precisely to the individual, but it would change the nature of health insurance as we know it. Now, health insurance effectively insures against the risk that you will get Alzheimer's, for example. In the future, perhaps it would already be known that you will get Alzheimer's (even now, there are some genetic markers identified that are highly correlated with it). Then, since your premiums would already reflect your future disease, you would bear the cost for your Alzheimer's treatment, and your insurance would only be effective against risks such as being hit by a car and needing emergency surgery. Would this be preferable? To me, probably not. YMMV. [[User:Calliopejen1|Calliopejen1]] ([[User talk:Calliopejen1|talk]]) 11:11, 18 January 2011 (UTC)

What does this have to do with science? [[User:Ndteegarden|thx1138]] ([[User talk:Ndteegarden|talk]]) 13:01, 18 January 2011 (UTC)
::For the areas of science related to the question, see: [[Actuarial science]]. Risk assessment and probability are aspects of this thread. Also see [[Social sciences]]. The thread relates to economics, sociology, and psychology. See [[Medicine]]. The question is related to [[Health care]]. See [[Disease#Social significance of disease]]. [[User:Edison|Edison]] ([[User talk:Edison|talk]]) 16:41, 18 January 2011 (UTC)
:Generally this is misplaced, although the degree to which disease is a "choice" could be more scientific. After all, obesity and even high cholesterol are controlled by genetic and other propensities (I suspect that there should be a dramatic role for epigenetics here...) I feel that nothing but prejudice controls what people like the OP pick out as a "choice" - after all, the commuter who suffers injuries in a traffic accident also made a choice, right? The skier who leaves his testes with a buried pole on the slopes, likewise. The HIV victim, few would disagree, but what about the victim of flu or rotavirus or meningitis or malaria, who could have avoided it all just by staying inside his front door?
:It may be more for the humanities desk to answer what the point is of having insurance if suffering a diagnosis means being denied insurance. Why not just keep your money and pay as you go, with no private bureaucrats pretending to serve you by driving up costs? The problem is, no one wants to be diagnosed with anything, and as they are servants only of corporations and government the doctors are trusted far less than Mafia bookies, so preventative care becomes a pretty myth. [[User:Wnt|Wnt]] ([[User talk:Wnt|talk]]) 15:30, 18 January 2011 (UTC)
:"Why should society pay for the consequences of someone's bad choices?" begs the question of bad things happening to people who have in no way brought it on themselves. It is a reasonable question whether a society should spread the medical costs across all members, or have a "Every man for himself and the devil take the hindmost" lifeboat ethic. A person can be bopping along, healthy as a horse at 20 and at 30, and at 40, never took a sick day, eating right, not smoking, exercising regularly, then suddenly they have some crippling injury or ailment, like a debilitating and persistent and ineradicable infection, or a crippling injury, or cancer, or heart disease, or insulin dependent diabetes, with little hope of getting individual health insurance from a for-profit insurer. Many Americans live in a fantasy world where they will always have a policy from work. If the hospitalization means you can't work, and you don't have or lose your employer-provided insurance, you are just out of luck when you need surgery or hospitalization which might cost $100,000. Figure on losing the home and spending every dollar of retirement savings. "Move in with the folks" is not an option when they are dead and gone. Even if someone with severely impaired health found a private insurer, it would cost more per month than they could pay, especially if they are not healthy and can only work part time or a lesser pay level than their previous fine career allowed. "Paying more for insurance because of a preexisting condition" may mean paying an extra $12,000 a year more than the basic insurance, or even more than that, which the unwell person simply cannot afford. According to [http://www.emaxhealth.com/1275/72/34414/sebelius-releases-report-health-insurance-reform-and-diabetes.html a report], "80 percent of people with diabetes were uninsured after they lost health insurance coverage due to loss of a job or job change, divorce, change in income or health status, or a move. " It is not presently affordable for them. [http://articles.baltimoresun.com/2010-09-22/news/bs-ed-health-reform-20100922_1_health-care-health-reform-bill-health-spending Over 50 million Americans] have no health insurance, so that if they do go to see a doctor or have tests, they must pay "list price," typically several times what the cost is for those with insurance, since Blue Cross et al reduce the price to a fraction of what the doctor, hospital or lab says it should cost. For instance, Xrays that the insurer writes down to $100, and which cost me $15 out of pocket, would have cost the uninsured $350. An EKG that would have cost the uninsured $1100 got written down by the insurer to under $300 (and only cost me under $30). [[User:Edison|Edison]] ([[User talk:Edison|talk]]) 16:59, 18 January 2011 (UTC)
:::Fom a european persepective the "devil take the hindermost" American health care system, or lack of it, is scandalous. Add that to guns being used freely, with gun-murders being 100 times more common per capita. They are not good adverts for the US. [[Special:Contributions/92.24.183.183|92.24.183.183]] ([[User talk:92.24.183.183|talk]]) 14:42, 19 January 2011 (UTC)
::While I agree with all that you said (in particular, many medical conditions are not or not fully the result of irresponsible choices), there is also another aspect: It might be ''cheaper'' to pay for the effect of someones bad choices. Sick people typically can contribute less to society - imagine [[Stephen Hawking]] without universal health care. Similarly, desperate people will do desperate deeds. Would you rather pay for someones cancer treatment via a social insurance system, or pay for more police to keep desperate husbands, wives, or parents from robbing banks to pay for treatment? --[[User:Stephan Schulz|Stephan Schulz]] ([[User talk:Stephan Schulz|talk]]) 17:24, 18 January 2011 (UTC)
:::This is a valid point, but it really doesn't go far enough. Society has prevented disaster by various back-door ways of paying for health care for the poor, such as hospitals providing care and then just not getting paid for it, then working the costs into care for other patients. But hospitals have become more and more commercialized, with less profitable sites even shutting down, and if political efforts to correct the situation are rebuffed, then you're left with a situation where a segment of the population denied access to health care would begin to turn on it as an enemy. The effect of even a single ambulance bomb would be devastating (and how do you apply security precautions?). The effect of three or four simultaneous attacks against all remaining hospitals of a metropolitan area would essentially put major health care out of commission for a hundred miles in any direction - even if such a homegrown al Qaida ''didn't'' manage to get their hands on the radiation sources inside the hospital ... [[User:Wnt|Wnt]] ([[User talk:Wnt|talk]]) 05:12, 19 January 2011 (UTC)

:I have a concise answer: The argument (that people with pre-existing conditions should just suck it up and pay a lot more for their medical insurance) makes complete sense from an [[actuarial table|actuarial]] point of view, but it collides with many people's belief that a society should take care of its sick, even though this makes insurance more costly for the more-healthy. [[User:Comet Tuttle|Comet Tuttle]] ([[User talk:Comet Tuttle|talk]]) 18:37, 18 January 2011 (UTC)
:: Yes. Once we have decided we will not let people simply die on the street, then we must assess the cost of emergency treatment and conclude that an ounce (or penny) of prevention is worth a pound (dollar) of cure, therefore the next step is to try to fund preventative care. Because whether through taxes or insurance premiums, you still have to pay for the sick, so might as well pay less for better results. [[User:SamuelRiv|SamuelRiv]] ([[User talk:SamuelRiv|talk]]) 22:10, 18 January 2011 (UTC)
:::<small>This isn't really a science issue and is OT on the RD anyway but that probably applies to most of this discussion. The above is one of the arguments for measures to tackle obesity, smoking and other problems as well as encouraging healthy eating, exercise etc in countries with some sort of socialised medical care. Of course depending on the measures this can lead to claims of nanny state, food police, sin tax and whatever else and measuring the efficacy of such measures is generally difficult at best. And I've seen it argued smokers at least actually cost the system less in the long run, particularly where there are other costs (e.g. in NZ where a government funded pension is something most citizens past a certain age are entitled) due to the shorter lifespan even with their added medical bills. (Of course you then get in to complicated issues like whether they contribute less.) Unrelated to this but related to the earlier point you also get the complex issue of how much should be spent on treating people with a given expected outcome. For example, is it worth spending $100k on treatment that may increase the chance of living past 5 years from 5% to 10%? (Very simplistic of course, the outcome of many treatments come in things like improved quality of life which are generally subjective.) Ironically this was seen to some extent in the US with the recent debates when the [[death camp]]s nonsense came up. [[User:Nil Einne|Nil Einne]] ([[User talk:Nil Einne|talk]]) 22:46, 18 January 2011 (UTC)</small>

:Most of the others have hit the main points here, but I just want to point out that insurance that covers you when you are healthy, but not when you are sick, is not really insurance. That's just a series of subsidized physical exams. The entire point of insurance is that you pool the resources of many so that when others are in hard times, they have resources as well. It is a sign of how screwed up the American medical system has become that the basic notion of insurance has been completely forgotten by most Americans, who just understand it as "the way I pay for routine checkups and prescriptions." It's about spreading the risk, plain and simple. --[[User:Mr.98|Mr.98]] ([[User talk:Mr.98|talk]]) 03:30, 19 January 2011 (UTC)
::The deal with paying for routine checkups is that's how the insurance companies manage their own risk. The costs of dealing with tragic diseases goes up considerably the later they are treated; insurance companies actually save themselves money by making sure you go to the doctor regularly to for physicals, and to get even "nuisance" issues checked (like the non-specific and usually benign but occasionally deadly "Flu-like symptoms"). The ideal "deal" between insurance companies and their customers is something like "You agree to get checked by doctors more often, and we'll cover it when shit goes really bad". Same deal with prescription coverage; people would forgo necessary medicine if it's too expensive, and on the bulk these people will end up costing the insurance companies much more if you end up in the hospital getting treated for something your medicine could have prevented. This is not a defense of the American insurance industry at all; they've basically dropped the ball with regards to their [[fiduciary]] responsibility towards their clients. But in theory, the system is supposed to work as I describe. --[[User:Jayron32|<font style="color:#000099">Jayron</font>]]'''''[[User talk:Jayron32|<font style="color:#009900">32</font>]]''''' 05:26, 19 January 2011 (UTC)
:::Isn't it the insurance company's fiduciary duty to do the bare minimum to brand the sick person with a permanent record of poor health, and help their client (the employer) to get rid of the problem before it amounts to a major expense? [[User:Wnt|Wnt]] ([[User talk:Wnt|talk]]) 18:11, 19 January 2011 (UTC)
:Thank god for the [[National Health Service]]. It has made me disinclined to emigrate to a sunnier cheaper or less crowded country as I would not have the piece of mind of knowing that I would get unlimited free health care if I needed it. See also [[Publicly funded health care]]. [[Special:Contributions/92.24.183.183|92.24.183.183]] ([[User talk:92.24.183.183|talk]]) 13:17, 19 January 2011 (UTC)


:I am the OP, sorry it took me so long to get back to this post....
Edison, you are really sucked into the idea that obesity and high cholesterol is almost all about genetics, when it has little to do with it. The issue here is that for a genetic defect (not sure if this would be the right word) to be present, it would require the proper environment. Almost every single obese person eats horribly and that is a choice. Same with many people with high cholesterol. You can be thin and have high cholesterol, but if you really take a look at it, almost every single person with high cholesterol is overweight, many of them even obese. If you look at both groups, neither one in almost every consumes a "healthy" or a low calorie diet (low cholesterol diet in the high cholesterol patient) and they almost never exercise if they do at all. Most these people "think" they know how to diet, but they have no clue, they think 3000 calories a day or more is a diet. They think exercising is walking around the block or taking the stairs instead of taking the elevator. Try going into the gym and pounding the weights for an hour and then doing 45 minutes of hardcore cardio 5 days a week and then lets see if you still have high cholesterol or are obese after a year doing that and consuming a low calorie diet. I can guarantee you over 90% of the people will be cured as long as they keep it up.

:Now lets discuss choices in life. Everything you do is a "choice." Many or all of them involve some degree of risk. Everyone knows that driving a car involves risk, you can get into an accident. Some people choose to take that risk and some paranoid people or people that have been in very bad accidents that have not recovered emotionally choose to not take that risk. So is it a choice? Yes, it is. At my age, I choose to take the risk of maxing my car out to see how fast it goes on the freeway. I know there is more risk involved going very fast, but if I had children, I would not take that risk. Same with skiing, there is a risk involved. An example that may be more clear would be weight lifting, there is a great deal of risk involved. Personally, I choose not to do some exercises at the gym because of the risk involved. And yes, even with sex and contracting HIV there is a risk. Have you not heard of getting tested yourself and having your partner tested before you have unprotected sex? Do you think its a good idea to have sex with a woman or a male that has had 50 partners in the past? Would you take that risk? Some people would, some people wouldn't. That is called a choice. There is a distinction between acceptable risk and unacceptable risk.

:The real issue here is that many diseases are preventable. Look at most people with type 2 diabetes. If they lose the damn weight and exercise, most of them would be cured. I work with one of them, he has either Jack in the Box, Kentucky fried chicken, or McDonalds every single day. Then he has 2 snickers and peanut M&Ms on top of it. He has high cholesterol and is overweight because of all that junk too. He never goes to the gym and has no interest at all in dieting. His blood sugar was dangerously high before he found out (at work by the way). As previously mentioned, going to the doctor for preventative care can reduce the chances of contracting several diseases or at least you can cut down costs to treat the disease if you catch it early. Most people don't even do that and it costs us healthy people tons of money. Some older women don't get mammograms as often as they should, others don't get them at all. Some older people don't get colonoscopies. Other people don't get their yearly physicals and dont people don't want to pay for their medication and think they know better than their doctor. In the end, it costs us money. It just hurts my wallet and others when I take good care of myself, eat healthy, exercise, and get regular checkups and do everything necessary to stay healthy and other people don't and I have to pay for it. There is no reason why I should have to. ALSO, let me add something to what I originally said. I don't have an issue with supporting people who are disabled and cannot work, I don't have a problem paying for their healthcare. I'm also kind of OK supporting some people who's income is around the poverty level, but then again, I do believe if they really wanted to, they could somehow, someway, come up with the money. I mean just imagine, the last time you really wanted something you could not really afford, didn't you figure out a way to come up with the money? I always do and so do many other people. <span style="font-size: smaller;" class="autosigned">—Preceding [[Wikipedia:Signatures|unsigned]] comment added by [[Special:Contributions/76.169.33.234|76.169.33.234]] ([[User talk:76.169.33.234|talk]]) 09:34, 20 January 2011 (UTC)</span><!-- Template:UnsignedIP --> <!--Autosigned by SineBot-->

::I'm gonna skip all the non-sense rant about having to burden the expenses of other people's choices (Welcome to life in society. Go be an hermit if you don't like it) and go directly to the only point of your post worth commenting on. YOU SHOULDN'T SPEED IN THE FREEWAY JUST FOR THE FUN OF IT. I sincerely hope that you get caught next time you do it, making the freeways safer for the rest of us. [[User:Dauto|Dauto]] ([[User talk:Dauto|talk]]) 16:27, 20 January 2011 (UTC)

== Tidal Locking Formula ==

You have an article [[Tidal locking|Tidal Locking]]

You have two formulas under the section "Timescale"

Two different science textbooks are quoted for the source of these formula:
B. Gladman et al. (1996). "Synchronous Locking of Tidally Evolving Satellites". Icarus 122: 166. [[doi:10.1006/icar.1996.0117]]. (See pages 169-170 of this article.
Formula (9) is quoted here, which comes from S.J. Peale, Rotation histories of the natural satellites, in J.A. Burns, ed (1977). Planetary Satellites. Tucson: University of Arizona Press. pp. 87–112.)

Do both formulas come from the same publication?[[User:PaulNethercott|PaulNethercott]] ([[User talk:PaulNethercott|talk]]) 09:25, 18 January 2011 (UTC)

:I fixed the link for you, and placed a link on the doi from the reference. Hopefully someone has access to read the article and can check for you. [[User:Ariel.|Ariel.]] ([[User talk:Ariel.|talk]]) 10:49, 18 January 2011 (UTC)
::The article is available online [http://audiophile.tam.cornell.edu/randpdf/gladman.pdf here] (PDF). It gives the first formula and cites to Peale. I didn't look thoroughly for the second formula - maybe someone else can do that? (Note that the Wikipedia article does not cite the second formula to this journal article specifically, though perhaps it also comes from here.) Peale does not seem to be available online anywhere. (I searched in google books using a variety of ISBNs that seem to be associated with the book, and I didn't get any results.) If you are interested in verifying the formula in the original source, the Burns book seems to be available in many university libraries and large cities' public libraries,[http://www.worldcat.org/title/planetary-satellites/oclc/3312910] or you can buy it through Barnes & Noble for $1.99.[http://search.barnesandnoble.com/Planetary-Satellites/Joseph-A-Burns/p/9780816505524] [[User:Calliopejen1|Calliopejen1]] ([[User talk:Calliopejen1|talk]]) 10:54, 18 January 2011 (UTC)

== An object hanging on a string ==

On this page: http://www.croomphysics.com/notes/app_hewitt/chapter2.pdf (page 11) I found an answer to a question that confuses me:<br />"Consider what would happen if you suspended a 10-N object midway along a very tight, horizontally stretched guitar string. Is it possible for the string to remain horizontal without a slight sag at the point of suspension?"<br />
Answer: "No way! If the 10-N load is to hang in equilibrium, there must be a supporting 10-N upward resultant. The tension in each half of the guitar string must form a parallelogram with a vertically upward 10-N resultant."<br />
I understand this for a guitar string and a 10-N load, but what if the string is replaced by a strong rope, and the load is only 1 N. Surely it must be possible to pull at both ends of the rope with enough force to make it remain horizontal without a sag??? Actually, I tried this myself with a piece of rope that I pulled apart and a pair of scissors, and there was no perceptible sag. My reasoning is that a stretched rope must be able to exert an upward directed normal force in the same way as a table exerts an upward directed normal force. Am I correct?? [[User:Lova_Falk|<font size="2"><span style="font-family:Segoe Print;color:#e75e03">'''Lova Falk'''</span></font>]] [[User talk:Lova Falk|<font size="2"><span style="font-family:Segoe Print;color:#336699">talk</span></font>]] 13:05, 18 January 2011 (UTC)
:No. Your rope cannot exert a force at right-angles to itself. The only way it can hold something up is if it is pulling up at the point of the suspended object (to cancel the object's gravitational downward pull). If it's pulling up, it must have a vertical component to its direction, which means it's distorted down from perfectly horizontal. Once you recognize "one mass, one string", there's no basis for making a cutoff "only if the mass is less than a certain amount" or "only if the string has certain elasticity". For a very light mass on a very stiff string, the deflection is just very small (''because'' the mass is light, doesn't require much upward pull, and that can be accomplished without much deflection because the string is so non-stretchy). [[User:DMacks|DMacks]] ([[User talk:DMacks|talk]]) 13:13, 18 January 2011 (UTC)
::{{ec}} No, there should always be a sag, but the sag may not be perceptible under certain parameters. A non-ridged string should always sag in the middle if supported only at the ends, regardless of how hard you pull it. What your eye defines for you as "horizontal" may have a higher tolerance than a true horizontal. In fact, horizontal is one of the ''least'' stable positions for such an arrangement, if you look at something like bridges; all [[rope bridge]]s sag in the middle; if you try to make them ''not'' sag, then you end up putting so much force on the ropes as to make them too close to the breaking point for safety. In [[Suspension bridge]]s, you often engineer the roadway to sort-of "reverse sag", that is many such bridges actually have a hill in the middle, in effect redirecting the stress into the ground anchors on either side of the bridge. --[[User:Jayron32|<font style="color:#000099">Jayron</font>]]'''''[[User talk:Jayron32|<font style="color:#009900">32</font>]]''''' 13:16, 18 January 2011 (UTC)
:::Thank you! :) [[User:Lova_Falk|<font size="2"><span style="font-family:Segoe Print;color:#e75e03">'''Lova Falk'''</span></font>]] [[User talk:Lova Falk|<font size="2"><span style="font-family:Segoe Print;color:#336699">talk</span></font>]] 13:26, 18 January 2011 (UTC)
:::Indeed, the rope will sag under its own weight, even if you don't have ''any'' additional suspended mass. If the rope's mass is uniformly distributed along its length, it will form what is known as a [[catenary]] curve. [[User:TenOfAllTrades|TenOfAllTrades]]([[User_talk:TenOfAllTrades|talk]]) 15:12, 18 January 2011 (UTC)
::::Another perspective: in physics problems, 'rope' is often modeled as a mass-less object that has [[tensile strength]] but no [[compressive strength]]. Real ropes DO have compressive strength, especially large diameter ropes of short length. A short length of thick rope (~6" diam X 3' long), anchored at both ends can support an upright load-bearing [[catenary]] like the [[gateway arch]]. This would not usually be considered 'sagging'. [[User:SemanticMantis|SemanticMantis]] ([[User talk:SemanticMantis|talk]]) 17:37, 18 January 2011 (UTC)
:::::I seem to recall an unintentional verse in a book by a distinguished scientist:
:::::For no force, however great,
:::::Can stretch a thread, however fine,
:::::Into a horizontal line
:::::That shall be absolutely straight.
:::::--[[User:Ross Burgess|rossb]] ([[User talk:Ross Burgess|talk]]) 17:44, 19 January 2011 (UTC)
:::::::*The exact wording of [[William Whewell]]'s accidental poem was
::::::::Hence no force however great
::::::::Can stretch a cord however fine
::::::::Into an horizontal line
::::::::Which is absolutely straight.
:::::::[http://books.google.com/books?id=ZL4aAAAAYAAJ&pg=PA44 as you can see here]. Of course, he's talking about the weight of the cord itself, but the sentences just before that are about the case of a weight hanging from a cord whose own weight is negligible. According to [[Martin Gardner]] in a ''Mathematical Games'' column reprinted in ''Martin Gardner's Sixth Book of Mathematical Games from Scientific American'', Whewell was annoyed to learn about the "poem" and had the wording changed in [http://books.google.com/books?id=eQdKAAAAMAAJ&pg=PA88 the next edition]. Gardner misquotes the "poem" slightly, though. --Anonymous, 05:10 UTC, January 20/11.
::::::Even the ''ground'' will sag if enough weight is put on it. Compressive strength means only that the material ''resists'' sagging, not that it avoids it entirely. [[User:Wnt|Wnt]] ([[User talk:Wnt|talk]]) 18:42, 19 January 2011 (UTC)
<!--:::::::See, amongst other articles, [[post-glacial rebound]] for an example of this on vast scales. [[User:Csmiller|CS Miller]] ([[User talk:Csmiller|talk]]) 20:12, 19 January 2011 (UTC) --> <!-- different phenomena, redact ~~~~ -->
:::::::Sure, I just wanted to point out the limitations of the way string is often modeled for simple problems. In particular, a string modeled with no compressive strength cannot take the shape of an upright catenary, but a real rope can. This may help explain some of the confusion between real-world problems and physics 101 type stuff. [[User:SemanticMantis|SemanticMantis]] ([[User talk:SemanticMantis|talk]]) 20:59, 19 January 2011 (UTC)
::::::::<small>What if both ends of the string are attached to [[spherical cow]]s? --[[User:Jayron32|<font style="color:#000099">Jayron</font>]]'''''[[User talk:Jayron32|<font style="color:#009900">32</font>]]''''' 00:57, 20 January 2011 (UTC)</small>

== [[Planck length]] for Idiots ==

This basically translates into "the absolute smallest a thing can be and still physically exist", right? [[User:HalfShadow|<font color="gray">'''Half'''</font>]][[User talk:HalfShadow|<font color="black">'''Shadow'''</font>]] 18:23, 18 January 2011 (UTC)
:Not really. It's a really, really small distance, so there are some theories that it's the fundamental granularity of the universe, but none of the well-accepted, well-proven physics theories hold that there is anything special about that length. It's just the length unit you get if you attempt to scale units such that major physical constants come out to unity ("[[natural units]]"). Keep in mind that the mass unit you get when you do that (the [[Planck mass]], is 2.2 x 10<sup>-9</sup> kg, about the size of a human egg cell (see [[Orders of magnitude (mass)]]. -- [[Special:Contributions/140.142.20.229|140.142.20.229]] ([[User talk:140.142.20.229|talk]]) 18:46, 18 January 2011 (UTC)
:As it says in [[Planck length]]: "In some theories or forms of quantum gravity, it is the length scale at which the structure of spacetime becomes dominated by quantum effects, giving it a discrete or foamy structure, but other theories of quantum gravity predict no such effects." Basically, we don't know what happens at such small scales. --[[User:Tango|Tango]] ([[User talk:Tango|talk]]) 21:56, 18 January 2011 (UTC)
::I think you might fairly say that it's the absolute smallest a ''[[black hole]]'' that can exist. Any smaller, and the [[Compton wavelength]] gets bigger; but the [[Schwartzschild radius]] gets smaller. Which means that the hole is too fuzzy to fit in the hole, so to speak. So if you have a black hole that is evaporating by [[Hawking radiation]], something ought to give out at least mathematically around this time; the [[Heisenberg principle]] says you're not supposed to know where the black hole is that precisely, but how is it going to get away? You can play a merry chase through [[Planck mass]], [[Planck length]], [[Planck momentum]], [[Matter wave]] to figure this out, but one word of caution - I initially looked up [[de Broglie wavelength]] to see how the math worked out and had a curious disagreement by a factor of 2 pi (on the Planck momentum; with the de Broglie wavelength you can't say a mass has a given wavelength without giving it some arbitrary velocity). It turns out that there's a distinction between the "reduced" and "non-reduced" [[Compton wavelength]] as explained in that article. A distinction which I don't understand... you'll need a real physicist for that... :( [[User:Wnt|Wnt]] ([[User talk:Wnt|talk]]) 05:02, 19 January 2011 (UTC)

== Coal mining efficiency ==

I saw on a TV show this gigantic drilling machine many hundreds of feet below the surface around Pittsburgh, PA and started thinking about the massive amount of energy that must be needed to turn the thing's huge and undoubtedly heavy drill bits against the resistance of the earth it was drilling through, and the energy needed to move the thing forward, along with all the energy used for the other trolley cars, lights, ventilation systems, and of course the energy expenditures to raise the mined material back to the surface, and I wonder roughly how much energy <i>quantified as an amount of coal</i> (I know the machines don't take coal for fuel, but I want to know the figure in terms of coal) the average coal mining plant expends in all the actions it takes to extract 100 tons of coal? Just a very ballpark estimate is all I'm after here. For instance, do they use about a ton of coal's worth of energy to get 100 tons? Do they use over 50 tons' worth? Thanks. [[Special:Contributions/20.137.18.50|20.137.18.50]] ([[User talk:20.137.18.50|talk]]) 18:38, 18 January 2011 (UTC)
:This is a very important and valuable analytic tool: it is formally known as [[EROEI]] (energy returned on energy invested). It is heavily studied by economists, business analysts, and geoscientists and engineers in the mining and production industries. I know quite a bit about the ballpark numbers for the oil industry, but unfortunately not for coal; I imagine coal mining experts know similar metrics for coal mining. I have heard reputable petrophysicists claim that [http://www.theoildrum.com/node/3839 the EROI for tar sands] is dramatically and dangerously approaching 1.0 in the long run; this means that it is economically unsustainable (in other words, you must use one full barrel of oil to power the extraction of one barrel of oil). Coal mining is different from petroleum, because the energy to extract coal comes from many sources: electricity (... from coal); chemical energy (from explosives... and you could trace the energy budget farther back to the factory, to electricity, and again, ...to coal); and petroleum (...imported from the oil industry). The oil industry, on the other hand, powers almost all of its field operational energy budget using energy derived from oil - so the budget is easier to balance. (Machines, drills, motors, trucks, and so on, are all diesel or even fuel-oil powered). As discussed in the [http://www.theoildrum.com/node/3839 article I just linked], the EROI is very difficult to measure exactly. Nonetheless, the EROIE definitely does impact the business model of large-scale energy extraction. Here is an OilDrum node on [http://www.theoildrum.com/node/3496 the EROI of coal]. Let me know if you need help deciphering the energy-budget numbers in there. ("Lower-" and "Higher-" heating values, and so on, are all "engineering adjustments" that cloud the nice, pure theoretical thermodynamic analysis of "how many pounds of coal did we burn?") And, as one post anecdotally claims, if you still live in an area where coal can be mined with a pick and shovel, (like Indonesia), you burn zero pounds of coal to extract 1 pound of coal - a net EROI of "infinity." I would [[WP:OR|posit]], based on comparative prices between coal and oil in [[Barrel of oil equivalent|BBOE]], that coal extraction is ''significantly'' more energy-efficient than oil - let's say, 1 ton of coal burned to extract 10 tons of coal (in a developed, mechanized, American Wyoming strip mine). In the spirit of providing solid, numerical references, there's no substitute for the cold, hard economic and production statistics collected by the United States Department of Energy, and made available at no charge to the public through the [http://www.eia.gov/ Energy Information Administration] website. [[User:Nimur|Nimur]] ([[User talk:Nimur|talk]]) 20:57, 18 January 2011 (UTC)

== Stability ==

Did [[first-rate]] [[ships of the line]] such as the [[HMS Victory]] have any stability problems? It looks very top-heavy. Did it go a long way under the water? And does anyone have a picture of a model of it out of water? --<span style="text-shadow:grey 0.3em 0.3em 0.1em">'''[[User:THFSW|<font color="black">T H F S W</font>]]''' (''[[User talk:THFSW|T]] '''·''' [[Special:Contributions/The High Fin Sperm Whale|C]] '''·''' [[Special:EmailUser/The High Fin Sperm Whale|E]]'')</span> 18:46, 18 January 2011 (UTC)
:Some of your questions are obliquely answered in the article [[Ship of the line]]. In the earliest era of European warship building when trial-and-error and guesstimation ruled, stability was indeed a problem, as witnessed by the fates of HMS [[Mary Rose]] (1509) and Sweden's [[Vasa (ship)|Vasa]], but by Victory's era [[Naval architecture|Naval architects]] worked by well-established scientific laws. Some of the more basic means of achieving stability are [[Sailing ballast|ballast]] below the waterline and the '[[Tumblehome|tumblehome]]' or inward angling of the upper decks, which significantly improved weight distribution. [[Special:Contributions/87.81.230.195|87.81.230.195]] ([[User talk:87.81.230.195|talk]]) 19:25, 18 January 2011 (UTC)
::And does anyone have a picture of what the Victory would look like out of water? --<span style="text-shadow:grey 0.3em 0.3em 0.1em">'''[[User:THFSW|<font color="black">T H F S W</font>]]''' (''[[User talk:THFSW|T]] '''·''' [[Special:Contributions/The High Fin Sperm Whale|C]] '''·''' [[Special:EmailUser/The High Fin Sperm Whale|E]]'')</span> 19:30, 18 January 2011 (UTC)
:::Well seeing as the Victory has been "out of the water"/in dry-dock for almost a century, I'd expect that most photographs that exist of the hull are "out of the water". [[User:Dodger67|Roger]] ([[User talk:Dodger67|talk]]) 20:12, 18 January 2011 (UTC)
::::My mistake, I thought it was in the water. Still, that doesn't answer my question: does anyone know where I could find a picture of the whole thing, or an accurate model? I mostly want to see the bottom. --<span style="text-shadow:grey 0.3em 0.3em 0.1em">'''[[User:THFSW|<font color="black">T H F S W</font>]]''' (''[[User talk:THFSW|T]] '''·''' [[Special:Contributions/The High Fin Sperm Whale|C]] '''·''' [[Special:EmailUser/The High Fin Sperm Whale|E]]'')</span> 20:53, 18 January 2011 (UTC)
:::::I think [http://www.victorymodel.com/Images/Photographs/HullChronology/HullChronology.htm this site] might be helpful. There are a number of reasonably good photos of what appears to be an accurate scale model, particularly [http://www.victorymodel.com/Images/Photographs/HullChronology/images/hull13_jpg.jpg this one]. [[User:Karenjc|<font color="red">Ka</font>]][[User_talk:Karenjc|renjc]] 21:55, 18 January 2011 (UTC)
::::::<small>Out of water? Drink rum! Arrrrhhhh. [[User:Clarityfiend|Clarityfiend]] ([[User talk:Clarityfiend|talk]]) 07:37, 19 January 2011 (UTC)</small>
A [[centreboard]] is a useful addition for stability. Trouble is, that article doesn't give much history. I'm personally aware of them being used on some sailing ships of the nineteenth century. [[User:HiLo48|HiLo48]] ([[User talk:HiLo48|talk]]) 22:07, 18 January 2011 (UTC)
:<small>Personally aware? How old are you? [[User:DuncanHill|DuncanHill]] ([[User talk:DuncanHill|talk]]) 00:06, 19 January 2011 (UTC)</small>
::A centreboard or keel, as I learned earlier here, is mostly to keep the ship in a straight line. Anyways, thank you, the site Karenjc pointed out is exactly what I was looking for. --<span style="text-shadow:grey 0.3em 0.3em 0.1em">'''[[User:THFSW|<font color="black">T H F S W</font>]]''' (''[[User talk:THFSW|T]] '''·''' [[Special:Contributions/The High Fin Sperm Whale|C]] '''·''' [[Special:EmailUser/The High Fin Sperm Whale|E]]'')</span> 00:37, 19 January 2011 (UTC)
:::Actually, a centerboard is mostly to keep a ship from drifting sideways with the wind. As a side effect, it does reduce rolling, but it adds little if anything to improve real stability, as it does not significantly change [[metacentric height]]. A keel can serve either or both purposes, drift reduction and increasing stability. --[[User:Stephan Schulz|Stephan Schulz]] ([[User talk:Stephan Schulz|talk]]) 19:35, 19 January 2011 (UTC)

= January 19 =

== Volume (sound) ==

How did it come to be that Volume became associated with loudness?[[User:Smallman12q|Smallman12q]] ([[User talk:Smallman12q|talk]]) 00:09, 19 January 2011 (UTC)

:According to the [http://www.oed.com/view/Entry/224527?rskey=zJDZkc&result=1&isAdvanced=false# OED], the first use of the word "volume" to mean loudness of sound occurs in [[Byron]]'s ''Werner'', 1822: {{color|purple|"I heard‥, Distinct and keener far upon my ear Than the late cannon's volume, this word—‘Werner!’"}}. And it was defined in a musical dictionary as early as 1786 to refer to {{color|green|"the compass of a voice from grave to acute: also to its tone, or power"}}. [[User talk:WikiDao|<span style="font-family: Segoe print;">WikiDao</span>]] [[User:WikiDao|<span style="color:#000;">&#9775;</span>]] 00:40, 19 January 2011 (UTC)

: We've done this one [[Wikipedia:Reference_desk/Archives/Science/2010_December_11#Musical_volume | before]]. [[Special:Contributions/213.122.48.63|213.122.48.63]] ([[User talk:213.122.48.63|talk]]) 01:09, 19 January 2011 (UTC)

::Yes, [[Wikipedia:Reference_desk/Archives/Science/2010_December_11#Musical_volume| here]]. [[User:Dbfirs|''<font face="verdana"><font color="blue">D</font><font color="#00ccff">b</font><font color="#44ffcc">f</font><font color="66ff66">i</font><font color="44ee44">r</font><font color="44aa44">s</font></font>'']] 08:35, 19 January 2011 (UTC)

== Fire in a Microwave Oven? ==

Recently, a friend had me watch a video of someone putting a candle in a microwave oven and turning it on. This produced some neat effects, supposedly creating "plasma". (The video was not made by professionals.) My question(s) are: Is this actually plasma? And regardless whether it is or not, what exactly is happening?
<sup>[[User:Avicennasis|<font color="red">Avic</font>]]</sup>[[User talk:Avicennasis|<sub><font color="blue">ennasis</font>]]</sub><small> @ 07:19, 14 Shevat 5771 / 19 January 2011 (UTC)</small>

: I have seen such experiments in reality. a [[flame]] is already very slightly ionized. Apparently, the microwaves transfer energy to the plasma, ionizing it further and enlarging it, though I don't know the exact mechanism by which this happens. [[Special:Contributions/157.193.175.207|157.193.175.207]] ([[User talk:157.193.175.207|talk]]) 07:47, 19 January 2011 (UTC)

:Is there a video online of this anywhere please? [[Special:Contributions/92.24.183.183|92.24.183.183]] ([[User talk:92.24.183.183|talk]]) 13:23, 19 January 2011 (UTC)

::May not be the same video that Avicennasis saw, but it certainly is a [http://www.youtube.com/watch?v=r6PPq8GwZgU candle in a microwave] [[User:Darigan|Darigan]] ([[User talk:Darigan|talk]]) 13:27, 19 January 2011 (UTC)
:::It's even more impressive when you do it with a [http://www.youtube.com/watch?v=0i2lhO3bSjQ&t=1m42s grape]. [[Special:Contributions/90.193.232.5|90.193.232.5]] ([[User talk:90.193.232.5|talk]]) 14:20, 19 January 2011 (UTC)

::::There are ''many'' such videos on YouTube. If you can name it, someone has microwaved it, with comic effect. "Coconut in a microwave" isn't as impressive as you'd think, but I remember when I looked for it, I found it.
::::There is a wide subset of videos about "[[ball lightning]]" in a microwave. I ''think'' that this is overly promoted - it is simply that fire is conductive and absorbs more energy, and sometimes the gasses continue to glow a moment after they escape from a hole, etc. that has been built for this purpose. I don't think that the sort of energy or structure present in real ball lightning is actually duplicated --- but if someone disagrees, by all means let us know! [[User:Wnt|Wnt]] ([[User talk:Wnt|talk]]) 17:55, 19 January 2011 (UTC)

== Need a Formula ==
i need information the formula 10extera abrasive becuse used oxalic acid <span style="font-size: smaller;" class="autosigned">—Preceding [[Wikipedia:Signatures|unsigned]] comment added by [[Special:Contributions/85.185.25.251|85.185.25.251]] ([[User talk:85.185.25.251|talk]]) 07:30, 19 January 2011 (UTC)</span><!-- Template:UnsignedIP --> <!--Autosigned by SineBot-->
:I think you need to make your question a little clearer. --[[User:ColinFine|ColinFine]] ([[User talk:ColinFine|talk]]) 08:48, 19 January 2011 (UTC)
::Does [http://www.erowid.org/archive/rhodium/chemistry/oxalic.otc.html Extracting oxalic acid from household cleaner] help? [[User:Cuddlyable3|Cuddlyable3]] ([[User talk:Cuddlyable3|talk]]) 11:42, 19 January 2011 (UTC)

== Why do frozen vegetables taste so bland? ==

It's not just the brand, but this is the kind of products I'm wondering about.[http://products.peapod.com/12585.html] How are they nutrition-wise? Thanks. [[User:Imagine Reason|Imagine Reason]] ([[User talk:Imagine Reason|talk]]) 12:58, 19 January 2011 (UTC)
:I understand the process of freezing damages the cell-structure (water/liquid expanding as it freezes), and then means that when cooked they lose some of their flavour (I know, I've worded that terribly, but have a quick read of the [[Frozen vegetables]] article, and I'll have a look for some bette source)... [http://books.google.co.uk/books?id=IGwfMAZ60t0C&pg=PA344&dq=frozen+vegetables+comparison&hl=en&ei=keI2TeTjPMnDhAfWhMTSAg&sa=X&oi=book_result&ct=result&resnum=4&sqi=2&ved=0CEQQ6AEwAw#v=onepage&q=frozen%20vegetables%20comparison&f=false this] extract points to the difference in texture between frozen and fresh veg.[http://books.google.co.uk/books?id=HNq2du_u_xoC&pg=PA36&dq=frozen+vegetables+risk&hl=en&ei=F-M2TanWMYyGhQf8qoXuAg&sa=X&oi=book_result&ct=result&resnum=3&ved=0CDMQ6AEwAg#v=onepage&q=frozen%20vegetables%20risk&f=false this] source argues that frozen veg are higher in nutritional value than fresh (or, as the case may be, not entirely fresh vegetables). [http://books.google.co.uk/books?id=rGCdFJISc20C&pg=PA37&dq=frozen+vegetables+risk&hl=en&ei=F-M2TanWMYyGhQf8qoXuAg&sa=X&oi=book_result&ct=result&resnum=8&ved=0CE4Q6AEwBw#v=onepage&q=frozen%20vegetables%20risk&f=false this] source also argues that frozen veg are less at risk from contamination, as long as they are not frozen, defrosted and then refrozen - Although, having said that, I am sure that I have seen other sources before that argue that frozen veg in bags that have not been properly sealed are at risk from contamination. [[User:Darigan|Darigan]] ([[User talk:Darigan|talk]]) 13:16, 19 January 2011 (UTC)
:It could also be related to the way they are cooked after being frozen -- are you boiling them in water? I've always found that makes veggies taste a lot more bland. -- [[User:Jsbillings|<span style="color:green">JSBillings</span>]] 13:48, 19 January 2011 (UTC)
::OR answer here: I have found that frozen vegetables taste better when steamed than when boiled; the "steam in the bag they came in" varieties are particularly close to their fresh cousins. Some vegetables take to freezing better than others. I find frozen carrots and frozen corn-on-the-cob to be pale comparisons to their fresh versions, while green beans (haricots) and garden peas are usually fairly comparable. I'm not much of a fan of many [[cruciferous vegetables]] in any form, so I can't comment on frozen vs. fresh broccoli or cauliflower or brussel sprouts. --[[User:Jayron32|<font style="color:#000099">Jayron</font>]]'''''[[User talk:Jayron32|<font style="color:#009900">32</font>]]''''' 15:48, 19 January 2011 (UTC)
:::With peas, it's vastly superior to freeze (or can, though I find canned peas unappetizing) them rather than shipping them "fresh" to a supermarket: the sugar in peas breaks down incredibly rapidly, making it nearly impossible to get good fresh peas, other than by growing them yourself. This is mentioned in [[Pea#Use]], though I'm surprised we don't have anything more on this. I think typically, peas need to be [[Blanching (cooking)|blanched]] and preserved within a couple of hours of being picked. [[User:Buddy431|Buddy431]] ([[User talk:Buddy431|talk]]) 17:15, 19 January 2011 (UTC)
:::I think your example of corn is illustrative of a few things. Really fresh corn (as in, you put the pot on to boil while you run out to pick the ears) is unbelievably good; it barely even needs a blanching, you just want to warm it up for the butter. Within a day, it's good, but not nearly the same kind of thing. If you [[IQF]] freeze it at that point, you'll get a pretty decent product - good enough that it's better than the so-called fresh corn you find in grocery stores that's already 2-3 days old, if not more. Canned corn is pretty abysmal. So you've got a weird sort of graph where really fresh is best, but frozen is better than just about anything else. The trouble with many vegetables, particularly carrots and peas, is that freezing them wrecks the crispness; the flavour might not change much, but the mouth feel is completely different. [[User:Matt Deres|Matt Deres]] ([[User talk:Matt Deres|talk]]) 19:00, 19 January 2011 (UTC)
:::::IQF seems to mean individually quick frozen in this context - it stops the corn/peas etc from sticking together. [[User:Csmiller|CS Miller]] ([[User talk:Csmiller|talk]]) 19:37, 19 January 2011 (UTC)
::::That's the problem I have with frozen corn: it loses the snappy mouthfeel that makes fresh corn-on-the-cob so tasty. I usually buy my corn at the local farmer's market the day I plan to use it; when it is in season it's usually picked within 24 hours before I buy it. I find all canned vegetables taste more like ''can'' than vegetable, so I avoid them pretty much at all cost. --[[User:Jayron32|<font style="color:#000099">Jayron</font>]]'''''[[User talk:Jayron32|<font style="color:#009900">32</font>]]''''' 19:23, 19 January 2011 (UTC)
:::::You might find the article on [[Clarence Birdseye]] interesting. [[User:Vespine|Vespine]] ([[User talk:Vespine|talk]]) 22:09, 19 January 2011 (UTC)

::::::Wouldn't the vegetables that are used by the frozen vegetables processors be those most manageable for that process? There are lots of varieties of a vegetable. Those destined for freezing are likely to be those that have the handling characteristics that best lend them to mechanical manipulation—from the field to the packaged and frozen product. If any tradeoffs have to be made, taste might get sacrificed in favor of the ease of large scale mechanized agriculture. Even the nonorganic but fresh vegetables on the supermarket shelf might tend to be those varieties of a given vegetable that don't require those qualities that lend themselves so well to the highly mechanized freezing (and harvesting) process but may instead retain some of those qualities that made them desirable in times gone by. [[User:Bus stop|Bus stop]] ([[User talk:Bus stop|talk]]) 22:23, 19 January 2011 (UTC)

:::::The one exception, I think, is tomatoes, where the canned ones can have better flavor than either fresh or frozen. Freezing destroys them, and even farmer's markets can't usually handle them at the peak of flavor -- canners, however, can process them directly out of the field. [[User:Looie496|Looie496]] ([[User talk:Looie496|talk]]) 00:12, 20 January 2011 (UTC)
::::::I'll second that; canning tomatoes also has the practical effect of concentrating the flavor components. I find good canned tomatoes to be more intensely "tomatoey" than even fresh-off-the-vine-minutes-before tomatoes. Which is not to say that fresh tomatoes don't have their place; canned tomatoes make a better sauce than fresh, but I wouldn't ever use them in a salad. --[[User:Jayron32|<font style="color:#000099">Jayron</font>]]'''''[[User talk:Jayron32|<font style="color:#009900">32</font>]]''''' 01:27, 20 January 2011 (UTC)

== Electrical ==

Hi I have some basic doubts in electrical I am an electrical student please help me
1)Why we maintain voltage as constant not current?

2)In producing AC there comes both +ve AND -ve voltage in both the ends (slip rings) but in power line only phase has while neutral remain null.

3)Is human body a conductor?

4)If a coil connected to power supply produces magnetic flux will that flux link on that same coil and induce a emf?

5)Fan is an inductive load thus on using it there should be 90° phase out b/w Amps and Volt if so won't it affect Transformer?

6)If a bulb excited by AC what happen when it changed to DC of same rms value of volt and current?[[User:Kanniyappan|Kanniyappan]] ([[User talk:Kanniyappan|talk]]) 14:08, 19 January 2011 (UTC)

:Regarding #3, everything is a conductor under sufficient voltage. The human body, being basically a giant bag of salt water (an [[electrolyte]]) is actually a pretty decent conductor, even under moderate voltages. Regarding #4, in order to produce an emf through the coil, you must ''move'' the coil in relation to the magnet (or visa-versa). Regarding #6, my understanding is that light bulbs (and resistance heaters, and other similar devices) behave roughly the same under AC and DC conditions, as the effects of [[resistive heating]] are the same regardless of the direction of the current; in other words the AC or DC nature of the current does not play into [[Joule's first law]]. I will let someone who actually knows about this answer the rest (and correct my mistakes). --[[User:Jayron32|<font style="color:#000099">Jayron</font>]]'''''[[User talk:Jayron32|<font style="color:#009900">32</font>]]''''' 15:42, 19 January 2011 (UTC)

::It is not possible to maintain a constant current except under carefully controlled conditions. "Voltage" is another name for "electro-motive force" (EMF) which you can compare with "pressure" in water pipes. The supply of both electrical and water services aims to maintain a constant "pressure" so that the amount of electricity or water that flows when something is turned on can be reliably predicted. In the case of water, it is usually the tap that controls how much water flows, but, for electricity, it is usually the resistance or reactance of the appliance that determines how much current flows (though dimmer switches work like taps).
::AC supply does '''not''' have positive and negative. Usually one side of the supply is connected to earth and for this reason is called "neutral", but it doesn't really matter whether this is done or not except for safety reasons. Both "live" and "neutral" wires behave in exactly the same way (except when one is shorted to earth), so there is really no difference between them if they are isolated from "earth". Three phase supply is generated and transmitted with three "lives", each one being 120 degrees out of phase with each of the others. Sometimes there is no "neutral" (delta configuration), and power can be transmitted without any other conductor. More often, each of the phases is generated at a fixed voltage to a common wire "3-phase neutral" which is usually connected to earth. The three main wires for the three phases still carry the main current because the three currents in the three phases will cancel each other out if they are equal, but the common neutral serves to carry any residual current if the phases are not balanced.
::Just one minor addition to Jayron's answer to #4: If AC is connected to a coil, then the flux does indeed produce an induced EMF in the same coil becuase the current is constantly changing. See [[autotransformer]]. [[User:Dbfirs|''<font face="verdana"><font color="blue">D</font><font color="#00ccff">b</font><font color="#44ffcc">f</font><font color="66ff66">i</font><font color="44ee44">r</font><font color="44aa44">s</font></font>'']] 18:24, 19 January 2011 (UTC)
:::To continue dbfirs points, see [[earthing system]]s (most houses use TN or TT, where neutral is tied to ground, see IT for why it is done), [[three-phase power supply]], and [[single wire earth return]] for other reasons for earthing neutral. [[User:Csmiller|CS Miller]] ([[User talk:Csmiller|talk]]) 19:34, 19 January 2011 (UTC)
::::As far as I know, single-wire earth return is never used in the UK. (I've tried it, unofficially, but probably illegally.) [[User:Dbfirs|''<font face="verdana"><font color="blue">D</font><font color="#00ccff">b</font><font color="#44ffcc">f</font><font color="66ff66">i</font><font color="44ee44">r</font><font color="44aa44">s</font></font>'']] 22:17, 19 January 2011 (UTC)
:::::A/C can have potential on both sides of circuit (although obviously not positive/negative, since it's A/C). Voltage can only be measured as a difference - there is no such thing as an absolute voltage. A/C in households is designed so that the earth is one side of the circuit, and since we stand on the earth, to us the feels like neutral. But it doesn't have to be that way. If you isolate your A/C generator completely from the Earth, then both sides "have" potential (although of course that's meaningless since you need two sides to measure anything). An interesting side effect of this kind of isolation is that you can't get shocked by touching just one wire - you need two to get shocked. Unless of course two different people happen to each touch one wire at the same time and you complete the circuit through both of them via the Earth. You can also make D/C with one side hot and the other neutral - you just need to attach one side to the earth. [[User:Ariel.|Ariel.]] ([[User talk:Ariel.|talk]]) 22:35, 19 January 2011 (UTC)
::::::<small>Well if you want to be pedantic, AC does have positive and negative on both sides, it just changes 100 or 120 times each second, so I meant that you can't label either conductor positive or negative. Good point that DC can have "live" and "neutral". Is it often supplied this way (other than in motor vehicles)? </small> [[User:Dbfirs|''<font face="verdana"><font color="blue">D</font><font color="#00ccff">b</font><font color="#44ffcc">f</font><font color="66ff66">i</font><font color="44ee44">r</font><font color="44aa44">s</font></font>'']] 23:09, 19 January 2011 (UTC)
:::::::Most circuit diagrams have a source and ground, rather that a positive and negative. Computer power supplies are also wired this way - a common ground for everything, and different wires for various voltages/rails/polarities. [[User:Ariel.|Ariel.]] ([[User talk:Ariel.|talk]]) 00:17, 20 January 2011 (UTC)
::Real mains fans have some corrective circuitry to bring the current much closer in phase to the voltage. See [[power factor]]. [[User:Dbfirs|''<font face="verdana"><font color="blue">D</font><font color="#00ccff">b</font><font color="#44ffcc">f</font><font color="66ff66">i</font><font color="44ee44">r</font><font color="44aa44">s</font></font>'']] 18:37, 19 January 2011 (UTC)

# Mains supplies provide a constant voltage so that any number of appliances can be connected and disconnected without affecting one another. Each receives the standard voltage and only the total current varies. A constant current mains supply is impractical because the current to an appliance would change every time another appliance was connected, and dangerously high voltage will be delivered if no appliance is connected.
# A basic two-wire or "single-phase" AC supply usually has one wire called neutral connected to earth which we may call 0V. The voltage on the other wire alternates between +ve and -ve voltages.
# The human body is a conductor. See the article [[Electric shock]].
# An emf is induced in a wire in a ''changing'' magnetic field. A coil is an example of an inductive load. When a coil is connected to a DC power supply, the current in the coil grows slowly because it produces an increasing magnetic field that induces an emf that opposes the supply voltage. For the same reason, when a coil is connected to an AC supply the current is slow to follow the voltage changes. The time delay can be expressed in degrees where a full alternating cycle is 360. If the coil has no resistance, the current lags the AC voltage by 90.
# A fan motor is an inductive load. When the fan shaft rotates the motor is also a generator of emf that opposes the supply voltage.
# The filament of a lamp bulb glows because of the heating effect of current flowing through its resistance. The heating power is proportional to the voltage squared (so both positive and negative voltages give heating). An AC supply with rms (root mean square) voltage equal to the voltage of a DC supply has the same heating power as the DC supply. So there will be no change in the lamp brightness if one switches between the two supplies. Actually the brightness flickers at double the frequency of the AC cycles but you cannot see this because mains supply frequency is too high. [[User:Cuddlyable3|Cuddlyable3]] ([[User talk:Cuddlyable3|talk]]) 22:59, 19 January 2011 (UTC)

== Rephrased: relaxing voices ==

Sorry to the board, it wasn't supposed to be a medical question. I will attempt to rephrase the question. Why do some voices, e.g., Bob Ross's, cause people to relax? I've read on other forums that people will watch his show to help them fall asleep. Thank you very much! [[User:Reflectionsinglass|Reflectionsinglass]] ([[User talk:Reflectionsinglass|talk]]) 18:54, 19 January 2011 (UTC)
:To answer a question with a question, why is some music relaxing and other music exciting? Why do some people find traffic noises soothing, and others find it irritating? [[User:Ariel.|Ariel.]] ([[User talk:Ariel.|talk]]) 22:37, 19 January 2011 (UTC)

== Ice expanding, minimum density ==
It is my understanding that when water is cooled (at [[standard temperature and pressure|standard pressure]]), it reaches a maximum density at 4&deg;C (due to [[hydrogen bonds]], and then starts to expand as it is cooled further. As it transitions to Ice Ih it expands by about 9%, and then continues to expand as it cools further. At what temperature does ice reach its minimum density? I did a quick websearch and found [http://www.btinternet.com/~martin.chaplin/explan2.html#icedensity] which indicates that it is at 70K. Is the minimum density really this cold? <small><span class="autosigned">— Preceding [[Wikipedia:Signatures|unsigned]] comment added by [[User:Csmiller|Csmiller]] ([[User talk:Csmiller|talk]] • [[Special:Contributions/Csmiller|contribs]]) 20:10, 19 January 2011 (UTC)</span></small><!-- Template:Unsigned --> <!--Autosigned by SineBot-->
:Under [[Ice#Characteristics]] it says: "The density of ice is 0.9167 g/cm³ at 0°C [...] Density of ice increases slightly with decreasing temperature and has a value of 0.9340 g/cm³ at −180 °C (93 K).", quite the opposite of your statement. But it doesn't say what ice does on further cooling or where minima/maxima of density are. Another image on commons ([http://commons.wikimedia.org/wiki/File:Wasseranomalie.png]) also implies that density rises when ice is cooled. Oh, just read through your link again, and as I understand it, ice increases its density when heated up ''starting from 0K'' to 70K, then it apparently decreases again--[[Special:Contributions/178.26.171.11|178.26.171.11]] ([[User talk:178.26.171.11|talk]]) 21:03, 19 January 2011 (UTC)EDIT: The minimum density should be at 0°C then (unless there's a way of "superheating" ice above that without melting it at standard pressure.--[[Special:Contributions/178.26.171.11|178.26.171.11]] ([[User talk:178.26.171.11|talk]]) 21:05, 19 January 2011 (UTC)

:A very similar question was asked a couple weeks ago.[http://en.wikipedia.org/wiki/Wikipedia:Reference_desk/Archives/Science/2011_January_7#At_what_temperature_does_the_volume_of_water_decrease.2C_regardless_of_whether_your_increase_or_decrease_the_temperature.3F] [[User:Red Act|Red Act]] ([[User talk:Red Act|talk]]) 23:23, 19 January 2011 (UTC)

== Allergen ==

I'm trying to pin down whether an item would be a high risk to cause an allergic reaction. The ingredients listed on the [http://www.cleanandgreenlife.com/norwex-enviro-products-norwex-mattress-cleaner-p-281.html website] for it are: ''Water, denatured alcohol (<5%), sodium iminodisuccinate (<1%), zinc diricinoleate (<1%), cocamidopropyl betaine (<1%), menthol (<1%), microorganisms (<1%), citric acid (<1%), polydimethylsiloxane emulsion (<1%). The microorganisms in this product are non-genetically modified, naturally sourced, non-pathogenic (i.e. do not cause disease) dormant bacteria, and are the primary “active ingredients” in this formulation. They work by eating up all of the accumulated dander, dead dust mites, and dust mite fecal matter that has accumulated in your mattress/ pillows/ furniture/ car upholstery over time, eventually converting those allergens into carbon dioxide and water.''<br>
Obviously, people can be allergic to just about anything, but do any of these items seem like likely candidates? Their use of "naturally sourced" bacteria kind of raised my eyebrow, for one thing. I've heard of cleansers touting their "enzymatic action!" before, but not bacterial. Anything else on there look weird? [[User:Matt Deres|Matt Deres]] ([[User talk:Matt Deres|talk]]) 20:37, 19 January 2011 (UTC)
:Wow. That's like an ingredient list that includes ''"...chemicals. All our chemicals are safe, trust us..."'' To give a parallel example with a more defined set of natural microorganisms, consider the regulation of raw-milk [[cheese]]s by the U.S. and Australia. [[User:Wnt|Wnt]] ([[User talk:Wnt|talk]]) 20:52, 19 January 2011 (UTC)
:You also may take interest in the zinc diricinoleate (the basic component there is [[ricinoleic acid]], with two conjugated to zinc. See [http://www.ncbi.nlm.nih.gov/pubmed/18080873] [http://www.ncbi.nlm.nih.gov/pubmed/11205395] [http://www.ncbi.nlm.nih.gov/pubmed/3621934] for some leads. (It ''is'' widely used, but I'd like to know who puts castor oil in ''chocolate''? Remind me to avoid that brand...) [[User:Wnt|Wnt]] ([[User talk:Wnt|talk]]) 21:03, 19 January 2011 (UTC)

::Castor oil in chocolate?! Sounds like something the [[Crunchy Frog|Whizzo Chocolate Company]] would trot out. [[User:Matt Deres|Matt Deres]] ([[User talk:Matt Deres|talk]]) 23:12, 19 January 2011 (UTC)
:::Apparently this is being done by [[Hershey's]], [[Nestle]], and [[Mars, Incorporated]], as part of a campaign to sell cocoa butter to the cosmetics industry and replace it with cheap substitutes in chocolate.<sup>[http://candyrecapper.com/pgpr.html]</sup> Which is just one small reason why American chocolate doesn't taste like chocolate. And people wonder why this country keeps running record trade deficits... [[User:Wnt|Wnt]] ([[User talk:Wnt|talk]]) 04:18, 20 January 2011 (UTC)

:This site: http://www.cosmeticsdatabase.com/ is pretty good for that kind of thing. You can either look up individual ingredients, or register an account and enter all the ingredients of a product and it will give you a grade. (Not just allergies, any kind of potential risk.) [[User:Ariel.|Ariel.]] ([[User talk:Ariel.|talk]]) 22:52, 19 January 2011 (UTC)

::That site looks helpful - thanks; I'll give it a look when I get some time to play around. I don't know if it matters, but the substance in question isn't a cosmetic, it's for spraying on beds and other furniture to reduce dust mite allergens and odour. [[User:Matt Deres|Matt Deres]] ([[User talk:Matt Deres|talk]]) 23:12, 19 January 2011 (UTC)

==Water and brick question==
My daughter has a science project in 5th grade this week. I was watching the discovery channel last night and the segament was about taking a 5gal. water jug,filling it a 1/4 with wate, connecting a hose to it,then connecting the end to a hot water bottle.Then take 5 blocks and put them on top of the water bottle.Once that is complete,turn the 5gal. jug upside down about 4ft. high ,then the water should fill the bottle and knock off the bricks. What is the topic of this project? <small><span class="autosigned">— Preceding [[Wikipedia:Signatures|unsigned]] comment added by [[User:Jeffryan777|Jeffryan777]] ([[User talk:Jeffryan777|talk]] • [[Special:Contributions/Jeffryan777|contribs]]) 22:01, 19 January 2011 (UTC)</span></small><!-- Template:Unsigned --> <!--Autosigned by SineBot-->
:Can't you ask your daugher? (I added a header) [[User:Smartse|SmartSE]] ([[User talk:Smartse|talk]]) 22:37, 19 January 2011 (UTC)
::The topic appears to be pressure, in that the head of water in the hose is sufficient to lift the bricks. --[[User:Tagishsimon|Tagishsimon]] [[User_talk:Tagishsimon|(talk)]] 22:41, 19 January 2011 (UTC)
:::This is a demonstration of [[hydraulics]] using [[communicating vessels]], the field is [[fluid mechanics]].[[User:Vespine|Vespine]] ([[User talk:Vespine|talk]]) 22:46, 19 January 2011 (UTC)
::::You could also discuss [[conservation of energy]]. Lifting the 5 gallon jug takes [[Work (physics)|work]] (in the literal and technical sense of the word). That work is transferred, via the hydraulic pressure, to lift the blocks. One useful property of [[Hydraulics|hydraulic machines]] is that they can ''multiply force'' - so it can be possible to lift some pretty heavy cinder-blocks (which requires a large force), by exerting a smaller force over a large distance to lift the water (hence, to pressurize the fluid). (The force is increased, but the ''energy'' is conserved). [[User:Nimur|Nimur]] ([[User talk:Nimur|talk]]) 23:33, 19 January 2011 (UTC)
:::::And because of this force multiplying, hydraulic systems similar to the one described can act like many types of levers, pulleys and other [[simple machine]]s. [[User:SemanticMantis|SemanticMantis]] ([[User talk:SemanticMantis|talk]]) 00:02, 20 January 2011 (UTC)

== t (radiation and testosterone)==

does radiation lower testosterone <small><span class="autosigned">— Preceding [[Wikipedia:Signatures|unsigned]] comment added by [[User:Tommy35750|Tommy35750]] ([[User talk:Tommy35750|talk]] • [[Special:Contributions/Tommy35750|contribs]]) 22:06, 19 January 2011 (UTC)</span></small><!-- Template:Unsigned --> <!--Autosigned by SineBot-->

"Radiation therapy may lower testosterone to the low-normal range in some men" [http://books.google.co.uk/books?id=rQfOWgQi_VoC&pg=PA610&lpg=PA610&dq=radiation+lower+testosterone&source=bl&ots=w1Mb5pKLPr&sig=1dueKxAAZkkHJhfz7o4MrhQXI5g&hl=en&ei=T2Y3TY21AqGqhAfl4bDzAg&sa=X&oi=book_result&ct=result&resnum=3&ved=0CCoQ6AEwAg#v=onepage&q=radiation%20lower%20testosterone&f=false source] - so yes. (That could be worked out a lot quicker by googling "radiation lower testosterone") [[User:Smartse|SmartSE]] ([[User talk:Smartse|talk]]) 22:35, 19 January 2011 (UTC)

:Radiation could increase free testosterone but our article on [[Testicular Cancer]] does not mention this. Could someone please expand this, as I'm not sure of certain details.--[[User:Aspro|Aspro]] ([[User talk:Aspro|talk]]) 22:38, 19 January 2011 (UTC)

::It depends on what type of radiation, how it was applied, where it was applied, and so on. A lot of research has been performed to study the effects of ''localized'' radiation applied to the testes or to the prostate as part of a cancer treatment. Here are some relevant papers on that topic that were found using [http://scholar.google.com/scholar?q=radiation+effects+on+testosterone+levels this Google Scholar search]: [http://www.jstor.org/pss/3574084 ''Effect of Graded Doses of Ionizing Radiation on the Human Testis''], [http://www.redjournal.org/article/S0360-3016%2897%2900311-8/abstract ''Serum testosterone levels after external beam radiation for clinically localized prostate cancer''], and so on. If you are trying to evaluate medical options, you should consult a physician. Significant other research has studied the effects of ''environmental'' radiation (like if you work in a room or laboratory near radioisotopes, or near a uranium mine, or if you have a [[radon gas]] problem in your home); for example, [http://www.ncbi.nlm.nih.gov/pmc/articles/PMC1533291/ ''Environmental endocrine disruption: an effects assessment and analysis.''] We can point you to better resources, and provide better answers if you can specify your question a little more clearly. "[[Radiation]]" is a very broad term for a lot of different kinds of energetic effects from a lot of different places: nuclear radioisotopes; strong electromagnetic sources (like mobile phones); and even [[ultraviolet radiation]] from overexposure to the Sun or tanning booths. [[User:Nimur|Nimur]] ([[User talk:Nimur|talk]]) 23:45, 19 January 2011 (UTC)

== Plant hardiness - temperature versus sunlight volume ==

It's about -20C outside these days, though my apartment (apart from near windows) is around 20 degrees at all times. I have a [[Creeping Charlie]] that was placed too close to a window and began to die. However, the ambient temperature was never at or below freezing.

This made me wonder about [[plant hardiness]] and why some plants cannot tolerate super-freezing temperatures. If a tropical plant like a banana tree was given ample, full-spectrum artificial light but kept at an ambient temperature of 5~10C, would it still die? If yes, why? There wouldn't be any cell damage from freezing... [[User:The Masked Booby|The Masked Booby]] ([[User talk:The Masked Booby|talk]]) 01:14, 20 January 2011 (UTC)

:I don't know, but a conjecture is that different enzymes work best at different temperatures. Perhaps the plants that like warmth have enzymes that only function at those temperatures. [[User:Ariel.|Ariel.]] ([[User talk:Ariel.|talk]]) 06:16, 20 January 2011 (UTC)

:Yes, it's basically the same reason why humans die if their body temperature is to far away from 37°C (without freezing). The physiology of most life forms is tuned (by evolution) to certain ranges of environmental and internal variables, and if you cross the thresholds, the processes in the cells don't work anymore as they should. That's the same principle for plants, animals and even single celled organisms (some bacteria that thrive in warm shallow sea water won't grow at all or even die in cold shallow sea water, for example). --[[User:TheMaster17|TheMaster17]] ([[User talk:TheMaster17|talk]]) 10:33, 20 January 2011 (UTC)

::[[Wikipedia:Reference_desk/Archives/Science/2010_May_4#refrigerating_foods|This previous question]] covers some of the relevant points (I was posting as 131.111). Chilling can damage plants because it disrupts the cell membranes by making them solid, instead of [[fluid mosaic|fluid]] which then makes them leaky, allowing all the parts of cells to mix, whcih is not good news at all. A banana plant will certainly die at 5-10C because they have never evolved to cope with those temperatures. Ariel's point about enzymes is probably partially true, but the disrupting of cell membranes is more of an issue - unlike us, plants have to cope with a wide range of temperatures and so they have enzymes which function over wider ranges, or will have [[isozyme]]s for different temperatures. As to how some plants survive super-freezing, it's pretty complicted! There are some details at [[Ecophysiology#Temperature]] (unfortunately unreferenced, but I can vouch for its accuracy as I wrote it from lecture notes - I will try to reference it some day!) and combined these allow trees like [[larch]] in Siberia to survive liquid nitrogen, because the cells are placed in [[suspended animation]]. If you want a paper discussing this, [http://ddr.nal.usda.gov/dspace/bitstream/10113/34556/1/IND43814130.pdf this one] might be a good start. [[User:Smartse|SmartSE]] ([[User talk:Smartse|talk]]) 15:43, 20 January 2011 (UTC)

== Pregnancy and radiation-blocking smocks... ==

Here in China pregnant women wear smocks with a thin lead liner in the workplace. I can understand the need for such clothing if someone worked in an area with serious radiation potential, like an x-ray equipment factory or something. However, 99% of these women only ever sit in front of a computer doing routine tasks. I suspect this practice, like many medical habits in China, is based on no real proven science. Is there anything to gain from such clothing? I didn't see any indications in the [[Electromagnetic radiation and health]] article... [[User:The Masked Booby|The Masked Booby]] ([[User talk:The Masked Booby|talk]]) 01:17, 20 January 2011 (UTC)
:Well, considering that lead doesn't even block [[EMF]] I don't know what they're trying to achieve. I would be much more worried about exposure to lead! Even if it is sealed, what if a section becomes exposed due to wear or tear? Lead rubs of extremely easily onto hands or clothes, even just touching it and then handling food or inadvertently touching your mouth sounds like a much higher risk then sitting in front of a computer or monitor. [[User:Vespine|Vespine]] ([[User talk:Vespine|talk]]) 03:35, 20 January 2011 (UTC)
::I'd hesitate to concur -- while lead doesn't attenuate all forms of [[electromagnetic radiation]] well, it does do a great job for [[x-ray]]s, for example. This may not relate to the circumstances posed in a positive fashion (it may reveal the ignorance of Chinese women and/or their husbands), but your blanket statement is certainly incorrect on the whole. '''[[User:DRosenbach|<span style="color:#006400">DRosenbach</span>]]''' <sup>([[User_talk:DRosenbach|<span style="color:#006400">Talk</span>]] | [[Special:Contributions/DRosenbach|<span style="color:#006400">Contribs</span>]])</sup> 05:02, 20 January 2011 (UTC)
:::EMF |= electromagnetic radiation. I didn't think computers and lcd monitors are significant sources of EMR, unless you have a wifi or bluetooth transmitter or something. It's hard to find reliable sources to look this up because of all the fear mongering but my guess is there would be small amounts of things like radio waves and stuff coming out of a normal computer, but there's certainly no ionizing EMR coming out of your computer. [[User:Vespine|Vespine]] ([[User talk:Vespine|talk]]) 05:24, 20 January 2011 (UTC)
:::: It could have started with ladies worried about the X-Rays emitted from older model CRTs (Which, for all I know might still be in wide use in China.). Once these sorts of things start they usually become divorced from their rational underpinnings pretty quickly, so it's not hard to imagine that some ladies would still wear them even if they never use anything that emits anything dangerous. [[User:APL|APL]] ([[User talk:APL|talk]]) 15:30, 20 January 2011 (UTC)

== Biology vs. the Cosmetics Industry ==

''(My last question of the day...)'' My wife, like many women, spends a fair bit of money on facial cleansers and moisturizers, always used in conjunction. She also makes a big stink about my need to have empty pores on my face in order to be "clean". I am skeptical of this entire process. I've read [[Cleanser]], [[Sweat gland]], [[Acid mantle]], and [[Sebaceous gland]] and have not been able to definitively confirm or refute my contention that: '''stripping oil from your face with cleansers and then applying moisturizer is equivalent to doing nothing at all and letting your skin regulate itself, it just costs loads more.''' But again, none of those articles directly say whether this is truly necessary or not. Can anyone shed some scientific light on this? [[User:The Masked Booby|The Masked Booby]] ([[User talk:The Masked Booby|talk]]) 01:28, 20 January 2011 (UTC)
:<small>We have a refdesk rule about medical diagnoses, do we need one about marital harmony? [[User:Ariel.|Ariel.]] ([[User talk:Ariel.|talk]]) 01:33, 20 January 2011 (UTC)</small>
::<small>This is not a medical diagnosis request. This is a question about the biological/chemical function of the face and the utility of cosmetics. I am inherently distrustful of the cosmetics industry given their proven non-sense like conditioner making your hair "healthier" and would like to know if facial cleanser/moisturizers fall into the same sphere. I included the bit about my wife for color, not because I'm going to print out your replies and wave them in her face and say SEE! WIKIPEDIA SAID SO! [[User:The Masked Booby|The Masked Booby]] ([[User talk:The Masked Booby|talk]]) 01:38, 20 January 2011 (UTC)</small>
:::<small>Stuff in small print is usually for more light hearted (or off topic) comments.</small> [[User:Ariel.|Ariel.]] ([[User talk:Ariel.|talk]]) 01:42, 20 January 2011 (UTC)
:You aren't just stripping the oil off of your face, you are stripping all of the stuff that tends to get stuck to the oil, like dead skin cells, dirt, (for many women) makeup, etc. It may be quite difficult to remove, for example, dead skin cells from pores with just soap, while an [[astringent]] cleanser which dissolves the oil the skin cells are stuck to will make it much easier to get them out. Bacteria like to feed on these dead skin cells; this can be a primary cause of [[acne]]. The deal is, since you just removed all that oil, you need to replace it to prevent your skin from drying out, hence the moisturizer. In summation, it isn't the skin oils ''per se'' that need removing, its all of the stuff that is in the oil. --[[User:Jayron32|<font style="color:#000099">Jayron</font>]]'''''[[User talk:Jayron32|<font style="color:#009900">32</font>]]''''' 01:37, 20 January 2011 (UTC)
::I guess the question is, ''Does that stuff really need removing?'' Where is the science behind it? <small>(PS: Agree with comment about marital harmony.)</small> [[User:HiLo48|HiLo48]] ([[User talk:HiLo48|talk]]) 02:52, 20 January 2011 (UTC)
:::Which is a fair question. I'm more familiar with the argument against [[shampoo]]-conditioning, which is a somewhat similar arrangement: the hair is cleaned of oil (and the attached dirt) and then has a moisturizer ([[hair conditioner]]) added back on. Our article on that argument can be found at the humorously titled "[[no poo]]", though it's a pretty poor article. Western habits (and obviously Chinese ones as well) on cleanliness are hugely influenced by social expectations, to the point where the "necessity" of having to shower multiple times a day, for example, borders more on [[ritual purification]] than anything to do with an increase in health or other objective goal. As with anything else, moderation is probably the sensible option. [[User:Matt Deres|Matt Deres]] ([[User talk:Matt Deres|talk]]) 03:02, 20 January 2011 (UTC) <small>Quick aside - I got a chuckle by reading that our article on [[cleanliness]] has been flagged for "cleanup" for seven months... </small>
::::@HiLo48 for some people, probably not, but for others definately. I am a life-long sufferer of acne (I basically have the exact same skin problems at 34 that I had at 14); through years of trial and error I have found that some products work for me, and some do not, and I use those that do. Clearly, for people who have acne, skincare can be tricky business. It's one of those "if you don't have the problem, you just don't understand" sort of deals. For people without acne problems, washing daily with the bar soap they use for the rest of their body works fine. For others, however, it just doesn't cut it. --[[User:Jayron32|<font style="color:#000099">Jayron</font>]]'''''[[User talk:Jayron32|<font style="color:#009900">32</font>]]''''' 03:35, 20 January 2011 (UTC)
:::::I seem to remember discussing something similar about washing hair too, but can't find an archive link. If you're correct, it almost certainly has something to do with differences in [[skin flora]]. I found [http://www.healthmegamall.com/Articles/BabeskinArticle71.pdf this] which says "soap may reduce the normal skin flora, leading to an increased colonization of the skin with coagulase-negative [[staphylococci]]; this effect has been linked to the shift in skin pH caused by soaps. Lastly, it has been found that applying agents that specifically inhibit gram-positive cocci, such as antibacterial soap, generally increases gram-negative rods." and [http://www.ncbi.nlm.nih.gov/pmc/articles/PMC2631732/pdf/11294712.pdf this paper] tells a similar story. I think it's probably difficult to give you a definitive answer and as everyone else is ORing so will I - I think that cleansers + moisturisers change the balance of skin flora and that if you then stop using them, your skin condition detiorates because the equilibrium was being maintained by the products. For those of us who don't use such products (myself included) our skin flora is in equilibrium, and generally speaking we don't suffer from unhealthy skin. Carrying on, we obviously evolved in a world without soap, and our skin produces oils for a reason, I see no reason why you would need to replace this with something else. <sub>(Maybe you can experiment with your wife's face, only letting her clean + moisturise one side and come back with photos in a few months)</sub> Interestingly, I don't think anyone has done any [[metagenomic]] studies of skin flora, which would be a very interesting thing to do, when comparing you and your wife. [[User:Smartse|SmartSE]] ([[User talk:Smartse|talk]]) 16:12, 20 January 2011 (UTC)
::::::What about situations where skin flora is "off" in the absense of products, but correctly balanced with them, before such products were used by the person? I had acne before I started using products... Your explanation isn't consistant in that skin problems exist ''in absense'' of even the existance of skin care products. It would only make sense if skin problems didn't exist ''prior to people using skin care products''. However, that is demonstratably false. --[[User:Jayron32|<font style="color:#000099">Jayron</font>]]'''''[[User talk:Jayron32|<font style="color:#009900">32</font>]]''''' 16:29, 20 January 2011 (UTC)
:<small>[[WP:OR]] Face wash products work well. They just include sticky substance which once applied to your face you must wash your face thoroughly with lot of water to remove it, and any other dirt/oil on your face, if any, also washes out along with the water :) - [[User:Ranemanoj|manya]] ([[User talk:Ranemanoj|talk]]) 04:10, 20 January 2011 (UTC)</small>

== Calcium oxide reaction ==
calcium oxide reaction with water <span style="font-size: smaller;" class="autosigned">—Preceding [[Wikipedia:Signatures|unsigned]] comment added by [[Special:Contributions/125.99.5.139|125.99.5.139]] ([[User talk:125.99.5.139|talk]]) 09:08, 20 January 2011 (UTC)</span><!-- Template:UnsignedIP --> <!--Autosigned by SineBot-->
:{{Refq|calcium oxide}}. [[User:DMacks|DMacks]] ([[User talk:DMacks|talk]]) 11:04, 20 January 2011 (UTC)

== Is there anybody to reply me positively[[Special:Contributions/117.194.164.185|117.194.164.185]] ([[User talk:117.194.164.185|talk]]) 14:35, 20 January 2011 (UTC) ==

1.Is a photon massless?
If yes,then why does it has particle nature as a particle has a must property of mass and it also has momentum?
If no,then why it cannot be proved?

2. Is a vacuum really vacuum or not?
If yes,then why does there exist a dark energy as energy cannot be held without any medium?
If no,then what does there exist?

3. Does a string really exist?
If yes,then what is the matter the string made of?
If no,then what is the cause of producing charge in an electron?

4. Do the photons carry the energy from one place to another?
If yes,then why does the intensity of radiation decrease gradually and if an electro magnetic radiation is one kind of transverse wave,then the main property of this kind of radiation should be transfer of energy not the particle and it definitely needs a medium to traverse?

5.What is the situation before the Big Bang? Is it a cyclic process?

6. What is the way to transfer energy from one place to another?
If it is vibration of different mode which produce different kind of energy,then is it possible to construct the T.O.E. equation?

(I think I have some definite reasons to explain those things stated above. But I need a definite person to explain these mathematically. {{unsigned|117.194.164.185}}
:You asked a lot of very hard questions, so it is impossible to answer them all here. You should look at the articles on Wikipedia and ask if you have specific questions.

:1. A [[photon]] has no rest mass, though it does have [[momentum]]. Who says that particles must have mass? It's perfectly possible to conceive a massless particle.

:2. [[Vacuum#In_quantum_mechanics]] and [[Vacuum energy]] explain this better than I could. The exact nature of a vacuum is complex - however it is quite well defined in quantum theory. The idea that energy cannot be held without a medium is just a hypothesis and may not be true.

:3. [[String theory]] is still a matter of debate, and it's not clear if any version of it is true. Therefore nobody is certain what the strings are made of, or even if it makes sense to ask if they are made of anything (just as Einsteinian [[space-time]] is not made of anything, and electrons are, in standard models, made of electron without any internal structure).

:4. Photons do indeed carry energy. Intensity of radiation decreases with distance from the source because photons spread out as they travel and the further from the source you are, the fewer photons hit you (see [[inverse-square law]]).

:5. Nobody knows. See [[cyclic model]].

:6. See [[energy transfer]] for a partial answer. --[[User:Colapeninsula|Colapeninsula]] ([[User talk:Colapeninsula|talk]]) 15:12, 20 January 2011 (UTC)

::For 1, see also [[invariant mass]] ('rest mass') versus [[relativistic mass]]. Blame Einstein. [[User:TenOfAllTrades|TenOfAllTrades]]([[User_talk:TenOfAllTrades|talk]]) 15:16, 20 January 2011 (UTC)

:{{ec}}<small>Almost identical to previous response, but I've written it now and don't want to discard it.</small>
:1. If you mean "does a photon have zero rest mass ?" then the answer is yes, although the question is somewhat meaningless as a photon is never observed at rest. This does not prevent a photon being a particle.
:2. If you mean "is the vacuum empty" then the answer is no - any vacuum in our universe will contain the electromagnetic field, its quanta (photons), a large number of passing [[neutrino]]s and a "sea" of [[virtual particle]]s.
:3. If you mean the strings in [[string theory]] then we don't know whether they really exist.
:4. Yes, photons carry energy. The intensity of radiation decreases with distance because the flux of photons per unit area decreases as the area over which the photons are spread increases. For large photon fluxes the intensity appears to follow a continuous curve; for small photon fluxes the continuous model breaks down and individual photons are observed. You can recover the continuous model by averaging the photon flux over a long enough period of time.
:5. We don't know what was before the Big Bang. We don't even know if this question makes any sense.
:6. In [[quantum field theory]], you can think of energy as being carried by waves in quantum fields or by the momenta of particles associated with those fields. These are dual models - "reality" is both, not one or the other. A field does not need a medium. [[User:Gandalf61|Gandalf61]] ([[User talk:Gandalf61|talk]]) 15:40, 20 January 2011 (UTC)

::@OP: Regarding 3: What does "really exist" mean? String theory is a [[Scientific modelling|model]] just like many other models. It is useful insofar as it matches observations, and has a useful predictive power. Is an electron a little ball, a standing wave, a probability distribution? It is all of these, depending on which model you use. None of this is ''wrong'', though some models better match observation, and some are more useful in certain applications. Is light a wave or a particle? Neither, it is just ''light'', but we have wave-based models of light which are useful in some explanations (see [[diffraction]], [[refraction]], etc) and we have particle-based models of light useful in others (see [[photoelectric effect]]). If you commit yourself to "The Truth" rather than "Consistent with observations" you miss the point of science. It is perfectly fine and consistant to have multiple, accepted models of the same phenomenon, and each of these models is "real" insofar as any can be. So, stop worrying about if strings "exist" or what they are "made of" and instead focus on what the model teaches us about how the universe works. --[[User:Jayron32|<font style="color:#000099">Jayron</font>]]'''''[[User talk:Jayron32|<font style="color:#009900">32</font>]]''''' 16:00, 20 January 2011 (UTC)

:::It remains to be seen whether or not string theory is a useful model of reality capable of offering testable predictions. In this context, showing that string theory is capable of making new verifiable predictions is basically the same as showing that strings "really exist". [[User:Dragons flight|Dragons flight]] ([[User talk:Dragons flight|talk]]) 16:28, 20 January 2011 (UTC)
::::That's kinda the point; though it would be better to say that "really exist" misses the boat in that it implies only a single "right" answer; after all if strings really exist, than other models which do not contain strings, really don't exist. Having a paradigm that searches for the One Right Answer completely misses the point of science. Lets go back to the light example. Does light really and truly exist as a particle or does it really and truly exist as a wave? If you commit yourself to that mindset, you are left with three very unsatisfying results:
::::*Light is only one or only the other (observationally inconsistent; light clearly behaves like a particle in some situations, and like a wave in others)
::::*Light is both at the same time (logically inconsistent; a little ball and a mode of movement don't occupy the same class of nouns even. It's akin to claiming that something smells like the color red. It has a poetic appeal, but it doesn't make any logical sense)
::::*Light switches between the two modes, depending on the type of observation (seems better at first, but it begs the question about what light is doing when it isn't being watched.)
::::People have grappled with this concept for a very long time, the only satisfying result comes when you come to realize that there is a difference between reality and perception, and most importantly '''reality is not observable at all''' at the most fundemental level. All you can say is that your mind ''perceives'' an object or phenomena to be real, but you have no way to confirm that. Look at your keyboard. How can you confirm your keyboard exists, '''as an entity seperate from your internal, mind-created construct of what it is'''. At some point, you need to accept that as true. But that's still an unproven (and unprovable) proposition. When we deal with scientific concepts which are only observable in the second or third order, where we are further removed from the observable properties of it, these sort of problems become manifest in the situation. Back to light, rather than trying to decide what light "really" looks like, and "really is", change your paradigm to just accept that light is what it is, and is fundementally unconcievable apart from the models we make to describe it. Accept that, while light exists and is consistent, our models being by definition imperfect representations, can never fully capture what it means to be light, and let yourself be OK with that. Understand that the two competing and mutually exclusive models of light work in their own domains, are uncontradicted by other models, and so are as close to "real" as we can get, insofar as anything is "real". On that level, string theory may be "real", but also be willing to accept other, competing, and apparently mutually exclusive models as equally "real" insofar as they contradict neither logic nor observations. --[[User:Jayron32|<font style="color:#000099">Jayron</font>]]'''''[[User talk:Jayron32|<font style="color:#009900">32</font>]]''''' 16:48, 20 January 2011 (UTC)

Latest revision as of 15:56, 11 June 2024

Welcome to the science section
of the Wikipedia reference desk.
Select a section:
Want a faster answer?

Main page: Help searching Wikipedia

   

How can I get my question answered?

  • Select the section of the desk that best fits the general topic of your question (see the navigation column to the right).
  • Post your question to only one section, providing a short header that gives the topic of your question.
  • Type '~~~~' (that is, four tilde characters) at the end – this signs and dates your contribution so we know who wrote what and when.
  • Don't post personal contact information – it will be removed. Any answers will be provided here.
  • Please be as specific as possible, and include all relevant context – the usefulness of answers may depend on the context.
  • Note:
    • We don't answer (and may remove) questions that require medical diagnosis or legal advice.
    • We don't answer requests for opinions, predictions or debate.
    • We don't do your homework for you, though we'll help you past the stuck point.
    • We don't conduct original research or provide a free source of ideas, but we'll help you find information you need.



How do I answer a question?

Main page: Wikipedia:Reference desk/Guidelines

  • The best answers address the question directly, and back up facts with wikilinks and links to sources. Do not edit others' comments and do not give any medical or legal advice.
See also:

May 29[edit]

Elderly digestion[edit]

Older people have more distended intestines due to loss of muscle tone. It also means food is pushed along more slowly along the digestive tract. Does this mean that per ounce of ingested food, elderly people will extract more nutrients?

Of course, if this causes them to eat less, it may not mean more calories absorbed. Imagine Reason (talk) 09:24, 29 May 2024 (UTC)[reply]

I don't know if any of the premises of your argument are correct. But consider this; for about half a billion years our ancestors have had a digestive system. Extracting every usable bit of nutrients from our food has been under strong selection the whole time, because starvation kills and food is limited. Why then would a malfunctioning elderly system do better? Abductive (reasoning) 18:54, 29 May 2024 (UTC)[reply]
There is an evolutionary pressure to extract every usable bit of nutrients from our food. There's also an evolutionary pressure to keep the power-to-weight ratio of the digestive system high. Wasting some nutrients to keep the digestive system light may be benificial. I'm not suggesting I disagree with your conclusion. PiusImpavidus (talk) 19:54, 29 May 2024 (UTC)[reply]
The human digestive system is far from perfect in extracting calories. Imagine Reason (talk) 21:52, 31 May 2024 (UTC)[reply]

Elvish astronomy[edit]

Considering Legolas' canonical feats, what would be his naked-eye limiting magnitude, assuming seeing conditions in which the average human would get 6.0?

(The main reason for this question is that in Morgoth's Ring an Elvish name for Neptune is given. Though I suppose they might've been using palantíri as telescopes.) Double sharp (talk) 09:36, 29 May 2024 (UTC)[reply]

Perhaps elves have eagle eyes; Tolkein is silent on the issue. Alansplodge (talk) 11:15, 29 May 2024 (UTC)[reply]
He does kind of imply at least that the naked-eye limiting magnitude is greater for Elves than for Men. In The Nature of Middle-Earth, "Dark and Light" it is written: The Quendian imagination of the shape of Arda and of the visible Heaven (Menel) above it, was due to the acute minds of a people endowed with sight far keener than the human norm. Though this is for the Round World version, in which the Sun and Moon already exist from the beginning. As for quantitative figures, I guess we're stuck with the data point of Legolas counting 105 horsemen from a distance of 24 km.
Actually, it occurs to me that (perhaps more interestingly than limiting magnitudes), Elves really ought to be able to resolve the Galilean moons of Jupiter. This provides an alternate solution to the longitude problem, at least if you take the Round World versions. :) Double sharp (talk) 15:20, 29 May 2024 (UTC)[reply]
The Galilean moons would be a cinch for an elf. I had a (human) friend who could do this (he was tested on it several times). Not being particularly interested in astronomy, he only found out in his adult years that this was not usual for most people. {The poster formerly known as 87.81.230.195} 94.2.67.173 (talk) 19:42, 29 May 2024 (UTC)[reply]
Hmm, probably also the crescent of Venus should be resolvable for them. This perhaps has implications on the shapes of the Silmarils. (Although in the Round World Version, Venus already exists beforehand, and its identification with Eärendil is said to be mythologising. From the same essay I quoted: Certain stars (no doubt those we call planets) and among them especially Venus, which they called Elmō (and later mythologically Eärendil), they early observed were “wayward” and altered their places with regard to the “farstars” (fixed stars). These they called companions of the Sun and thought them quite small heavenly bodies – derived from the Sun.) Double sharp (talk) 02:31, 30 May 2024 (UTC)[reply]
The limiting magnitude can be increased with a sharper view. The sharper the view, the smaller the area of the detector (retina) on which the light falls. The background light is fixed per unit of surface area of the detector, so with the signal on a smaller area, less background competes in this area, increasing signal-to-noise. A sharper view can also help to take a faint object out of the glare of a nearby bright object; relevant to see the Galilean moons.
The other way to increase the limiting magnitude is by increasing sensitivity. No matter how sharp your eyes are, you need a couple of photons before you can see anything. The more photons you detect, the lower the relative Poisson noise. The sensitivity can be increased by (A) a better detector, detecting a larger fraction of the incoming photons; (B) larger aperture, i.e. a bigger pupil; (C) increasing integration time. Many nocturnal animals (and elves may be somewhat nocturnal) have better, more sensitive detectors, although at a price. The tapetum lucidum found in many animals reduces resolution somewhat; some species sacrificed colour vision for better low-light vision. Bigger eyes help to see better, but although Tolkien often writes that Elven eyes are keen and fair, he never writes (AFAIK) that they're big. Maybe elves can at will increase the integration time of their eyes. For humans it's fixed at several centiseconds, but if elves can boost it to a full second (they would largely loose the ability to detect motion), seeing Neptune shouldn't be too hard. Still takes a lot of dedication and patience to find out which of those tens of thousands of faint stars slowly moves, but patience is something you should have if you live forever. Ents might disagree. PiusImpavidus (talk) 20:55, 29 May 2024 (UTC)[reply]
Why would Ents disagree? They have loads of patience (though it is unclear if they are immortal or just extremely long-lived). Clarityfiend (talk) 22:23, 29 May 2024 (UTC)[reply]
Ents might disagree with the statement that elves have patience. PiusImpavidus (talk) 11:07, 30 May 2024 (UTC)[reply]
Thanks for the details! I guess I'm personally more inclined now towards explaining the Neptune thing by an Elvish invention of the telescope.
Or maybe the Valar told them where to look, noting that Neptune's magnitude is actually brighter than the most extreme reports of naked-eye viewing of stars. It would be a lot easier to find Neptune if you already know where it is, than to find which of those myriad faint stars is slowly moving. In NoME, Elvish Reincarnation implies that the Eldar were informed about isotopes by the Valar, so this isn't unreasonable in-universe. (Though finding that passage makes me amused by the thought of seeing Galadriel's NMR spectra.) Double sharp (talk) 04:23, 30 May 2024 (UTC)[reply]
In canon Men are sometimes mistaken for Elves, so there must not be any gross difference in eye size. —Tamfang (talk) 21:37, 30 May 2024 (UTC)[reply]
Assuming a comparable physiology, an Elven retina may pack more sensitive photoreceptor cells, while the lens may have better optical qualities.  --Lambiam 06:08, 31 May 2024 (UTC)[reply]

I wouldn't take it for granted that the Solar System outside of Middle Earth is the same as the real one. According to The Silmarillion Earth (or "Arda") was explicitly created by God and smaller deities with their omnipotent powers, and so were the peoples of that world (Elves, Men, Dwarves, and surely Hobbits and Ents, too). And the part that would drive mad the astronomers reading that book, the Sun and the Moon were also created by those beings... ages after the creation of Arda, and after life existed on that world. Cambalachero (talk) 13:00, 31 May 2024 (UTC)[reply]

Indeed. But Tolkien planned in his later years to make an altered Round World Version in which the Sun and Moon exist from the beginning of Arda. In that case the Two Trees simply preserve their light as it was before they were tainted by Melkor. As I quoted above, these late rewrites imply that the planets in Tolkien's world are the same ones that we have. :) Double sharp (talk) 11:05, 1 June 2024 (UTC)[reply]
Hmm, curious where you heard about these supposed intended revisions to create a lore more consistent with actual astral bodies. I'm a little skeptical without seeing a source on this, just because it doesn't seem to add up with other well-established details. Bear in mind that every element of the legendarium that bears light on the creation and cosmology of Eä was published after J.R.R. Tolkien's death. Christopher Tolkien finished The Silmarillion by trying to faithfully patch together the content (and fill in gaps himself) using an express desire to have the final work reflect what his father, the original author, would have intended. Nobody would have been in a better position than the younger Tolkien (who made the completion of the legendarium a substantial part of his own life's work and had the fullest access to the existing materials) to know what the elder intended in this respect, and he would have had every opportunity to cause the final work to reflect it, as he did with countless other details.
Further, thematically it just doesn't seem to fit: the trees are so fundamental to the cosmonogy of the legendarium, as well as its evolution and eschatology. The trees are part of the more gnostically "pure" version of the world after it was sung into existence out of the higher pleroma by the music of Ainur. The fall of the trees, though instigated by the machinations of Melkor and Ungoliant, are thematically (and arguably psuedo-naturally) the result of an inevitable and unarrestable trend of the world trending away from the direct influence of the Valar and towards a world more defined by physicality and all the ills that come with it.
All of this being the result of Melkor's discordant notes, which Eru permitted to remain a part of the song of creation. First the Valar leave Middle Earth and retire to Valinor, and as time wears on, begin making less and less in the way of even indirect influence over it. Next, the Trees are destroyed and the First Age begins. Later the world is reshapped such that Valinor is not even entirely on the same physical plane as the rest of Arda, and reachable only via the Straight Path. The elves diminish and go into the West, returning ultimately to Valinor, the magic and grace of their realms failing and converting them into mundane lands. Meanwhile the light of the trees persists in the silmarils and the phial of Galadriel, bitter sweet echoes of a purer but irretrievable age. Magic fades and Illuvatar's second (and less ethereal) children, humans, inherit the world.
You see what I mean? The idea of the world's inclination to a state defined increasingly more by a base, more purely physical state and away from the direct influence of the spiritual animus that gave birth to it is baked into the narrative and the lore, from start to finish. And the trees are the ultimate symbol of the starting point (or at least the start after the music was finished and the Ainur descended into Arda). Besides, the sun and the moon predating the round world just doesn't make much sense: a flat Arda wouldn't be able to rotate on is axis. Mind you, not to imply that there's in way to get the legendarium's cosmology to work with actual astrophyics. Which Tolkien very well knew: all indications are that this was a part of the point. But introducing the sun and the moon before the First Age just makes the discontinuities more obvious and intrusive.
All that said, very curious to see where this comes from originally. Certainly there is no shortage of matters that Tolkien went back and forth on over his decades of revisions of the relevant works, nor issues where his son had to make best-guess efforts in choosing among the disparate versions of events. But personally, I tend to doubt that Tolkien seriously considered this particular shake-up. Tonally and in terms of continuity, it just doesn't add up. -sighs a sigh of glutted satisfaction, having sucked out all of the fun the subject matter and wrecked it as surely as a giant spider sucking the magic out a world tree and poisoning it.- SnowRise let's rap 02:51, 3 June 2024 (UTC)[reply]
@Snow Rise: There is a good summary here, though the primary sources (JRRT's texts) are still in Morgoth's Ring (ed. Christopher Tolkien) and The Nature of Middle-earth (ed. Carl Hostetter). A scholarly paper about it is here.
A slightly shorter summary: pre-ROTK, JRRT produced a version of the Ainulindalë with the changes to the cosmology. Melkor seizes part of the Earth to make the Moon for his stronghold, before the Valar cast him out of it and cleanse it. For the time being, it was only an experiment.
But in the late 1950s, JRRT came to believe that the making of the Sun and Moon was too "astronomically absurd" to write in an age when most people believe that the Earth is spherical and is more or less like an island in space. So he came up with a new concept: the "Flat World Versions" are traditions that were handed down by the Númenoreans and then in Arnor and Gondor, that are inescapably blended and confused with Mannish myths. The Elves had their own lore from Valinor that was astronomically and geologically in accord with what we know instead.
JRRT describes the new cosmology across some essays published in the aforementioned collections and in an interview. The Sun and Moon were created together with the Earth, and originally they had the Primeval Light, and what Melkor did instead was corrupt them. Middle-earth was then twilit, because Morgoth darkened the Earth with clouds, such that the stars and moon were invisible and the Sun was only a dim twilight (something like the real Venus without the greenhouse effect, I might add). The significance of the Two Trees is that only there was the Primeval Light preserved, and Varda domed Valinor over to keep Morgoth's corruption out and have it only be lit by the stars. The world was also round from the beginning, but you could not circumnavigate it before the drowning of Númenor because Aman would block the way. The Númenorean Catastrophe removes the inhabitants of Aman from the physical world, though the landmass remains and becomes America after significant geographical upheaval. Thus JRRT writes in these notes Aman and Eressëa would be the memory of the Valar and Elves of the former land.
Crucially, in 1966 The Hobbit was slightly edited in accordance with this new revision: where once read In the Wide World the Wood-elves lingered in the twilight before the raising of the Sun and Moon (completely correct per the Flat World version), there now read In the Wide World the Wood-elves lingered in the twilight of our Sun and Moon. JRRT's late texts published and edited by Christopher in The Peoples of Middle-earth sometimes also imply this: the new description of Fëanor's burning of the ships specifies that it was done "in the night", and that "In the morning the host was mustered", which makes no sense in the old cosmology because the Sun wouldn't yet have first risen; and Thingol's throne room of Menelrond is supposed to be based on the domes of Varda (which don't exist before the changes). The heraldry JRRT drew in the 1960s for the House of Finwë is a "Winged Sun", which also makes more sense in the new cosmology because Finwë otherwise could not have seen a Sun that only rose after he died. So the evidence is clear that the elder Tolkien really seriously intended this change.
As for why it was not adopted by Christopher: Christopher's commentary on these texts indicates that he seems to have thought it a bad idea on the part of his father. I think Christopher made a sensible call, as going on with the intended changes would require much more editorial intervention than leaving things as they were: it does not seem that JRRT ever finished the planned rewrites. But from my perspective as a fan, the whole idea is fascinating and allows the amusing conceit of trying to figure out what was really going on behind the scenes, which is why I asked the initial question about how astronomically plausible the reworking was in one aspect (the visibility of Neptune). Double sharp (talk) 03:45, 3 June 2024 (UTC)[reply]
Fascinating! Just when I thought the paratextual narrative of this particular piece of literature couldn't get more complicated! Personally I like the ultimate approach adopted (even beyond the concern you rightly point out of it requiring more intervention--I still think I prefer the narrative irrespective of whether you hypothesize a scenario in which J.R.R. could make the adjustments himself). The Age of the Trees being treated as literal (or at least facially literal from the perspective of the Silmarillion/legendarium's narrative; all of the books are presented to varying degrees as possibly unreliable ancient historical text, after all) just feels very at home with both the broader cosmogony as well as the thematics of the overall work. But then, I have revisited those books so many times over the years (at least compared to my once, maybe twice and done policy for most literature: soooo many things to read!) that I may have some bias! SnowRise let's rap 16:54, 3 June 2024 (UTC)[reply]
@Snow Rise: I kind of think of JRRT's problem as being that in seeking increased verisimilitude, he'd gone from wanting to write a mythology to wanting to write a science fiction novel. Only, instead of starting a new science fiction novel and leaving the mythology as it was in its almost-complete state, he ended up trying to turn his mythology into a science fiction novel, with predictable results. But I suppose the whole narrative was too dear to his heart for him to come up with another like it, just like the Silmarilli were to Fëanor. Which is understandable: I share your obsession, and how much greater must it have been for the author? :)
With that said: even in this take, the Age of the Trees would still be literal in the sense that there were luminescent trees in Valinor that got killed by a giant spider. They would only not be literal in the sense that the Sun would still be shining (in its form post-tainting by Melkor) over Middle-earth. Only in the northwest (over Beleriand) is there a volcanic winter due to the fumes of Thangorodrim – and from that, we can see where the idea of the Sun first rising later comes from as an in-universe myth. Double sharp (talk) 10:42, 4 June 2024 (UTC)[reply]

May 30[edit]

Volume of honey in a bee nest[edit]

What is the average volume of honey in a bee nest in the wild? I was able to find information on the average volume of a bee nest, but I know that not all of that volume is honey, of course. Thank you! HeyArtemis (talk) 07:49, 30 May 2024 (UTC)[reply]

It varies alot. Honey#Production has a number for Apis mellifera. Sean.hoyland (talk) 08:39, 30 May 2024 (UTC)[reply]

Does light decay?[edit]

Let's say that an object, such as a star, emits a beam of light and it moves across the space. It goes at the speed of light and, unless it reaches an opaque object, it would keep going... for how much time? Forever? Or is there a point when light would simply dimish and disappear? Cambalachero (talk) 19:53, 30 May 2024 (UTC)[reply]

No.
It may react with matter, if it encounters any. A flux of many photons will spread out to a larger volume and so the intensity (number of photons passing through an area) will diminish in accord with the inverse square law. But light passing through a vacuum does not 'decay' or have a limit on its range.
BTW, this theory that light can only travel a few thousand miles before 'running out' is part of flat earther canon for some models, as an explanation of how nighttime happens. But then they're flat earthers. Andy Dingley (talk) 20:11, 30 May 2024 (UTC)[reply]
The idea of tired light used to be somewhat popular but has been entirely discarded by now. --Wrongfilter (talk) 20:22, 30 May 2024 (UTC)[reply]
I'm afraid it is a zombie idea.[1]  --Lambiam 05:58, 31 May 2024 (UTC)[reply]
I guess the apparent reality that things in motion continue in motion forever by default, if unperturbed, seems a bit unnatural when you live in a macroscopic world. Sean.hoyland (talk) 07:32, 31 May 2024 (UTC)[reply]
Correct. If that wasn't the case, we wouldn't be able to see stars that are too far away. We are currently seeing stars so far away that they aren't just stars. They are galaxy or similar objects so far away that they show up as one little blob of light. The limit is not how far light can travel before pooping out. It is how old the stars are. For example, a 100 year old star that is 90 light years away would not be visible becuase the original light hasn't reached us yet (ignoring the complication of direction of relative movement). So, the argument is that the night sky should be flooded with light from all directions. And, it is. It only looks black in a relative sense. 12.116.29.106 (talk) 14:51, 4 June 2024 (UTC)[reply]
I imagine it would be much worse than not being able to see stars. If excitations of quantum fields got tired we would be in big trouble. Sean.hoyland (talk) 15:05, 4 June 2024 (UTC)[reply]

May 31[edit]

Is redshift calculated differently for different spectra?[edit]

I ask because I came across an article, TXS 1545-234, in the course of regular gnoming. The article claims this radio galaxy to be one of the most distant known objects, but its redshift is only around z = 2.754 (which I take to be measured from radio emissions). It does appear in this source (|date= at least 2006), and I'm wondering whence the claim of such great distance paired with such pedestrian redshift. I'm not able to understand our article Redshift. Also, if anyone has any ideas about how to de-orphan the article linked, please do have at. Folly Mox (talk) 11:31, 31 May 2024 (UTC) edited 11:52, 31 May 2024 (UTC)[reply]

It is certainly not one of the most distant galaxies known. The author of the article, @Galaxybeing:, should explain why they think it is. --Wrongfilter (talk) 12:12, 31 May 2024 (UTC)[reply]
They appear to have made a similar claim at MRC 0406-244 (z = 2.44) although thankfully not at another recent creation, QSO J0100-2708 (z = 3.52). Folly Mox (talk) 12:57, 31 May 2024 (UTC)[reply]
The redshift, a dimensionless quantity, is the same for the whole spectrum emitted by an object. Compared to TXS 1545-234, JADES-GS-z13-0 is thought to be more than three times as far away from us.  --Lambiam 12:37, 31 May 2024 (UTC)[reply]
Thanks both for your answers, and confirming my suspicion that the claim was merely incorrect. Folly Mox (talk) 13:00, 31 May 2024 (UTC)[reply]
A redshift of 2.754 is pretty far away, but stating it's one of the most distant known objects is overstating it a bit. Distances to far-away galaxies are normally expressed in redshift, as redshift is directly observable, in contrast to distance, which depends on a model of the expansion of the universe. But when using redshift as a distance measure, one has to keep in mind that it's highly non-linear. Also, distance is a bit of a strange concept when dealing with these cosmologically distant objects. Are we talking about the distance today, or at the time the light was emitted, or the distance travelled by the light? That last at least has some relevance as it translates to the time that the light has travelled and therefore when it was emitted. JADES-GS-z13-0 may be several times farther away today than TXS 1545-234, but most of that is thanks to the expansion of the universe after the light was emitted. When the universe was young and small (although still infinite), it expanded fast in absolute numbers (percentage per year). In light travel distance, the difference isn't so much.
These objects can be studied to learn more about the early universe. For that, knowing the distance to us is not so important; we want to know about the distance (or time) to the Big Bang. At some point, distances (times) to the Big Bang are known more accurately than distances (times) to us. In any case, the redshift tells us immediately that the universe expanded by a factor of five between the times when the light of JADES-GS-z13-0 was emitted and when the light of TXS 1545-234 was emitted. That puts JADES-GS-z13-0 a lot closer to the Big Bang, although only a small fraction farther from us. PiusImpavidus (talk) 11:05, 1 June 2024 (UTC)[reply]
On a tangent but an amusing use (abuse?) of "red shift": chemists use red shift generically to mean "moves to lower energy". Even in the IR part of the spectrum, the term red shift would be used to describe the shift of a band to lower E, say 2000 to 1950 cm-1. This language is of course strange because, formally speaking, a shift toward red for an IR band would mean a shift to higher E. Just sayin'.--Smokefoot (talk) 17:11, 1 June 2024 (UTC)[reply]
Radioastronomers apply the same "abusive" terminology. We see this linguistic phenomenon also in uses of the verb "to dial", as in the advise to "dial 9-1-1 for any emergency" given to users of smartphones with touch screens. (Using the original rotary sense of the verb in connection with the casual parlance of "butt dial" results in the unfortunate mental image of Giuliani twerking.) Other examples are referring to cotton bed sheets as "linens", or (in the US) to stainless-steel knives, forks and spoons as "silverware", and the computer-graphics terminology calling a screen region a "canvas". I think there is a learned term for this phenomenon if the sense of a word getting abstracted from the physical embodiment after which it was originally named.  --Lambiam 05:29, 2 June 2024 (UTC)[reply]
JADES-GS-z13-0
Astronomers talk about redshift if it goes to longer wavelengths, in radio, IR, UV etc., and about blueshift if it goes to shorter wavelengths, in radio, IR, UV etc. Things can also redshift past red. See how red that distant object in this picture is? It's a feature coming from UV, shifted to IR. The Lyman-alpha absorption line is at 121.6 nm, here broadened into a Gunn–Peterson trough, redshifted to 1.6 μm, between the F150W and F200W filters of the camera in JWST. It's how they make a first estimate of the redshift, based on broadband images. A precise number follows later from spectroscopy, but takes far more observing time, so this is only done for the most promising targets. PiusImpavidus (talk) 10:18, 2 June 2024 (UTC)[reply]

Physics problem[edit]

A shop sign is made of a panel that protrudes slightly from the wall on which it is hung, forming an angle of 5° with it. It is 0.74 m tall and has a mass of 8.9 kg. The upper side of the panel is attached to the wall by two cables, one from the right side and one from the left side. Find the tension of the 2 cables 78.211.54.11 (talk) 19:40, 31 May 2024 (UTC)[reply]

Original in Italian, now edited by IP to English.Translates as: "A shop sign consists of a panel that protrudes slightly from the wall on which it hangs, forming an angle of 5° with it. It is 0.74 m high and has a mass of 8.9 kg. The upper end is attached to the wall by two cables, one on the right side and one on the left side. Find the tension in the 2 cables."
IP editor: as it says at the top of this page, we don't answer homework questions and what we do answer should preferably be asked in English. Mike Turnbull (talk) 19:46, 31 May 2024 (UTC)[reply]
Asking homework questions also causes tension. ←Baseball Bugs What's up, Doc? carrots→ 01:15, 1 June 2024 (UTC)[reply]
Doesn't the tension depend on the angle the cables (idealized as straight line segments) make with the panel?  --Lambiam 02:53, 1 June 2024 (UTC)[reply]
Correct. The answer to the question as asked is a curve of tension vs cable length or vertical location. Greglocock (talk) 22:56, 1 June 2024 (UTC)[reply]
Is it like this? Graeme Bartlett (talk) 22:42, 1 June 2024 (UTC)[reply]
or is it like this? Graeme Bartlett (talk) 22:22, 1 June 2024 (UTC)[reply]
like this but not equilateral

In my interpretation of the problem – which is not necessarily the intended one – the lower edge of the panel is attached to the wall and the panel can rotate along that edge, like in the "or is it like this?" diagram, which has an exaggerated thickness for the panel. Unlike that diagram, the panel does not stick out at a 90° angle but is standing almost upright. Also, the cables are attached to the upper edge. So it is more like the situation here to the right, but instead of a bottom 60° angle we have a 5° angle. Not enough info has been given to determine the other angles.  --Lambiam 04:45, 2 June 2024 (UTC)[reply]
P.S. Another way to frame the problem could have been in terms of a drawbridge.  --Lambiam 04:53, 2 June 2024 (UTC)[reply]

The answer is 42 (glorps). I'll leave it to you to figure out the conversion factor to mks. Clarityfiend (talk) 12:06, 3 June 2024 (UTC) [reply]
                |
      		|'.
		|  '.
		|    '.
		|      '.   
		|        '.
		|          ::
		|         ::
		|< 5deg >::
		|       ::
		|      ::
		|     ::.
		|    :: .
		|   ::  .
		|  ::   .
		| ::    .
		|::     V
		|*      8.9kg

Equate turning moments at *
                      clockwise  =  anticlockwise
 8.9 kg x sin(5 deg) x 0.74m / 2 = t/2 x 0.74m
                         where t = tension in each of 2 cables
                                 = 8.9 sin(5 deg)
                                 = 0.775686... kg

Philvoids (talk) 13:16, 3 June 2024 (UTC)[reply]

Shouldn't there be a sine or cosine factor in the anticlockwise term? Imagine the cables being attached to the wall very close to the *. Consider dU/dL, where U is potential energy of the panel and L is the length of the cables.  --Lambiam 15:27, 3 June 2024 (UTC)[reply]
Although not accurately shown in the ASCII sketch my calculation presumes that the two cables are at right angles to the sign, are parallel and are fastened to the wall separately at two points. This minimises the cable lengths and tensions. Philvoids (talk) 10:28, 4 June 2024 (UTC)[reply]
The homework problem did not specify where the cables are attached or that the tension should be minimized, making it unsolvable for the average student. For an old-fashioned drawbridge over a castle moat, as in the animation, a better choice is to place the attachment point of the cables at a distance from the hinge equal to the length of the bridge, making the triangle isosceles. Otherwise, the tension grows without bounds as the bridge near a vertical position.  --Lambiam 15:37, 4 June 2024 (UTC)[reply]
I could not find an ASCII symbol for a hinge but installing one at the base of the sign is a well thought mechanical improvement that can stop the thing flapping about in a wind. This reference desk cannot grant permission to construct a drawbridge with walkway at this location and the shop owner of the wall would likely protest at your plan to cut a hole for passage. Unless he is a herder of sheep or other small animals and himself less than 74cm tall and sees some advantage. The OP doesn't ask for the cable tension when the sign/bridge is vertical but we can say there are two cases: Case #1: The hinge is broken or absent. t = 8.9/2 = 4.45 kg plus distributed weight of the cable itself; Case #2: The hinge takes the weight, t = 8.9 sin (0) so both cable lengths and tensions are zero. In that case it would be simpler just to nail the sign to the wall. Or put out a call to Banksy who can save us the cost of the sign, usually upset someone and greatly increase the value of the wall. Philvoids (talk) 18:39, 4 June 2024 (UTC)[reply]
For all we know, there is already an opening behind the sign for letting the proprietor's homing pigeons in. If they are equipped with an RFID chip, the sign can be let down automatically on their arrival.  --Lambiam 05:39, 5 June 2024 (UTC)[reply]
Pigeon RFIDs can be compromised by malicious Yinpterochiroptera and Yangochiroptera. Members of this notorious "YinYangBat Gang" equipped with ultrasound-to-UHF converters can mount Spoofing attacks on the shared ID frequency. An ad hoc aposematic remedy such as a wall painting of a cat with the sign text "I EAT BATS" does not alone give security. To this end all pigeons must be urged to register a pass-coo that is less obvious than the too common "Coo Coo". Philvoids (talk) 10:28, 5 June 2024 (UTC)[reply]

June 2[edit]

Why females produce androgens[edit]

If human embryons of both sexes start off from a female blueprint and given that females lack the male Y chromosome, how it came that women also produce androgens (even if in small quantity), with related limb and facial hair? 212.180.235.46 (talk) 19:59, 2 June 2024 (UTC)[reply]

The article you linked says that the ovaries also produce androgens. ←Baseball Bugs What's up, Doc? carrots→ 22:33, 2 June 2024 (UTC)[reply]
Biological systems weren't built by any sort of logical designer. They in no way resemble a computer program, a computer, or, for that matter, anything else in the universe. In the case of androgens, the article mentions that androgens are the precursors to estrogens. Males need estrogens too, btw. All these are steroids, which are fundamental to life and are derived from cholesterol. Abductive (reasoning) 23:18, 2 June 2024 (UTC)[reply]
"Biological systems weren't built by any sort of logical designer." Which is why so-called "intelligent design" is just nonsense. The "design" is emphatically not "intelligent". --User:Khajidha (talk) (contributions) 12:07, 3 June 2024 (UTC)[reply]
I'm not sure biology and computing are as far apart as they used to be now that systems essentially create the gigantic opaque function that transforms input to output themselves in response to their environment/what they have seen and the objectives. Generative adversarial networks for example seem a bit closer to biology than systems used to be. Sean.hoyland (talk) 13:06, 3 June 2024 (UTC)[reply]
Not in a billion years. Abductive (reasoning) 17:51, 3 June 2024 (UTC)[reply]
The technology for synthetic biology, still in its infancy, is advancing with large strides. Whether you like it or not, sooner rather than later it will become possible to design and create complete viable and functioning biological organisms.  --Lambiam 06:40, 4 June 2024 (UTC)[reply]
Further, the idea that it is so complicated that nobody understands what it does because it thinks for itself is a farce. No matter what is being used for the computing hardware, be it electronic or biological, the mechanism of operation is very well understood by the engineers who developed it. It just sounds cool to say that it is beyond comprehension. It doesn't sound cool to say that the engineers understand it very well and could trace input through to the output if they wanted to, but simply don't care to do because they have other things to work on. 12.116.29.106 (talk) 14:42, 4 June 2024 (UTC)[reply]
Tracing the input through to the output is IMO not a helpful concept. Not only do we not know why the most advanced chess or go playing programs make certain surprising moves, but it is not even clear what it means to "understand" why they did this. The computing platform performs a calculation with a certain outcome. The engineers can perform the same calculation by hand, or using abaci, and if they make no mistake they may arrive at the same result in a few billion years: 42. But can they say more than that the answer is 42 because this is the consequence of the rules applied to the input? They knew that already. If someone wants to know why it is the consequence, they can tell them to repeat the calculation.  --Lambiam 15:51, 4 June 2024 (UTC)[reply]
IP, LLMs for example involve activations in very high dimensional spaces. Trying to map those activation patterns to things we can understand, like concepts etc., is the whole field of interpretability, and it is in its infancy (and safety critical). Engineers are still far from understanding why input A to a model is transformed into output B. If this is something that interests you, have a look at the work being done in Anthropic's lab. Sean.hoyland (talk) 17:16, 4 June 2024 (UTC)[reply]
Overgrown Markov Chain models are pretty much useless in any technical field. BTDT Greglocock (talk) 04:48, 5 June 2024 (UTC)[reply]
I guess that's one of the reasons why "Attention Is All You Need" turned out to be such a great title for a paper. Sean.hoyland (talk) 08:39, 5 June 2024 (UTC)[reply]

June 5[edit]

Prostheses[edit]

what are the matrials uesed in implantable artificial prostheses, such as artificial hearts and small-diameter blood vessels; in the engineering of living tissues ? -- 196.153.184.240

Heart valves usually made from pyrolytic carbon. https://www.ncbi.nlm.nih.gov/pmc/articles/PMC10034107/#:~:text=Mechanical%20valves%2C%20usually%20made%20from,stroke%20(3%2C%204). 41.23.55.195 (talk) 06:13, 5 June 2024 (UTC)[reply]
Many different materials are used depending on requirements. For details see prosthesis and the many links therefrom.Shantavira|feed me 15:30, 5 June 2024 (UTC)[reply]

June 6[edit]

Health risk of taurine consumption[edit]

Realistically, how worried should people be about the health risks of consuming taurine as a supplement? I only ask because half the literature says they find it helpful and beneficial at some unknown dosage, while the other half says it is potentially carcinogenic and could contribute to colon cancer. As a layperson, I find this very confusing. Some of the literature says it could be simply a matter of dosage, but nobody seems to know what the safe or harmful limits are. Can anyone offer some risk analysis devoid of emotion? Should we avoid anything with taurine in it, or not worry at all about it? Viriditas (talk) 01:27, 6 June 2024 (UTC)[reply]

As far as Wikipedia is concerned, WP:MEDRS applies. That means that the minimum quality level for WP:RS making medical claims are systematic reviews indexed for MEDLINE (there are some exceptions from this indexation, but generally speaking MEDLINE is the gold standard). tgeorgescu (talk) 03:29, 6 June 2024 (UTC)[reply]
Our article states that there is no good clinical evidence that taurine supplements provide any benefit to human health, Why pay for useless supplements? The human body naturally produces a large amount of taurine, far more than one can reasonably take in as a supplement. There is increasing evidence that taurine actually plays a role in preventing cancer.[2] Any carcinogenicity of supplements can only be due to their being fake, or a lack of quality control in their production.  --Lambiam 07:23, 6 June 2024 (UTC)[reply]
The underlying issue is that large doses of taurine are added to energy drinks. Nobody seems to know why. A current study is looking at an association between energy drink consumption and the rise in colon cancer in young adults.[3] Viriditas (talk) 08:23, 6 June 2024 (UTC)[reply]
Even disregarding any potential carcinogenic risks, there are enough studies that show damaging health effects of high consumption levels of energy drinks.[4][5]  --Lambiam 18:28, 6 June 2024 (UTC)[reply]
¼ liter of my favorite energy drink has 80 mg caffeine, while adults usually consume up to 400 mg caffeine per day. And I use the energy drink totally without sugar. tgeorgescu (talk) 18:59, 6 June 2024 (UTC)[reply]
The energy drink thing may be related; most energy drinks contain vitamin B12. There is a concern that (mega)dosing B12 in excess of daily requirements carries with it a slight increase in risk of cancer due to B12 containing cobalt, which is both a heavy metal and has a trace of radioactive cobalt-60. Abductive (reasoning) 20:25, 6 June 2024 (UTC)[reply]
Thank you, that's helpful. I just read the comments by the epidemiologist on that topic, and while my reading might be flawed, they seemed to indicate that dosage and tobacco smoking played a significant role in the risk. Viriditas (talk) 22:02, 6 June 2024 (UTC)[reply]
Real skepticism cuts both ways. Taurine is in human breast milk, evidence of benefit of at least one energy drink exceeding risk. :-) But our article has imho excessive, even dubious doubt in that section on its conditional essentiality or benefit for infants, and thus its common use in another energy drink. Doubt that appears to stem from OR or opinion rather than the source, which says e.g. "Thus the new data provide further support for the view that taurine is a conditionally essential nutrient for the preterm infant" & that ethical considerations seem to prevent further research.John Z (talk) 03:57, 7 June 2024 (UTC)[reply]



June 9[edit]

Weigeltisaurus species clarification[edit]

Is this Weigeltisaurus reptile in this family Rhynchocephalia. Would that statement be true to say. Its for this article Johannes Weigelt. There is source that states it but I don't know how accurate it is. scope_creepTalk 21:23, 9 June 2024 (UTC)[reply]

While Weigeltisaurus flourished in the Late Permian, our article on the order Rhynchocephalia states that the oldest record of the group is dated to the Middle Triassic. According to this chronology, a gap of several million years separates them.  --Lambiam 07:21, 10 June 2024 (UTC)[reply]
Right. I'll leave the Rhynchocephalia bit out and only mention the Weigeltisaurus bit, since I don't understand it. Thanks @Lambiam: scope_creepTalk 08:52, 10 June 2024 (UTC)[reply]

June 11[edit]

Can we agree on what the rule of the Council of Nicaea actually was?[edit]

  • Transferred from Humanities desk

Let me highlight Jack's comment above:

...What is or is not legal is a matter for courts or legislatures to determine. Not the media, not individual partisan politicians, not the man in the street, not the reasonable man, and not random commentators (on Wikipedia or anywhere else). -- 22:23, 6 June 2024

So why is Wikipedia still saying Wedding of Prince Harry and Meghan Markle#Preceding private ceremony "The Church of England sources commented that this was not a legally recognised marriage ceremony, which requires two witnesses", citing journalist and radio commentator Camilla Tominey, who screamed down the telephone I WILL NEVER WRITE THE STORY that Camilla and Charles' wedding ceremony was a "non-qualifying ceremony" (i.e. void), following up with a stream of invective which only ended when she was cut off? And as the words "Registrar General" are eiusdum generis with the list of occupations provided by Jack, why is the linked article still saying "the Registrar General, Len Cook, determined that a civil marriage would in fact be valid"? The articles are so full of editorialising that it is best not to read them and go to [6] and [7] instead. Needless to say the "impediments to marriage" listed in the licence do not apply to the marriages of non-royals, because they were abolished by the Marriage Act which specifically states that its provisions have no applicability whatsoever to royal marriages. 92.25.129.245 (talk) 16:45, 8 June 2024 (UTC)[reply]

Readers may be interested in a dream I just had. Following the inception of the discussion (Wikipedia:Reference desk/Humanities#June 7) about al-Biruni and the Byzantine calendar, before I went to bed last night (30 James, the day before the date marked with this year's "golden number" (11), 1 Eloise), I looked for the new moon and saw a very slender crescent low in the north-west. In the dream, I was at work and was told that Camilla Tominey and another presenter had arrived to interview me. The male presenter remained outside and Camilla came in. She was aggressive and incoherent, and after a while an aide entered and bundled her out. I heard him say as he took her away "You're not on the programme any more. We're changing the presenter." Then the other presenter came in and said "You're going to be on the programme - ATV." As we left, he commented "You're casually dressed." I said "Shall I go home and change into something more formal?" and he said "We'll dress you at the studio." Then he asked an assistant "She's carrying a handbag, take it from her." I thought "This is for the general election." Outside there were a number of vehicles. Beside one of them a woman staff member was restraining a short man with his hands behind his back. I woke up at this point and went downstairs. It was 1:30 AM. I switched on the radio and this song began playing: [8]. 92.12.79.187 (talk) 15:42, 9 June 2024 (UTC)[reply]
As a matter of historical interest, one grouse the Muslims had about the imposition by the Caliphs of the new, unintercalated Islamic year for administrative purposes to replace the 365-day Zoroastrian year previously used in Persia was that it was shorter, which meant that the land tax would come round before the harvest had been collected! The Caliphs relented and reinstated the araji (land-tax) year for fiscal purposes.
But then another problem presented itself. The Muslim era began in AD 622 with the Hegira (flight) - the Persians later fixed the beginning of this era on Friday, 19 March. The Zoroastrian year needed periodic readjustment because it was itself on average six hours shorter than the Julian. Al-Biruni notes that there was a double adjustment of the start of the araji year during the reign of Yazdegerd I (AD 399-420). An araji era was introduced dating from AD 621. In a further recalibration, the Yazdegerdi era, still used by the Zoroastrians (Parsees), dates from the accession of Yazdegerd III on 16 June AD 632, so the Yazdegerdi era is eleven years behind the araji. In AD 895 there was another double readjustment of the start of the araji year. It moved from 1 Frawardin (12 April) to 1 Khordad (11 June), referred to as 11 Haziran (I think the Turks call June by this name to this day).
Today's Byzantine date (Monday, 2 Eloise), marks the imminent arrival of the Jewish Feast of Weeks (Pentecost), observed on the 6th of the corresponding Jewish month. It is barred from falling on Tuesday, Thursday or Saturday, and slots in this year this coming Wednesday (4 Eloise). Orthodox Whitsun is not till Sunday, 23 June (15 Eloise), because of the rule that Orthodox Easter falls on the Sunday after the Wednesday after the date of the Paschal Full Moon (14 Miri) between 1800 and 2099 inclusive, a sharp contrast with the Roman Catholic Church (but not it's Byzantine arm), which likes to time its Easter festivities to fall in the week after the "Purim Full Moon". The Orthodox consider the Catholics to be in error and they are right - the Council's direction was that the Easter full moon must not precede the equinox, but in 1582 Gregory XIII directed that it must be the first after the equinox. This led to his calendar being banned in Orthodoxy. 2.30.124.132 (talk) 15:43, 10 June 2024 (UTC)[reply]
Some common Serbian names feature in the story. This article [1] includes the following points:
  • In addition to the issue of calendar reform, the Council of Constantinople (1923) also discussed possible union with the Anglican church and second marriage for priests
  • Vladimir Dimitrijevic, a conservative author, said the Council of Constantinople was among the greatest mistakes of the Orthodox Church in the 20th century

The author is Jovan Trpkovic. An article by M S Dimitrijevic and others[2] includes these points:

  • The Serbian delegation came to the Congress with a proposition for calendar reform authored by Maksim Trpovic. He proposed the intercalation rule that the secular years in centuries which when divided by 9 have remainders of 0 or 4 will be leap years
  • The general opinion of the participants was that the better solution was to retain the Julian calendar and only delete thirteen days...

The information was duly sourced and added to Julian calendar. This led to one editor demanding that the contributor be banned from Wikipedia for "inserting false information into articles" because "if thirteen days are excised from the Julian calendar it is no longer Julian." He further demanded that articles be pre-emptively semi-protected to prevent editors removing any unsourced falsehoods added by others. His campaign was successful and false information, for example that the Greek government introduced the Gregorian calendar in 1923, is embedded in articles all over Wikipedia. The original Serbian proposal was in fact a proposal that the calendar already legislated by the Greek government should now be adopted by the Orthodox Church. As to where the Greek government got its calendar from, it's described here[3] although Trpkovic (who proposed it in 1900) denied all knowledge of Barnaba Oriani's calendar. Another example is the claim that "The reform of the calendar was authorised by a canon of the Council of Trent in 15.." (the last two digits of the date of the alleged canon are something of a movable feast, since they change periodically). 92.19.71.221 (talk) 15:14, 11 June 2024 (UTC)[reply]

References

  1. ^ Trpkovic, Jovan (3 May 2024). "Orthodox Easter: calendar question continues to split the Church". Retrieved 11 June 2024.
  2. ^ Dimitrijevic, M S; Theodossiou, E Th; Mantarakis, P Z (2008). "Milutin Milankovic and the reform of the Julian calendar in 1923". Journal of Astronomical History and Heritage. 11 (1): 50–54.
  3. ^ Oriani, Barnaba (1785). De usu fractionum continuarum ad inveniendos ciclos calendarii novi et veteris. In: Appendix ad ephemerides anni 1786. Milan. pp. 132–154.